Anda di halaman 1dari 143

BUSINESS COMBINATION

1. On December 2015, Killua Ltd. acquired all the assets and liabilities of Gon Ltd. with Killua Ltd. issuing 100,000
shares to acquire these net assets. The fair value of Gon Ltd.s assets and liabilities at this date were:

Cash P50, 000


Furniture and Fittings 20, 000
Accounts Receivable 5, 000
Plant 125, 000
Accounts Payable 15, 000
Current Tax Liability 8, 000
Provision for annual leave 2, 000
The financial year for Killua Ltd. is January- December.
The fair value of each Killua Ltd. share at acquisition date is 1.90. At acquisition date, the acquirer could only
determine a provisional fair value for the plant. On March 1, 2016, Killua Ltd. received the final value from the
independent appraisal, the fair value at acquisition date being P131, 000. Assuming the plant had a further five
year life from the acquisition date.
The amount of goodwill arising from the business combination at December 1, 2015 ?
a. P15, 000 c. P5, 000
b. 9, 000 d. 0

ANSWER: B

Consideration transferred (100, 000 x 1.90) P190, 000


Less: Fair Value of net identifiable assets acquired
Cash P50, 000
Furniture & Fittings 20, 000
Accounts Receivable 5, 000
Plant 131, 000
Accounts Payable (15, 000)
Current tax liability (8, 000)
Liabilities (2, 000) 181, 000
Goodwill P9, 000

2. The E. Vendivel Company acquired the net assets of the Vivar Company on January 1, 2015 and made the
following entry to record the purchase:
Current Assets 100, 000
Equipment 150, 000
Land.. 50, 000
Buildings. 300, 000
Goodwill. 100, 000
Liabilities. 80, 000
Common Stock, P1 par. 100, 000
Paid-in capital in excess of par 520, 000

Assuming that the additional shares on January 1, 2017 would be issued on that date to compensate for any fall
in the value of E. Vendivel common stock below P16 per share, the settlement would be to cure the deficiency by
issuing added shares based on their fair values on January 1, 2017. The fair price of the shares on January 1,
2017 was P10.

What is the additional number of shares issued on January 1, 2017 to compensate for any fall in the value of the
stock?

a. P160, 000 c. 60, 000


b. 100, 0000 d. 10, 000
ANSWER: C
Deficiency (16-10) x 100, 000shares issued to acquire. P600, 000
Divided by: Fair value of the share 10
Additional number of shares to issued. P60, 000

3. X Company acquires all of Y Company in an acquisition properly accounted for as an asset acquisition. X
issues 80,000 shares of common stock with a fair value of P8,000,000 for Ys net assets. The fair values of Ys
assets and liabilities approximate their book values, except Y has customer lists valued at P3,000,000 that are not
reported on its balance sheet, and its plant assets are overvalued by P5,000,000. Here are the balance sheets of
X and Y prior to the acquisition:

X Company Y Company

Assets P30,000,000 P10,000,000

Liabilities P16,000,000 P 6,000,000

Common stock, $1 par 1,000,000 100,000

Additional paid-in capital 9,000,000 2,900,000

Retained earnings 4,000,000 1,000,000

P30,000,000 P10,000,000

How much goodwill is recognized for this acquisition?

a. P 2,000,000
b. P 3,000,000
c. P 6,000,000
d. P 11,000,000

ANS: C

Cost P8,000,000
Fair value of net assets acquired
Reported assets P 5,000,000
Customer lists 3,000,000
Liabilities (6,000,000) 2,000,000
Goodwill P6,000,000

4 .P acquires all of the voting shares of S by issuing 500,000 shares of P1 par common stock valued at
P10,000,000. Included in the agreement is a contingency guaranteeing the former shareholders of S that
P's shares will be worth at least P18 per share after one year. If the shares are worth less, P will pay the
former shareholders of S enough cash to reimburse them for the decline in value below P18 per share. P
estimates that there is a 5% chance that the stock value will be P16 at the end of one year, and a 95%
chance that the stock value will be P18 per share or higher. A discount rate of 10% is appropriate. What
is the value of the stock price contingency at the date of acquisition?

a. P 1,000,000
b. P 45,455
c. P 50,000
d. P 863,636

ANS: B

Rationale: [(P18 - P16) x 500,000] x .05 = P50,000/1.10 = P45,455

5. P purchased all of the outstanding shares of S for P1,300,000 at a time when the underlying book value of
S was P1,200,000. S's assets and liabilities consist of the following:

Fair value Book value

Cash, receivables P250,000 P250,000

Inventory 360,000 380,000

Equipment 900,000 600,000

Liabilities 30,000 30,000

The gain on acquisition is:

a. P140,000
b. P180,000
c. P220,000
d. P260,000
ANS: B

Rationale:

Cost P1,300,000

Book value 1,200,000

Excess of cost over book value P 100,000

Excess

Inventory P(20,000)

Equipment 300,000 280,000

Gain P 180,000

For questions 6-7:

P Company acquired all of the net assets of S Company. The balance sheet of S Company immediately prior to
the acquisition, along with market values of its assets and liabilities, is as follows:

Accounts S Company
book value market value
Current assets P 800,000 P 1,000,000
Plant & equipment (net) 28,000,000 35,000,000
Patents 100,000 2,000,000
Identifiable intangible: brand names 0 13,000,000
Skilled work force 0 4,000,000
Goodwill 200,000 700,000
Liabilities 21,000,000 20,000,000
Common stock, $10 par 2,000,000
Additional paid-in capital 3,000,000
Retained earnings 3,100,000

6. P Company pays P40,000,000 in cash for S Company, in an acquisition properly reported as a statutory
merger. P records goodwill of:

a. P18,000,000
b. P17,300,000
c. P 9,000,000
d. P 4,300,000

ANS: C
Rationale: P9,000,000 = P40,000,000 (P1,000,000 + P35,000,000 + P2,000,000 + P13,000,000 -
P20,000,000).

7. Now assume P Company pays P30,000,000 in cash to acquire S Company, in an acquisition properly reported
as a statutory merger. P records a gain on acquisition of:

a. Zero
b. P1,000,000
c. P1,700,000
d. P 5,700,000

ANS: B
Rationale: P(1,000,000) = P30,000,000 (P1,000,000 + P35,000,000 + P2,000,000 + P13,000,000 -
P20,000,000).

8.Bats Inc, a new corporation formed and organized because of the recent consolidation of II Inc, and JJ Inc.,
shall issue 10% participating preferred stocks with a par value of P100 for II and JJ net assets contribution, and
common shares with a par value of P50 for the difference between the total shares to be issued and the preferred
shared issued. The total shares to be issued by Bats shall be equivalent to average annual earnings capitalized at
10%. Relevant data on II and JJ follows:

II JJ
Total assets P720,000 P921,600
Total liabilities 432,000 345,600
Annual earnings(average) 46,080 69,120

The total preferred shares to be issued and the amount of goodwill to be recognized by Bats are:

a. Preferred shares: 8,640 Goodwill: P288,000


b. Preferred shares: 5,760 Goodwill: P288,000
c. Preferred shares: 2,880 Goodwill: P864,000
d. Preferred shares: 7,280 Goodwill: P864,000

ANSWER: A

II JJ TOTAL
Average annual earnings P 46,080 P 69,120 P 115,200
Divided by: Capitalized at 10%
Total stock to be issued P1,152,000
Goodwill (for Common Stock) 864,000

Preferred stock (same with Net Assets):


864,000/P100 par 8,640 shares

9.Companies A and B decide to consolidate. Asset and estimated annual earnings contributions are as follows:

Co.A Co. B Co.C


Net asset contribution P300,000 P400,000 P700,000
Estimated annual earnings 50,000 80,000 130,000
contribution

Stockholders of the two companies agree that a single class of stock be issued, that their contributions be
measured by net assets plus allowances for goodwill, and that 10% be considered as a normal rate of return.
Earnings in excess of the normal rate of return shall be capitalized at 20% in calculating goodwill. It was also
agreed that authorizes capital stock of the new company shall be 20,000 shares with a par value of P100 a
share.

What is amount of goodwill credited to Co. A, and the total contribution of Co.B(net assets plus goodwill)
a. P100,000; P400,000 c. P100,000; P600,000
b. P150,000;P500,000 d. P200,000; P600,000

ANSWER: C

Company A Company B
Net Asset Contribution P300,000 P400,000
Add: P50,000 P80,000
Goodwill Average/Annual Earnings
Less: Normal Earnings (10%of net 30,000 40,000
asset)
Excess earnings P20,000 P40,000
Divided by: Capitalized at 20% 20%
Goodwill P100,000 P200,000

Total contribution(stock to be issued P400,000 P600,000

10. Malakas Company acquired all of Maganda Corporation's assets and liabilities on January 2,2013, in a
business combination. At that date, Maganda reported assets with a book value of P624,000 and liabilities of
P356,000. Malakas noted that Maganda had P40,000 of research and development costs on its books at the
acquistion date that did not appear to be of value. Malakas also determined that patents developed by Maganda
had a fair value of P120,000 but had not been recorded by Maganda. Except for building and equipment, Malakas
determined the fair value of all other assets and liabilities reported by Maganda approximated Malakas recorded
amounts. In recording the transfer of assets and liabilities to its books, Malakas recorded goodwill of P93,000.
Malakas paid P517,000 to acquire Maganda's asset and liabilities.
If the book value of Maganda's buildings and equipment was P341,000 at the date of acquisition, what was their
fair value?
a. P441,000
b. P417,000
c. P341,000
d. P417,000
Answer: B.
Solution
Computation of Fair Value
Amount paid P517,000
Book Value of assets P624,000
Book Value of liabilities. (356,000)
Book Value of net assets. P268,000
Adjustment for RandD costs. (40,000)
Adjusted book value. P228,000
Fair value of patent. 120,000
Goodwill recorded. 93,000 (441,000)
Fair value increment of
building and equipment P76,000
Book value of building and Equipment. 341,000
Fair Value of buildings and equipment P417,000
11. Richard Ltd. and Liway Ltd. are two family owned ice cream producing companies in Pampanga. Richard Ltd.
is owned by the Melad family, while the Basilio family owns Liway Ltd. The Melad family has only one son. and he
is engaged to be married to the daughter of Basilio family. Because the son currently managing Liway Ltd., it is
proposed that he be allowed to manage both companies after the wedding. As a result, it is agreed by the two
families that Richard and Ltd. should take over the net assets of Liway Ltd.

The balance sheet at Liway Ltd. immediately prior to the takeover is as follows:

Carrying Amount Fair Value

Accounts receivable P20,000 P 20,000


Inventory 140,000 125,000
Land 620,000 840,000
Buildings (net) 530,000 550,000
Farm equipment (net) 360,000 364,000
Irrigation equipment (net) 220,000 225,000
Vehicles (net) 160,000 172,000
Total assets P2,050,000

Accounts payable P80,000 P 80,000


Loan-Metrobank 480,000 480,000
Share capital 670,000
Retained earnings 820,000
Total P2,050,000

The takeover agreement specified the following details:

* Richard Ltd. is to acquire all the assets of Liway Ltd. and except one of the vehicles (having a carrying amount
of P45,000 and of fair value of P48,000) and assume all the liabilities except for the loan from Metrobank. Liway
Ltd. is then to go, into liquidation.
* Cash at P20,000, half to be paid on date of exchange and half in one year's time. The incremental borrowing
rate is 10% per annum (present value for P1 at 10% for 1 period is 0.909091).
* Supply of a patent relating to the manufacture of ice cream. This has a fair value of P60,000 but has not been
recognized in the records of Liway Ltd. because it resulted from an internally generated research project.
* Richard Ltd. is to supply sufficient cash to enable the debt to Metrobank to be paid for and to cover the
liquidation costs of P5,500. it will also give P150. 000 to be distributed to Mr. an Mrs. Melad to assists in paying
the wedding costs.
* Richard Ltd. is also to give a piece of its own prime land to Liway Ltd. to be distributed to Mr and Mrs. Melad,
this eventually being available to be given to any offspring of the forthcoming marriage. The piece of land in
question has a carrying amount of P80,000 and a fair value of P220,000.
* Richard Ltd. is to issue 90,000 shares, these having a fair value of P14 per share, to be distributed via Liway
Ltd. to the soon to-be-married-daughter of Mr. and Mrs. Melad, who is currently a shareholder in Liway Ltd.

The takeover proceeded as per the agreement with Richard Ltd. incurring incidental acquisition costs of P25,000,
while there were P 18,000 share issue costs.

The amount of goodwill or (bargain purchase gain):

a. P45.682
b. 70,682
c. 118,682
d. P(109,818)
Answer: A
Solution
Consideration transferred:
Shares: (90.000 x P14 per share) P1,260,000
Cash: Payable Now 20,000
Deferred (P20,000 x 0.909091) 18,182
Patent 60,000
Cash (to Metrobank) 480,000
Liquidation costs 5,500
Wedding costs 150,000
Land 220,000 P2,213,682
Less: Fair value of net identifiable assets acquired.
Accounts receivable P20,000
Inventory 125,000
Land 840,000
Buildings 550,000
Farm equipment 364,000
Irrigation equipment 225,000
Vehicles ( P172,000 - P480,000) 124,000
Accounts payable (80,000) 2,168,000
Goodwill P45,682

12. The Boy George, Company acquired the net assets of the Girl Conrad Company on January 1, 2015, and
made the following entry to record the purchase:
Current Assets100,000
Equipment 150,000
Land 50,000
Buildings 300,000
Goodwill 100,000
Liabilities 80,000
Common stock,P1 par 100.000
Paid in capital in excess at par 520,000

Assuming that additional shares on January 1, 2017 would be issued on that date to compensate for any fall in
the value at Boy George common stock below P16 per share. The settlement would be to cure the deficiency by
issuing added shares based on their fair value on January 1,2017. The fair price of the shares on January 1, 2017
was P10.

What is the additional number of shares issued on January 1, 2017 to compensate for any fall in the value at the
stock?

a. 160,000
b. 100,000
c. 60,000
d. 10,000
Answer: C
Solution
Deficiency: (P16 - P10) x100,000 shares issued to acquire P600,000
Divided by: fair value of share P 10
Additional number of shares to issued 60,000

Another example at contingencies is where the acquirer issues to the acquiree and the acquiree is concerned
that the issue of these shares may make the market price at the acquirer s shares decline over time.

Therefore the acquirer may offer additional cash or shares if the market price falls below specified amount over a
specific period of time.

13. Fay acquires assets and liabilities of May Company on January 1,2016. To obtain these shares, Fay pays
P400,000 and issues 10,000 shares of P20 par value common stock on this date. Fay's stock had a fair value of
P36 per share on that date. Fay also pays P15,000 to a local investment firm for arranging the transaction. An
additional P10,000 was paid by Fay in stock issuance costs.
The book values for both Fay and May as of January 1,2016 follow. The fair value of each of Fay and May
accoubts is also included. In addition, May holds a fully amortized trademark that still retains P40,000 value. The
figures below are in thousands. Any related questions also in thousands.
May Company
Fay, Inc. Book Value Fair Value
Cash P900 P80 P80
Receivables 480 180 160
Inventory 660 260 300
Land 300 120 130
Buildings(net) 1,200 220 280
Equipment(net) 360 100 75
Accounts Payable 480 60 60
Long-term liabilities 1,140 340 300
Common Stock 1,200 80
Retained earnings 1,080 480
Assuming the combination is accounted for as an acqusition, immediately after the acquisition, in the balance
sheet of Fay:
What amount will be reported for goodwill?
a. P55 c. P70
b. 65 d. 135

Answer: A.
Consideration Transferred:
Cash P400
Shares (10,000x36) 360
Total P760
Less: Fair value of net iden. assets acquired
Cash P80
Receivables 160
Inventory 300
Land 130
Buildings(net) 280
Equipment(net) 75
Trademark 40
Accounts Payable (60)
Long-term liabilities (300) 705
Goodwill P 55

14. Using the same information in No. 1, what amount will be reported for retained earnings?
a. P1,065 c. P1,525
b. 1,080 d. 1,560

Answer: A.
Acquirer - Fay (at book value) P1,080
Less: Acquisition-related costs 15
Acquiree - May (not acquired) 0
Retained Earnings P1,065

15. Using the same information in No. 1, what amount will be reported for cash after the purchase transaction?
a. P980 c. P875
b. P900 d. P555

Answer: D.
Acquirer - Fay (at book value) P900
Less: Cash paid to acquire net assets of May 400
Acquisition-related costs 15
Stock issuance costs 10
Acquiree - May (fair value) 80
Cash P555

16.Villena Company issued its common stock for the net assets of Wynona Company in a business combination
treated as an acquisition. Villena's common stock issued was worth P 1,500,000. At the date of combination,
Villena's net assets had a book value of P 1,600,000 and a fair value of P 2,000,000 ; Wynona's net assets had a
book value of P 950,000 and a fair value of P 1,100,000. Immediately following the combination, the net assets of
the combined company should have been reported at what amount?

a. P 3,500,000 b. P 3,100,000 c. P 4,200,000 d. P 2,550,000

Answer: b. P 3,100,000

Solution:

Acquisition Cost P 1,500,000

Less: Fair Value of Identifiable

Assets Acquired 1,100,000

Goodwill P 400,000

Villena's Net Assets at Book Value 1,600,000

Wynona's Net Assets at Fair Value 1,100,000

Total Assets After Combination P 3,100,000

17. On July 1, 2014, Trence Company acquired the net assets of the Yasser Company for a price of P
42,000,000. At the acquisition date the carrying value of Yasser's net asset was P 35,000,000. At the acquisition
date a provisional fair value of the net assets was P 37,000,000. An additional valuation received on April 30,
2015 increased the provisional value to P 38,500,000 and on July 31, 2015 this fair value was finalized at P
40,000,000.

What amount should Trence Company present the goodwill in its statement of financial position at December 31,
2015?
a. P 2,000,000 b. P 7,000,000 c. P 3,500,000 d. P 5,000,000

Answer: c. P 3,500,000

Solution:

Acquisition Cost P 42,000,000

Fair Value of Identifiable Assets

Acquired 38,500,000

Goodwill P 3,500,000

Items 18-19 are based on the following data:

Statement of financial position position reflecting uniform accounting procedures l, as well as faire value that are
to be used as basis of the combination are prepared on September 1, 2016 as follows:

Company AceCompany BeeCompany CidAssets

P5,250,000P6,800,000P900,000Liabilities P3,900,000P2,600,000 P480,000Capital stock, all P15 par 1,900,000


1,400,000475,000Additional paid-in capital 400,00040,000Retained earnings(deficit) (450,000)
2,400,000(95,000)Total equitiesP5,250,000P6,800,000P900,000

Ace Company shares have a market value of P22 per share. Market values is not available for shares of Bee
Company and Cid Company .

On September 1, 2016 Ace Company acquires all of the assets and assumes the liability of Bee Company and
Cid Company by issuing P200,000 shares of its stock to Bee Company andpaid 29,000 shares of its stock to Cid
Company. Ace Company pays P10,000 for registering and issuing securities and P20,000 for other acquisition
costs combination.

18. What is the goodwill to be recorded Ace Company on September 1, 2016?

a. P448,000
b. P220,000
c. P400,000
d. P418,000

19. What is the total stockholders equity in the combined statement of financial position after combination on
September 1, 2016?

a. P6,488,000
b. P3,252,000
c. P6,468,000
d. P6,458,000

Solution #18

Answer: D

Bee Company
Price paid P4,400,000
Net assets. 4,200,000
Goodwill P 200,000

Cid Company
Price paid P638,000
Net assets 420,000
Goodwill. P218,000
Total goodwill P418,000

Solution #19

Answer: A

Total Equity P1,450,000


Additional share issuance 3,435,000
Additional paid-in capital. 1,603,000
Registering and issuing fee. (10,000)
Other acquisition costs . (20,000)
Total stockholders equity P6,458,000

20. The statement of financial position of B.o.B. Company as of December 31, 2013 is as follows:
Assets Liabilities and Shareholders Equity
Cash 175,000 Current Liabilities 250,000
Accounts Receivable 250,000 Mortgage payable 450,000
Inventory 725,000 Ordinary Share Capital 200,000
Property, plant and equipment 950,000 Share Premium 400,000
2,100,000 Accumulated Profits 800,000
2,100,000

On December 31, 2013 the Taylor Swift Inc. bought all of the outstanding shares of B.o.B. Company for P
1,800,000 cash. On the date of acquisition, the fair market value of B.o.B.s inventories was P 675,000, while the
fair value of B.o.B.s property, plant equipment was P 1,100,000. The fair value of all other assets and liabilities of
B.o.B. were equal to their book values. In addition, not included above were costs in-process research and
development of B.o.B Company amounting to P 100,000.

Goodwill amounted to:

a. P 400,000 b. P 300,000 c. P 200,000 d. P -0-

Ans. C
Consideration Transferred P1,800,000
Book Value of Net Assets:
Ordinary Share Capital P200,000
Share Premium P400,000
Accumulated Profits (P800k+P100k) P900,000
Allocable excess P300,000
Increase/Decrease in assets:
Inventory (675k-725k) P50,000
P.P.E (1100k-P950K) (P150,000)
P200,000

21.Bruno Mars Company acquired Billboard Companys net assets by issuing its own P 14 par value ordinary
shares totaling 50,000 shares at market price of P 14.55. Bruno Mars Company had the following expenditures
incurred:

Finders fee paid P 50,000


Pre-acquisition audit fee, 40,000
30% was paid
General administrative costs 15,000
Doc stamp paid on issuance 3,500
for the combination
Legal fees for the 32,000
combination paid
Audit fees for SEC 46,000
registration of share issue
SEC registration for the 10,000
share issue paid
Share issuance costs paid 10,000
(inclusive of taxes paid)
sOther indirect costs paid 16,000

The total amount debited to expense should be

a. P 153,000 b. P 156,500 c. P 195,000 d. P 191,500

Ans. D

Finders fee paid P 50,000


Pre-acquisition audit fee, 40,000
30% was paid
Doc stamp paid on issuance 3,500
for the combination
Legal fees for the 32,000
combination paid
Audit fees for SEC 46,000
registration of share issue
SEC registration for the 10,000
share issue paid
Share issuance costs paid 10,000
(inclusive of taxes paid) P191,500

22. On 1 December 2015, Casio Ltd. acquired all the assets and liabilities of Aurora Ltd. With Casio Ltd. Issuing
100, 000 shares to acquire these net assets. The fair value of Aurora Ltd.s assets and liabilities at this date were:

Cash P50, 000


Furniture and fittings 20, 000
Accounts receivable 5, 000
Plant 125, 000
Accounts payable 15, 000
Current tax liability 8, 000
Provision for annual leave 2, 000

The financial year for Casio Ltd. is January December.

The fair value of each Casio Ltd. Share at acquisition date is P1.90. At acquisition date, the acquirer could only
determine a provisional fair value for the plant. On 1 March 2016, Casio Ltd. received the final value from the
independent appraisal, the fair value at acquisition date being P131, 000. Assuming the plant had further five-year
life from the acquisition date.

The amount of goodwill arising from the business combination of December 1, 2015:
a. P15, 000
b. P9, 000
c. P5, 000
d. 0

Ans: B
Solution:
Consideration transferred (100, 000 shares x P1.90) P190, 000
Less: fair value of net identifiable assets acquired:
Cash P50, 000
Furniture and fittings 2, 000
Accounts receivable 5, 000
Plant 131, 000
Accounts payable (15, 000)
Current tax liability (8, 000)
Liabilities (2, 000) 181, 000
Goodwill P9, 000

One of the problems that may arise in measuring the assets and liabilities of the acquiree is that the initial
accounting for the business combination may be incomplete by the end of the reporting period. For example, the
acquisition date may be August 18 and the end of reporting period may be August 31.

In this situation, in accordance with par. 45, the acquirer must report provisional amounts in its financial
statements. The provisional amounts will be best estimates and will need to be adjusted to fair values when those
amounts can be determined after the end of the reporting period. The measurement period in which the
adjustments can be made cannot exceed one year after the acquisition date.

The carrying amount of the plant must be calculated as if its fair value at the acquisition date has been recognized
from that date, with an adjustment to goodwill.

If the plant had a 5-year life from the acquisition dates. Casio Ltd. would have charged depreciation for 1 month in
2015. Extra depreciation of P100 being P6, 000 5 years x 1/12 is required in 2016.

The adjusting entry at March 1, 2016 is:


(Adjustment for provisional accounting)
Plant6, 000
Goodwill6, 000

(Adjustment to depreciation due to provisional accounting)


Retained earnings, 1/1/16100
Accumulated depreciation100

If depreciation has been calculated monthly for 2016, further adjustments would be required.
23. Jane Ltd., a supplier of snooker equipment, agreed to be acquire the business of a rival firm, Mercy Ltd. taking
over all assets and liabilities as at 1 June 2016.

The price agreed upon was P40, 000, payable P20, 000 cash and the balance by the issue to the selling company
of P16, 000 fully paid shared in Jane Ltd. these shares having a fair value of P2.5 per share.

The trial balances of the two companies as at 1 June 2016 were as follows (in thousand peso):

Jane Ltd. Mercy Ltd.


Dr. Cr. Dr. Cr.
Share capital P100 P90
Retained earnings 12 P24
Accounts payable 2 20
Cash P30 -
Plant (net) 50 30
Inventory 14 26
Accounts receivable 8 20
Government bonds 12 -
Goodwill - 10
P114 P114 P110 P110

All the identifiable net assets of Mercy Ltd. were recorded by Mercy Ltd. at fair value except for the inventory
which was considered to be worth P28, 000. The plant had an expected remaining life of five years.

The business combination was completed and Mercy Ltd. went into liquidation. Cost of liquidation amounted to
P1, 000. Jane Ltd. incurred incidental costs of P500. Cost of issuing shares in Jane Ltd. were P400.

The amount of goodwill:

a. P0
b. P2, 000
c. P2, 900
d. P3, 900

Ans. :B
Solution:

Consideration transferred:
Cash P20, 000
Shares:16, 000 shares x P2.50 40, 000 P60, 000

Less: fair value of net identifiable assets acquired:


Plant 30, 000
Inventory 28, 000
Accounts receivable 5, 000
Plant 20, 000
Accounts payable (20, 000) 58, 000
Goodwill P2, 000

It should be noted that acquisition-related costs is not the same with liquidation-related costs even though the
consequence of acquisition is liquidation of the acquiree. Any costs of liquidation or of similar item paid or
supplied by the acquirer should be part of the consideration transferred for reason that it was intended to
complete the process of liquidation. The reason for such inclusion is that the consideration received from the
acquirer may be used to pay for liabilities not assumed by the acquirer and for liquidation expenses which is
tantamount for unrecorded liabilities from liquidation point of view. These items should not be confused with
acquisition-related costs as noted earlier which are considered outright expenses. Further, any liquidation costs or
similar item which was not of the same situation as mentioned above should be treated as expenses.

When it liquidates, costs of liquidation paid by the acquiree should be for the account of the acquire and will be
eventually transferred to stockholders equity account. This payment made should considered expenses by the
acquiree in the process of liquidation not unlike payment supplied and made by the acquirer which is intended for
any unrecorded expenses.

Faith Company is acquiring the net assets of Love Company for an agreed upon price of P1000,000 on
April 1,2014. The value was tentatively assigned as follows:

Current Assets P 100,000


Land 70,000
Equipment - 5 year life 300,000
Building -20 year life 500,000
Current Liabilities (200,000)
Goodwill 230,000

Values were subject to change during the measurement period. Depreciation is taken to the nearest
month. The measurement period expired on April 1, 2015 at which time the fair value of the equipment
and building as of acquisition date were revised to 280,000 and 600,000, respectively.
24.How much total depreciation expense will be recorded for 2015.
a. 85,000
b. 86,000
c. 83,500
d. 86,500

Ans. B
Equipment 280,000/5 56,000
Building 600,000/20 30,000
86,000
25.How much goodwill is presented in 2015 statement of financial position?
a. 230,000
b. 180,000
c. 150,000
d. 200,000

Ans. C
Agreed price 1,000,000
Less: fair value of net assets
(1,050,000-200,000) 850,000
150,000

26. Westport Ltd. a suplier of snooker equipment, agreed to acquire the business of a rival firm, Manukau Ltd.
taking over all assets and liabilities as at 1June 20x4.
The price agreed upon was P40,000, payable P20,000 cash and the balance by the issue to the selling company
of P16,000 fully paid shares in Westport Ltd. these shares having a fair value of P2.50 per share.
The trial balances of the two companies aa at 1 June 20x4 were as follows:
Westport Ltd Manukau Ltd.

Share capital P100,000 P 90,000


Retained earning 12,000 P 24,000
Accounts payables 2,000 20,000
Cash P 30,000 -
Plant 50,000 30,000
Inventory 14,000 26,000
Accounts receivable 8,000 20,000
Government bonds 12,000 -
Goodwill ---- 10,000
P 114,000 P 114,000 P 110,000 P 110,000

All the identifiable net assets of Manukau Ltd. were recorded by manukau Ltd. At fair value except for the
inventory which was considered to be worth P28,000. The plant had an expected remaining life of five years.
The business combination was completed and Manukau Ltd. went into liquidation. Westport Ltd. Incurred
incidental costs of P500 in relation to the acquisition cost. Cost of issuing shares in Wesport Ltd. were P400. The
amount of goodwill to:

A. Nil or zero
B. P2,509
C. P2,900
D. P3,900

ANSWER: B

Cost of investment {20,000 + (16,000


P 60,500
shares x P2.50) + 500 incidental cost}
Less: markt value of net assets acquired: P 30,000
Plant 28,000
Inventory 5,000
Account receivable 20,000
Plant (20,000)
Accounts payable 58,000
Goodwill P 2,500

27. Bats Inc., a new corporation formed and organized because of the recent consolidationof II Inc. and JJ Inc.,
shall issue 10% participating preferred stocks with a par value of P100 for all II andJJ net assets contributions,
and common shares with a par value of P50 for the difference between the total shares to be issued and the
preffered shared to be issued. The total shares to be issued by Bats shall be equivalent to average annual
earnings capitalized at 10%. Relevant data on II and JJ follows:
II JJ
Total assets.................................... P720,000 P921,600
Total liabilities................................ 432,000 345,600
Annual earnings (average)............ 46,080 69,120
The total preferred shares to be issued and the amount of goodwill to be recognized by Bats are:

A. Preferred shares: 8,640 Goodwill: P288,000


B. Preferred shares: 5,760 Goodwill: P288,000
C. Preferred shares: 2,880 Goodwill: P864,000
D. Preferred shares: 7,280 Goodwill: P864,000

ANSWER: A

II JJ Total
Average annual arnings P 46,080 P 69,120 P 115,200
Divided by: capitalized at 10%
Total stock to be issued P 1,152,000
Less: net assets (for P/S) 864,000
Goodwill (for common stock) P 288,000
Preferred stock (same with
Net assets):864,000/100 8,640 shares

28. Cormorant Corporatlon paid 800,000 for a 40% Interest in Plumage Company on January 1, 2005 when
Plumage's stockholder's equity was as follows:

10% cumulative preferred stock, $100 par S 500,000


Common stock, $10 par value S 300,000
Other paid-In capital S 400,000
Retained earnings S 800,000
Total stockholders equity S 2,000,000

On this date, the book values of Plumage's assets and liabilities equaled their fair values and there were no
dividends In arrears. Goodwill from the investment is

a.S 0.
b. 150,000.
c. 200,000.
d. None of the above ls correct.

Answer: d

Cost of Cormorant's investment: $ 800,000


Less: book value acquired:
Total equity $ 2,000,000
Less: Preferred equity $ 500,000
Net common equity $ 1,500,000
x percent acquired 40%
= Plumage book value $ 600,000 $ 600,000
Goodwill $ 200,000
29. On February 5, Pryor Corporation paid $1,600,000 for all the issued and outstanding common stock of Shaw,
Inc., in a transaction properly accounted for as an acquisition. The book values and fair values of Shaw's assets
and liabilities on February 5 were as follows:

Book Value Fair Value


Cash $ 160,000 $160,000
Receivables (net) 180,000 180,000
Inventory 315,000 300,000
Plant and equipment (net) 820,000 920,000

Liabilities (350,000) (350,000)


Net assets $1,125,000 $1,210,000

What is the amount of goodwill resulting from the business combination?

a) $-0-.
b) $475,000.
c) $85,000.
d) $390,000.

Answer: d

FV of consideration transferred $ 1,600,000


Less: FV of Net Assets $ 1,210,000
Goodwill $ 390,000

On January 1, 20x5, the fair values of Crmes net assets were as follows:

Current Asset P100,000


Equipment 150,000
Land 50,000
Buildings 300,000
Liabilities 80,000

30. On January 1, 20x5, Brulee Company purchased the net assets of the Crme Company by issuing 100,000
shares of its P1 par value stock when the fair value of the stock was P6.20. It was further agreed that Brulee
would pay an additional amount on January 1, 20x7, if the average income during the 2-year period of 20x5-20x6
exceeded P80,000 per year. The expected value of this consideration was calculated as P184,000; the
measurement period is one year. What amount will be recorded as goodwill on January 1, 20x5?
a. Zero c. P180,000
b. P100,000 d. P284,000

Ans: d

Consideration transferred
Shares: (100,000 shares x P6.20) P620,000 Contingent
consideration 184,000
Total P804,000
Less: Current Assets (at fair values) P100,000
Equipment 150,000
Land 50,000
Buildings 300,000
Liabilities ( 80,000) 520,000
Goodwill P284,000

31.On July 1, 20x5 The Straw Company acquired 100% of the Berry Company for a consideration transferred of
P160 Million. At the acquisition date the carrying amount of Berrys net assets was P100 Million. At the acquisition
date a provisional fair value of P120 Million was attributed to the net assets. An additional valuation received on
May 31, 20x6 increased this provisional fair to P135 Million and on July 30, 20x6 this fair value was finalized at
P140 Million. What amount should Straw present for goodwill in its statement of financial position on December
31, 20x6, according to PFRS 3 Business Combinations?
a. P20 million c. P50 million
b. P25 million d. P60 million

Ans: b

Consideration transferred P160 million


Fair Value on May 31, 20x6 135 million
Goodwill P 25 million
32.Hazel Corp. was merged into Sebastian Corp. in a combination properly accounted for as acquisition of
interest. Their condensed sheets before the combination show:

Sebastian Hazel
Current assets.. P1,144,000 P 813,800
Plant and equipment, net... 2,327,000 520,000
Patents.. - 130,000

Total assets.. P3,471,000 P1,463,800

Liabilities... P1,352,000 P 85,800


Capital stock, par P100.. 1,300,000 650,000
Additional paid-in capital.. 195,000 195,000
Retained earnings...... 624,000 533,000

Total Liabilities and Equity. P3,471,000 P1,463,800

Per independent appraisers report, Hazels assets have fair market values of P826, 800 for current assets, P624,
000 for plant and equipment and P169, 000 for patents. Hazels liabilities are properly valued. Sebastian
purchases Hazels net assets for P1, 534,000. How should the difference between the book value of Hazels net
assets and the consideration paid by Sebastian be considered?

a. Goodwill: P 0 ; Increase in Assets: P156,000


b. Goodwill: P 0 ; Increase in Assets: P312,000
c. Goodwill: P169,000 ; Increase in Assets: P156,000
d. Goodwill: P169,000 ; Increase in Assets: P 78,000

ANSWER: (a)

Consideration transferred... P1,534,000


Less: Market value of net assets acquired, excluding GW:
Current assets... P826,800
Plant and equipment. 624,000
Patents 169,000
Liabilities. (85,800) 1,534,000

Goodwill P -0- (a)

Book Fair Increase


Value Value (Decrease)
Current Assets. P813,800 P826,800 P 13,000
Plant and Equipment.. 520,000 624,000 104,000
Patents.. 130,000 169,000 39,000

Increase in assets P156,000 (a)


33.On December 2015, Agulan Co. acquired all the assets and liabilities of Toquero Co. with Agulan Co.
issuing 150,000 shares to acquire these net assets. The fair value of Toquero Co.s assets and liabilities
at this date were:

Cash P75,000
Accounts receivable. 7,500
Fix and Furnitures. 30,000
Plant and Equipment 187,500
Accounts payable.. 22,500
Current tax liability. 12,000
Provision for annual leave 3,000

The financial year for Agulan Co. is January December.

The fair value of each Agulan Co. share at acquisition date is P2. At acquisition date, the acquirer could
only determine a provisional fair value for the plant and equipment. On March 1, 2016, Agulan Co.
received the final value from the independent appraisal, the fair value at acquisition date being P196,500.
Assuming the plant and equipment had a further five-year life from the acquisition date.

The amount of goodwill arising from the business combination at December 1, 2015:

a. P 0
b. P18,750
c. P37,500
d. P30,500

ANSWER: (c)

Consideration transferred (150,000 shares x P2) P300,000


Less: Fair value of net identifiable assets acquired:
Cash. P 75,000
Accounts receivable.. 7,500
Fix and Furniture 30,000
Plant and Equipment 187,500
Accounts payable.. ( 22,500)
Current tax liability ( 12,000)
Liabilities. ( 3,000) 262,500

Goodwill P37,500

34. Homer Ltd. is seeking to expand its share of the widgets market and has negotiated to take over the
operations of Tan Ltd. on January 1, 20x4. The balance sheets of the two companies as at December 31, 20x4
were as follows:
Homer Tan
Cash P 23,000 P 12,000
Receivables 25,000 34,700
Inventory 35,500 27,600
Freehold Land 150,000 100,000
Buildings (net) 60,000 30,000
Plant and equipment (net) 65,000 46,000
Goodwill 25,000 2,000
P383,500 P252,300

Accounts payable P 56,000 P 43,500


Mortgage loan 50,000 40,000
Debentures 100,000 50,000
Common stock, 100,000 shares 100, 000
Common stock, 60,000 shares 60,000
Additional paid-in capital 28,500 26,800
Retained earnings 49,000 32,000
P 383,500 P 252,300

Homer Ltd. is to acquire all the assets, except cash of Tan Ltd. The assets of Tan are all recorded at fair value
except:
Fair Value
Inventory P 39,000
Freehold land 130,000
Buildings 40,000

ln exchange, Homer Ltd. is to provide sufficient extra cash to allow Tan Ltd. to repay all of its outstanding debts
and its liquidation costs of P2,400, plus two fully paid shares in Homer Ltd. for every three shares held in Tan Ltd.
The fair value of a share in Hastings Ltd. is P320. An investigation by the liquidator of Tan Ltd. reveals that on
December 31, 20x3, the followmg outstanding debts were outstanding but had not been recorded:

Accounts payable P1,600


Mortgage interest 4,000

The debentures issued by Tan Ltd. are to be redeemed at a 5% premium. Costs of issuing the shares were
P1,200.
The excess of fair value of net assets over cost or gain on acquisition that will be recognized immediately in the
income statement is:

a. Nil or Zero
b. P17,700
c. P29,700
d. P34,300

ANSWER: C

Consideration transferred:
Shares: 2/3 x 60,000 x P3.20. 128,000
Cash
Accounts payable. 45,100
Mortgage and interest 44,000
Debentures and premium 52,500
Liquidation expenses 2,400
144,000
Cash held (12,000) 132,000
260,000
Less: Fair value of assets and liabilities acquired:
Accounts receivable P34,700
Inventory 39,000
Freehold land 130,000
Buildings 40,000
Plant and equipment 46,000 289,700
Bargain Purchase Gain P 29,700

35.. Westport Ltd., a supplier of snooker equipment, agreed to acquire the business of a rival firm, Manukau Ltd.
taking over all assets and liabilities as at 1 June 20x4.

The price agreed upon was P40,000, payable P20,000 cash and the balance by the issue to the selling company
of 16,000 fully paid shares in Westport Ltd. these shares having a fair value of P2.50 per share.

The trial balances of the two companies as at 1 June 20x4 were as follows.

Westport Ltd. Manukau Ltd.


Dr. Cr. Dr. Cr.
Share capital P100,000 P 90,000
Retained earnings 12,000 P 24,000
Accounts payable 2,000 20,000
Cash P30,000 -
Plant (net) 50,000 30,000
Inventory 14,000 26,000
Accounts receivable 8,000 20,000
Government bonds 12,000 -
Goodwill - -
.
P114,000 P114,000 P110,000 P110,000
All the identifiable net assets of Manukau Ltd. were recorded by Manukau Ltd. at fair value except for the
inventory which was considered to be worth P28,000. The plant had an expected remaining life of five years.

The business combination was completed and Manukau Ltd. went into liquidation. Westport Ltd. incurred
incidental costs of P500 in relation to the acquisition costs. Costs of issuing shares in Westport Ltd. were P400.
The amount of goodwill to:

a. Nil or zero
b. P2,500
c. P2,900
d. P3,900

ANSWER: B

Cost of Investment
[P20,000 + (16,000 shares x P2.50) + P500, incidental costs) P 60,500
Less: Market value of net assets acquired:
Plant P 30,000
Inventory 28,000
Accounts receivable 5,000
Plant 20,000
Accounts payable ( 20,000) 58,000
Goodwill P 2,500

When it liquidates, costs of liquidation paid by the acquiree should be for the liquidation account of the acquiree
and will eventually be transferred to shareholders equity account. Any costs of liquidation paid or
supplied by the acquirer should be capitalized as cost of acquisition which stent with the cost model under PFRS
No. 3 in measuring the cost of the combination.

Any direct costs of acquisition should be capitalizable under the cost model reiterated in PFRS No. 3 Phase I.
This model in PFRS No. 3 will be amended under Phase II (pending implementation possibly until early 2008),
wherein all direct costs will be outright expense. Costs of issuing shares will be debited to share premium or
APIC account.

Any costs of liquidation paid or supplied by the acquirer should be capitalized as cost of acquisition which is
consistent with the cost model under PFRS No. 3 in measuring the cost of the combination. The fair values of
liabilities undertaken are best measured by the present values of future cash outflows. Intangible assets are
recognized when its fair value can be measured reliably. Assets other than intangible assets must be recognized
if it is probable that the future economic benefits will flow to the acquirer and its fair value can be measured
reliably.

36.Mango Company acquired Apple Company on January 2, 2016 by issuing common shares. All of Apples
assets and liabilities were immediately transferred to Mango Company which reported total par value of shares
outstanding of P218,400 and P327,600 and additional paid-in capital of P370,000 and P650,800 immediately
before and after the business combination, respectively.
Assuming that Mangos common stock had a market of P25 per share at the time of acquisition, what number of
shares was issued?

a. 15,600
b. 10,000
c. 15,600
d. 10,000

Answer: C
Par value of shares outstanding following merger P327,600
Paid-in capital following merger 650,800
Total fair value of paid-in capital P978,400
Par value of shares outstanding before merger P218,400
Paid-in capital before merger 370,000 (588,400)
Increase in par value and paid-in capital P390,000
Divided by price per share P25
Number of shares issued 15,600

37.The stockholders equities of Milkita Corporation and Keanu Company at June 1,2016 before
combination were as follows:
Milkita Keanu

Capital Stock, P100 par value P10,000,000 P3,000,000

APIC 50,000 -
Retained Earnings 5,000,000 1,000,000

37.On June 2,2016, Milkita Corporation issued 50,000 of its unissued shares with a market value of P103 per
share for the assets and liabilities of Keanu Company. On the same day Milkita Corporation paid P100,000 for
legal fees, documentary stamp tax of P20,000 and P190,000 for SEC registration fees of equity securities.

Shareholders equity would include :

a. P15,000,000 Capital Stock ; P4,900,000 Retained earnings ; P10,000 Stock issuance cost
b. P15,000,000 Capital stock ; P10,000 APIC ; P4,880,000 Retained earnings
c. P15,150,000 Capital Stock ; P50,000 APIC ;P 4,690,000 Retained earnings
d. P15,000,000 Capital Stock ; P200,000 APIC ; P4,690,000 Retained earnings
Answer : A
Capital stock:
Before combination P10,000,000
Issued at par (50,000 x P100) 5,000,000 P15,000,000
APIC:
Before combination 50,000
Issuance (P3 x 50,000) 150,000
Documentary stamp tax ( 20,000 )
SEC Registration fees ( 180,000) --0
Retained earnings:
Before combination 5,000,000
Legal fees ( 100,000 ) 4,900,000
Stock issuance cost (P190,000+20,000-200,000) ( 10,000 )
Stockholders equity P19,890,000

38.Red Company issued its common stock for the net assets of Blue Company in a business combination
treated as acquisition. Reds common stock issued was worth P1,500,000. At the date of combination,
Reds net assets had a book value of P1,600,000 and a fair value of P1,800,000. Blues net assets had a
book value of P700,000 and a fair value of P850,000. Immediately following the combination, the net
assets of the combined company should have been reported at what amount?
a. P3,000,000
b. P2,400,000
c. P3,100,000
d. P1,850,000

ANSWER: C
Rationale
Acquisition Cost P1,500,000
Net assets acquired 850,000
Goodwill 650,000
Reds net assets @BV 1,600,000
Blues net assets @FV 850,000
Total net assets P3,100,000

39.Mata Inc. purchased all of the net assets of Torralba Company on February 1,2015 by issuing 8,000
shares of its P20 par common stock. At the time, the stock was selling for P40 per share. Direct costs
associated with consummating the combination totalled P5,000. Under IFRS 3, what total amount should
the net assets acquired be recorded by Mata Inc. Assuming the contingent consideration of P7,000 is
determined?

ANSWER: C
Rationale (8,000 shares X 40 = P320,000 + 7,000 contingent consideration = P327,000)

40.Payla Co. Will issue share of P12par common stock for the net assets of Talisay Co. Paylas common stock
has a current market value of P40 per share. Talisay balance sheet accounts follow:
Current Assets P500 000 Common stock, parP4 (P80 000)
Property and equipment 1 500 000 Additional paid-in-capital (320 000)
Liabilities (400 000) Retained earnings (400 000)
Talisay current assets and property and equipment, respectively, are appraised of P 400 000 and P1600 000; its
liabilities are fairly valued. Accordingly, Payla Co. Issued shares of its common stock with total market value
equal to that of Max net assets. To recognize goodwill of P200 000, how many shares were issued?
a. 55 000 c. 40 000
b. 45 000 d. 50 000
Solution:
ANS: B
Fair value of net identifiable assets acquired:
Current assets P 500 000
Property and equipment 1 500 000
Liabilities (400 000)
FMV of net assets P1 600 000
Add: Goodwill 200 000
Consideration transferred P1 800 000
Divided By: Current market value per share P 40
Number of shares issued 45 000

41. Companies of P and J decide to consolidate. Asset and estimated annual earnings contributions are as
follows:
Co. P Co. J Total
Net asset contribution P400 000 P350 000 P750 000
Estimated annual earnings contribution 80 000 70 000 150 000

Stockholders of the two companies agree that a single class of stock be issued, that their contributions be
measured by net assets plus allowances for goodwill, and that 10% be considered as a normal rate of return.
Earnings in excess of the normal rate of return shall be capitalized at 20% in calculating goodwill. It was also
agreed that the authorized capital stock of the new corporation shall be 20,000 shares with a par value of P100 a
share.
(1)The total contribution of Co. J(net assets plus goodwill), and (2)The amount of goodwill credited to Co. A:
a.(1)P475 000;(2)P100 000 c.(1)P525 000;(2)P200 000
b.(1)P500 000;(2)P150 000 d.(1)P600 000;(2)P100 000
Solution:
ANS: C
Company A Company B
Net Asset Contributions P400 000 P350 000
Add: Goodwill
Average/Annual Earnings P 80 000 P 70 000
Less: Normal Earnings
(10% on Net Asset) 40 000 35 000
Excess Earnings P 40 000 P 35 000
Divided by: Capitalized at 20% 20%
Goodwill P 200 000(c) P 175 000
Total Contribution (stock to be issued) P 400 000 P 600 000(c)

42. AB Corporation was merged into CD Corporation in a combination properly accounted for as acquisition of
interests. Their balance sheets before the combination are as follows:
AB Corp.
Current Assets................................................................ P 8,352,950
Plant and Equipment,net................................................ 6,450,700
Patents............................................................................ -

Total Assets.................................................................... P 14,803,650

Liabilities....................................................................... P 5,713,650
Capital Stock,par P100.................................................. 4,600,000
Additional paid-in capital.............................................. 950,000
Retained Earnings.......................................................... 3,540,000

Total Liabilities and Equity........................................P14,803,650

CD Corp.
Current Assets............................................................... .P 7,505,000
Plant and Equipment,net............................................... 3,130,450
Patents........................................................................... 153,800

Total Assets....................................................................P10,789,250

Liabilities.......................................................................P 939,000
Capital stock,par P100.................................................... 3,400,000
Additional paid-in capital............................................... 950,000
Retained Earnings........................................................... 5,500,250

Total Liabilities and Equity........................................ PP10,789,250

Per-independent appraisers report, the fair market value of CDs current assets is P7,808,000; plant and
eqipment is P3,452,000; and patents P286,900. Liabilities of CD Corporation are properly valued. AB Corporation
purchases the net assets of CD Corporation for P10,607,900. How should the difference between the book value
of CD Corporations net assets and the consideration paid by AB Corporation be considered?

A. Goodwill: P 286,900; Increase in Assets: P 757650

B. Goodwill: P 286,900; Increase in Assets: P 303,000

C. Goodwill: P 0; Increase in Assets: P 303,000

D. Goodwill: P 0; Increase in Assets: P 757,650

Answer: D

Consideration Transferred.................................................................................................P10,607,900
Less: Market value of net assets acquired, excluding GW:
Current Assets..........................................................P7,808,000
Plant and Equipment............................................... 3,452,000
Patents...................................................................... 286,900
Liabilities................................................................. ( 939,000) 10,607,900

Goodwill............................................................................ P -0- (D)


Current Assets Plant and Equipment Patents
Book value P 7,505,000 P 3,130,450 P 153,800
Fair Value 7,808,000 3,452,000 286,900

Increase(Decrease) P 303,000 P 321,550 P 133,100


in assets

Current Assets P 303,000


Plant and Equipment 321,550
Patents 133,100

Increase in Assets P 757,650 (D)

43.Companies XX, YY, and ZZ decide to consolidate. The parties to a consolidation have the following data:

Net Assets Average annual earnings


XX Co...................... P 6,800,000 P 680,000
YY Co. .................... 3,000,000 400,000
ZZ Co. .................... 10,200,000 920,000

The parties collectively agreed that the new corporation, RR Co. Will issue a single class of stock based on the
earnings ratio. What is the stock distribution ratio to companies XX, YY,and ZZ respectively?

A. 34:15:51
B. 33:15:52
C. 34:20:46
D. 33:21:46

Answer: C

XX: P 680,000 680,000/2,000,000 = 34%


YY: 400,000 400,000/2,000,000 = 20%
ZZ: 920,000 920,000/2,000,000 = 46%

P 2,000,000 100%

44.Pak companys owns 50% of Ganern Companys cumulative preference shares and 30% of its ordinary
shares.Ganerns shares outstanding at December 31, 2016 include of 10% cumulative preference shares and
P40,000,000 of ordinary shares.
Ganern reported profir of P8,000,000 for the year ended December 31,2016. Ganern declared and paid
P1,500,000 preference shares during 2016. Ganern paid no preference shares dividend during 2015. On January
31,2017, prior to the date that the financial statements are authorized to issue, Ganern distributed 10% ordinary
share dividend.
How much is the total amount to be recognized by Pak Company in its 2016 profit and loss related to these
investment?
a. P2,450,000
b. P2,600,000
c. P2,700,000
d. P2,850,000
Answer: D
Solution:
Ganern profit P8,000,000
Multiplied by: pak companys interest 30%
Pak Company share in Ganerns profit P2,400,000
Dividends declared and paid 1,500,000
Multiplied by: pak companys interest 30%
Dividend income 450,000
P2,850,000
45. Companies T, G, B, parties to consolidation have the following data:

T Co. G. Co. B. Co
Net Assets.. P400, 000 P600, 000 P1, 000, 000
Average annual earnings. 60, 000 60, 000 80, 000

The parties collectively agreed that the new corporation, RC Co. will issue a single class of stocks based on the
earnings ratio. What is the stock distribution ratio to companies T, G, B, respectively?

a. 20:30:50 c. 30:40:30
b. 30:30:40 d. 40:40:30

ANSWER:
Fraction
T: P60, 000 6/20 = 30%
G: 60, 000 6/20 =30%
B: 80, 000 8/20 =40%
P200, 000 100%
46. When should a business combination be undertaken?
A. When a positive net present value is generated to the shareholders of an acquiring firm.
B. When the two firms are in the same line of business, but economies of scale cannot be attained by the
acquiror.
C. When two firms are in different lines of business, creating diversification.
D. When cash will be paid for the acquired firm's stock.

Answer: A.

A business combination is beneficial when the result is a positive NPV. This effect results from synergy, which
exists when the value of the combined firm exceeds the sum of the values of the separate firms. It can be
determined by using the risk-adjusted rate to discount the change in cash flows of the newly formed entity. If a
positive net present value is generated, a combination is indicated.

Answer (B) is incorrect because a combination is indicated if economies of scale can be attained. Answer (C)
is incorrect because diversification may or may not result in a positive NPV. Answer (D) is incorrect because
some beneficial combinations involve exchanges of stock.

47.Which of the following statements is most correct?


a. A firm acquiring another firm in a horizontal merger will not have its required rate of return affected because the
two firms will have similar betas
b. In most mergers, the benefits of synergy and the price premium the acquirer pays over market price are
summed and then divided equally between the shareholders of the acquiring and target firms
c. Financial theory says that the choice of how to pay for a merger is really irrelevant because, although it may
affect the firms capital structure, it will not affect the firms overall required rate of return
d. The primary rationale for any operating merger is synergy, but it is also possible that mergers can include
aspects of both operating and financial mergers
Answer : D
48.A gain should be reported on an acquisition if:

a. The fair value of the consideration paid is less than the book value of the net assets acquired.
b. The fair value of the consideration paid plus the present value of any earnings contingency is less
than the book value of the net assets acquired.
c. The fair value of the consideration paid is less than the fair value of net assets acquired plus the
fair value of identifiable intangibles acquired.
d. The fair value of the consideration paid plus the present value of any earnings contingency is less
than the fair value of identifiable net assets acquired.

ANS: D
49. The following statements pertaining to business combination are not true except:

a. The pooling of interest method recorded the assets and liabilities of the acquired company at their fair values.

b. Statutory merger refers to the combining of two or more existing legal entities into one new legal entity wherein
the previous companies are dissolved and are then replaced by the new continuing company.

c. In a stock acquisition, the parent and the subsidiary has their own separate financial records and statements for
external financial reporting purposes.

d. The acquiring enterprise may inherit the acquired firm's inefficiencies and problems together with its inadequate
resources.

Answer: d

50. The cost of registering equity securities in a business combination should be recorded as;
a. An income of the period
b. an expense of the period
c. Deduction from additional paid in capital
d. Part of the cost of the stock acquired

Answer: C

DATE OF ACQUISITION
1. Jericel Company had common stock of P350,000 and retained earnings of P490,000. Cathrene Inc. had
common stock of P700,000 and retained earnings of P980,000. On January 1, 2016, Cathrene issued
24,000 shares of common stock with a P12 par value and a P35 fair value for all of Jericel companys
common stock. This combination was accounted for as an acquisition. Immediately after the combination,
what was the consolidated net asset?
a. P280,000
b. P2,520,000
c. P1,680,000
d. P1,190,000
ANS: A

Consolidated Stockholders Equity


Acquirer (Parent-Cathrene), book value
(P700,000 + P980,000) P1,680,000
Add: Newly issued shares
(34,000 x P35 fair value) 1,190,000
Acquiree (Subsidiary-Jericel) eliminated
in preparing consolidated balance sheet. 0
P2,870,000
2 .On January 1, 2016, Park Corporation and Strand Corporation and their condensed balance sheet are as
follows:

Park Corp. Strand Corp.


------------------ ---------------------
Current Assets. 70,000 20,000
Non-current Assets. 90,000 40,000
------------------ ---------------------
Total Assets 160,000 60,000
Current Liabilities.. 30,000 10,000
Long term debt.. 50,000 -
Stockholders Equity. 80,000 50,000
------------------ ---------------------
Total Liabilities and Equities 160,000 60,000

On January 2, 2016.Park Corporation borrowed 60,000 and used the proceeds to obtain 80% of the outstanding
common shares of Strand Corporation. The acquisition price was considered proportionate to Strands fair value.
The 60,000 debt is payable in 10 equal annual principal payments, plus interest, beginning December 31, 2016.
The excess fair value of the investment over the underlying book value of the acquired net assets is allocated to
inventory (60%) and to goodwill (40%).

On a consolidated balance sheet as of January 2, 2016, what should be the amount for each of the following?

The amount of goodwill using proportionate basis (partial):

A. Using the same information in No.60, the amount of goodwill using full fair value.(full/gross-up) basis:
a. P 0
b. 8,000
c. 10,000
d. 20,000

ANS:C
Fair value of Subsidiary:
Fair value of consideration given: 60,000 x 80% 75,000
Less :Book value of Net Assets/ Stockholders
Equity of Subsidiary 50,000
Allocated Excess 25,000
-------------
Less: Over/ Undervaluation of Assets and Liabilities:
Increase in Inventory (25,000 x 60%= 15,000 x 100%) 15,000
-------------
Goodwill (full/gross-up) 10,000
-------------
-------------
*100% increase of inventory should amount to 15,000/80%

B .Using the same information in No.60, the amount of stockholders equity using full fair value (full/gross up
goodwill) proportionate basis to determine non-controlling interest should be:
a. 80,000
b. 93,000
c. 95,000
d. 130,000
ANS:C
Park stockholders equity 80,000
Non-controlling interest (full goodwill)
Strand stockholders equity 50,000
Add: Adjustments to reflect fair value -
inventory 15,000
-------------
Strand stockholders equity at FV 65,000
Non-Controlling interests 20% 13,000
------------- -------------
Non-Controlling interests (partial) 93,000
Add: Non-Controlling interest in full goodwill
(10,000-8,000) 2,000
------------
Consolidated Stockholders Equity 95,000
------------
------------
3.On January 2, 2011, Pare Co. purchased 75% of Kidd Cos outstanding common stock. On that date, the fair
value of the 25% noncontrolling interest was P35,000. During 2011, Kidd had net income of P20,000. Selected
balance sheet data at December 31,2011, is as follows:

Pare Kidd
Total assets P420,000 P180,000
Liabilities P120,000 P60,000
Common stock 100,000 50,000
Retained Earnings 200,000 70,000

During 2011 Pare and Kidd paid cash dividends of P25,000 and P5,000 respectively, to their shareholders. There
were no other intercompany transactions.

In Pares December 31,2011 consolidated balance sheet, what amount should be reported as noncontrolling
interest in net assets?

a. P30,000
b. P35,000
c. P38,750
d. P40,000

ANSWER: C

Fair value of noncontrolling interest P35,000


Plus: Share of net income (25% x 20,000) 5,000
Less: Share of dividends (25% x 5,000) (1,250)
Noncontrolling interest P38,750

4.When it purchased Sutton, Inc. on January 1, 20x1 Pavin Corporation issued 500,000 shares of its P5 par
voting common stock. On that date the fair value of those shares totaled P4,200,000. Related to the acquisition,
Pavin had payments to the attorneys and accountants of P200,000, and stock issuance fees of P100,000.
Immediately prior to the purchase, the equity sections of the two firms appeared as follows:

Pavin Sutton
Common Stock P4,000,000 P700,000
Paid in capital in excess of par 7,500,000 900,000
Retained earnings 5,500,000 500,000
Total P17,000,000 P2,100,000
Immediately after the purchase, the consolidated balance sheet should report paid in capital in excess of par of.

a. P8,900,000
b. P9,100,000
c. P9,200,000
d. P9,300,000

ANSWER: B

FV, stocks issued P 4,200,000


Less: Par value of stocks issued (500,000 shares x P5) 2,500,000
APIC P 1,700,000
Add: APIC of P 7,500,000
Less: Stock issuance cost 100,000
P 9,100,000

5 .The Moon Company acquired a 70% interest In The Swan Company for P1,420,000 when the fair value of
Swan's identifiable assets and labilities was P1,200,000. Moon acquired a 65% interest In The Homer
Company for P300,000 when the fair value of Homer's identifiable assets and liabilities was P640,000. Moon
measures non-controlling interest at the relevant share of the identifiable net assets at the acquisition date.
Neither Swan nor Homer had any contingent liabilities at the acquisition date and the above fair values were
the same as the carrying amounts in their financial statements. Annual impairment reviews have not resulted In
any impairment losses being recognized.
Under PFRS 3 Bussiness combinations, what figures in respect of goodwill and of gains on bargain
purchases should be included in Moon's consolidated statement of financial position?
a. Goodwill: P580,000: Gains on the bargain purchases: P116,000
b. Goodwill: Nil or zero: Gains on the bargain purchases: P116,000
c. Goodwill: Nil or zero; Gains on the bargain purchases: Nil or zero
d. Goodwill: P580,000: Gains on the bargain purchases: Nil or zero
Answer: D
Solution
Fair value of subsidiary - Swan
Consideration transferred P1,420.000
less: Fair value at identifiable assets and liabilities of Swan
(70% x P1.2 million) 840.000
Goodwill (partial) P580,000
"Goodwill is carried as on asset in the consolidated statement of financial position."

Fair value of Subsidiary Homer

Consideration transferred P 300. 000

less: Fair value at identifiable assets and Liabilities of Homer


(65% x P640 000) 416,000

Gain on bargain purchases P(116,000)

6. Questions A and B are based on the following:


Winston has the following account balances as of February 1, 2014:
Inventory P 600,000 Common stock (P10 par value) P 800,000
Land 500,000 Retained earnings, Jan. 1,2014 1,100,000
Buildings (net) (FV P1,000,000) 900,000 Revenues 600,000
Expenses 500,000

Arlington pays P1.4 million cash and issues 10,000 shares of is P30 par value common stock (valued at P80
per share) for all of Winstons outstanding stock and Winston is dissolved. Stock issuance costs amount to
P30,000. Prior to recording these newly issued shares, Arlington reports a Common Stock account of
P900,000 and Additional Paid-in Capital of P500,000.

A. Determine the goodwill that would be Included in the February 1, 2014, financial statement of Arlington.

a. P200,000 b. P230,000 c. P100,000 d. P130,000

Answer: C.
Cost of acquiring Winston
Cash P1,400,000
Shares of stocks ( 10,000 x 80) 800,000 2,200,000
Fair value of net assets acquired:
Inventory P600,000
Land 500,000
Building 1,000,000 (2,100,000)
Goodwill P100,000

B. Assume that Arlington pays cash of P20 million. No stock is issued. An additional 40,000 Is paid In direct
combination costs, determine the net gain from business combination.

a. P100,000 b, P200,000 c. 260,000 d. 60,000

Answer: D.
Gain from business combination must be P60,000.
Cost of acquiring Winston P2,000,000
Fair value of net assets acquired 2,100,000
Additional Cost (40,000)
Net gain from business combination P60,000

7. On December 31, 2015, Seco Company paid P 950,000 for 95% of the outstanding common stock of Sana
Company. The remaining 5% was held by a stockholder who was unwilling to sell the stock. Sana's net assets
had a book value of P 810,000 and a fair market value of P 900,000 when it was acquired by Seco. If Sana
uses push- down accounting, the non- controlling interest should be reported at:

a. P 40,500 b. P 50,000 c. P 47,500 d. P 45,000

Answer: b. P 50,000

Solution:

Acquisition Cost P 950,000


Divided by: 95%
Total P 1,000,000
Multiplied by: 5%
Non- controlling Interest P 50,000

8. Ambrose Company acquires a controlling interest in Monica Company in the open market for P 220,0
00. The P 200 par value capital stock of Monica Company at the date of acquisition is P 250,000 and its
retained earnings amounts to P 100,000. The market value per share of Monica Company is P 220 per share.
In the consolidated statement of financial position on the date of acquisition, non- controlling interest would
show a balance of:

a. P 55,000 b. P 60,000 c. P 62,500 d. P 50,000

Answer: a. P 55,000

Solution:

Controlling (Parent) Interest:


Shares Acquired ( P 220,000/ P 220) 1,000 shares
Divided by Shares Outstanding ( P 250,000/ P 200)/ 1,250
Parent's Interest 80%
P 220,000/ 80% = P 275,000
Non- controlling/ Minority Interest in Net Assets of Subsidiary:
( P 275,000 x 20% ) = P 55,000

9. On August 31, 2016, Company P acquires 75% (750,000 ordinary shares) of Company S for P7,500,000
(P10 per share). In the period around the acquisition date, Company S's shares are trading at about P8 per
share. Company P pays premium over market because of the synergies it believes it will get. It is therefore
reasonable to conclude that the fair value of Company S as a whole may not be P10,000,000. In fact, an
independent valuation shows that the value of Company S is P9,700,000 ( fair value of Company S). Assuming
that the fair value of the net identifiable assets is P8,000,000 (carrying value is P6,000,000)

Goodwill arisung on consolidation is to be valued on the proportionate basis or "Partial" Goodwill:

a. P200,000
b. P1,500,000
c. P1,700,000
d. P2,000,000

Answer: B

Fair value of subsidiary:

Consideration transferred:. P7,500,000


Less:book value of Net assets
(P6,000,000 x 75%). 4,500,000
Allocates excess. P3,000,000
Less: over/under valuation of Assets and
Liabilities ((P8,000,000 - P6,000,000) x75% 1,500,000

Goodwill(partial). P1,500,000

10. Mark, a private limited company, has arranged filorman, a public limited company, to acquire it as a means
of obtaining a stock exchange listing. Man issue 15 million shares acquire the whole of the share capital of
Mask (6 million shares). The fair value of the net assets of Mask and Man are P30 million and P18 million
respectively. The fair value of each of the share of Mask is P6 and the quoted market price of Man's share is
P2. The share capital of Man is P25 million shares of acquisition. Compute the value of goodwill in the above
acquisition.

a. P16 million
b. P12 million
c. P 6 million
d. P10 million

Answer: C

Consideration transferred (4,000,000 shares* x P6) P24,000,000


Less: book value of equity Man .
(P18,000,000 x 100%). 18,000,000
Allocated excess P 6,000,000
Less: over/under valuation of assets and
liabilities( book value sme fair value). 0

Goodwill P 6,000,000

100%
Man > Mask
Currently issued . 15M 60%** 6M 60%
Additional shares issued. 10M 40% <4M /40%

Total shares. 25M. 10M

11. Condensed Statement of Financial Position of Dolce Inc. and Gabbana Inc. as of 12/31/2011 were as
follows:
Dolce Gabbana
Current assets 275,000 P65,000
Noncurrent 625,000 425,000
assets
Total assets 900,000 490,000
Liabilities 65,000 35,000
Ordinary shares, 549,700 296,700
P23 Par
Share Premium 35,300 28,300
Accumulated 250,000 130,000
Profits (losses)
On January 1, 2012, Dolce Inc. issued 30,000 shares with market value of P25/share for the assets and
liabilities of Gabbana Inc. Dolce Inc. also paid P125,000 cash. The book value reflects the fair value of the
assets and liabilities, except that the non-current assets of Gabbana Inc. have fair value of P630,000 and the
noncurrent assets of Dolce Inc. are overstated by P30,000. Contingent consideration, which is determinable, is
equal to P15,000. Dolce paid for the share issuance costs only amounting to P74,000 and incurred other
acquisition costs amounting to P19,000.

As a result of acquiring the net assets of Gabbana Inc., compute for the total liabilities in the books of Dolce.

a. P 100,000 b. P 115,000 c. P 134,000 d. P 65,000

Ans. C
Liabilities of Dolce P65,000
Liabilities of Gabbana P35,000
Contingent Consideration P15,000
Acquisition Cost Incurred P19,000
Total liabilities P134,000

12. On Dec. 31,2013, P Inc. paid P495,000 cash for all the outstanding stock of S Company. Ss assets and
liabilities on that day were as follows:

Cash P60,000
Inventory 150,000
P.P.E (net of accumulated dep. of P100,000) 350,000
Liabilities 70,000

On the day of business combination the fair value of the inventory was P125,000 and the fair value of
P.E (net) was P385,000. The goodwill (income from acquisition) resulting from this acquisition amounts
to:
a. (P5,000)
b. P85,000
c. P40,000
d. P5,000

Ans. A

Acquisition cost P495,000


Less: Book value of interest acquired (P560,000 P70,000) 490,000
Difference 5,000
Allocation:
Inventory P 25,000
Property and equipment ( 35,000) (10,000)
Income from acquisition P( 5,000)

13.The balance sheets of Pedro Ltd. and Santi Ltd. on June 30, 2016 were as follows:

Pedro Ltd Santi Ltd


Current assets P500 P700
Non-current assets 1, 300 3, 000
Total assets P1, 800 P3, 700
Share capital:
100 shares P300
60 shares P600
Retained earnings 800 1, 400
P1, 100 P2, 000
Current liabilities P300 P600
Non-current liabilities 400 1, 100
P700 P1, 700
Total equity and liabilities P1, 800 P3, 700

On July 1, Pedro Ltd. acquired all the issued shares of Santi Ltd. giving in exchange 2 Pedro Ltd. shares for
each ordinary share of Santi Ltd. Pedro Ltd, thus issued 150 shares to acquire the 60 shares issued by Santi
Ltd.

The fair value of each ordinary share of Santi Ltd. on July 1, 2016 is P40, while the quoted market price of
Pedro Ltd.s ordinary shares is P16. The fair values of Pedro Ltd.s identifiable assets and liabilities at
acquisition date are the same as their carrying amounts except for the non-current assets whose fair value was
P1, 500. The tax rate is 30%.

The amount of goodwill acquired on July 1, 2016:


a. P1, 160
b. P856
c. PP400
d. P360

Ans. : D
Solution:
Consideration transferred (40 shares* x P40) P1, 600
Less: Book value of SHE Pedro Ltd. (P300 + P800) x 100% 1,100
Allocated excess P500
Less: Over/under valuation of Assets and Liabilities:
Increase in Non-current assets: [(P1,500 P1,300) x 100% x 70%] 140
Goodwill P360

100%
Pedro Ltd. Santi Ltd.
Currently issued 150 60%** 60 60%
Additional shares issued 100 40% 40 40%
Total shares 250 100

**150/250

Pedro Ltd, issued 2 shared in exchange for each ordinary share of Santi Ltd. All of Santi Ltd.s shareholders
exchange their shares for Pedro Ltd. Pedro Ltd. Therefore issues 150 shares (60 x 2 ) for the 60 shares in
Santi Ltd.

Pedro Ltd. is now the legal parent of the subsidiary, Santi Ltd. However, analyzing the shareholding in Pedro
Ltd. shows that it consists of the 100 shares existing prior to the merger and 150 new shares held by former
shareholders in Santi Ltd. In essence, the former shareholders of Santi Ltd. now control both entities Pedro Ltd.
and Santi Ltd. The former Santi Ltd. shareholders have a 60% interest in Pedro Ltd [150/(100 + 150)]. The
IASB argues that there has been a reverse acquisition, and that Santi Ltd. is effectively the acquirer of Pedro
Ltd.

The key accounting effect of deciding that Santi Ltd. is the acquirer is that the assets and liabilities of Pedro
Ltd. are to be valued at fair value. This is contrary to normal acquisition accounting, based on Pedro Ltd. being
the legal parent of Santi Ltd., which would require the assets and liabilities of Santo Ltd. to be valued at fair
value.

14. Mask, a private limited company, has arranged for Man, a public limited company, to acquire it as a means
of obtaining a stock exchange listing. Man issues 15 million shares to acquire the whole of the share capital of
Mask (6 million shares). The fair value of the net assets of Mask and Man are P30 million and P18 million
respectively. The fair value of each of the shares of Mask is P6 and the quoted market price of Mans share is
P2. The share capital of Man is 25 million shares after the acquisition. Calculate the value of goodwill in the
above acquisition.

a. P16 million
b. P12 million
c. P10 million
d. P6 million

Ans. : D
Solution:
Consideration transferred (4, 000, 000 shares x P6) P24, 000, 000
Less: Book value of SHE Man: P18, 000, 000 x 100% 18, 000, 000
Allocated excess P6, 000, 000
Less: Over/Under valuation of Assets and Liabilities
(book value same fair value) 0
Goodwill P6, 000, 000

15. Clarisse Company acquires a controlling interest in Mimi Company in the open market for P120,000. The
P100 par value capital stock of Mimi Company at the date of acquisition is P125,000 and its retained earnings
amounts to P50,000. The market value per share of Mimi Company is P120 per share. In the consolidated
statement of financial position on the date of acquisition. Non controlling interest would show a balance of:

a. P30,000
b. P40,000
c. P25,000
d. P17

Ans. A

Controlling (Parent) interest:


Shares acquired (P120,000 / P120) 1,000 shares
Divided shares outstanding (P125,000 /P100) 1,250
Parents interest 80%

Minority interest in net assets of subsidiary (P150,000 x 20%) P30,000

16.On the day of acquisition Anne Inc. had the following assets and liabilities:

Book Value Fair Value


Current assets P100,000 P100,000
Plant assets(net) 220,000 260,000
Liabilities) (40,000) (40,000)
Sean Company paid P450,000 for 90% of the outstanding voting stock of Anne. The goodwill in the
consolidated statement of financial position at acquisition is:
P190,000
P120,000
P180,000
P230,000

Ans. C

Parent Company Interest P450,000


NCI 50,000
Consideration P500,000
Less: Fair value of net assets 320,000
Goodwill P180,000

17. Seminarian. Inc. has 100,000 shares of P2 par value stock outstanding. Priests Corporation acquired
30,000 shares of Seminarians shares on January 1, 20x4 for Pl20000 when Seminarians net assets had a
total fair value of P350000. On July 1, 20x7, Priests agreed to buy an additional 60,000 shares of Seminarian
from single stockholder for P6 per share Although Seminarians share 5. were selling in the P5 range around
July 1, 20x7. Priests forecasted that obtaining control of Seminarian would produce significant revenue
synergies to justify the premium price paid. if Seminarians net identifiable assets had a fair value of P500000
on July i, 20x7, how much goodwill on full fair value basis should Priests report in its, post-combination
consolidated balance sheet?

A. P -0-
B. P 60,000
C. P 90,000
D. P 100,000

ANSWER: B

60% FV, stocks issued: 60,000 shares x P6, fair value P 360,000
30% FV, of previously held equity interest: 30,000 shares x P5 fair
150,000
value
10% FV of NCI (100,000-60,000-30,000) x P4, fair value 40,000
100% FV of subsidiary P 560,000
Less: fair value of net asset of subsidiary 500,000

19. Robin Corporation purchased 150,000 previously unissued shares of Nest Company's $10 par value
common stock directly from Nest tor $3,400,000. Nest's stockholder's equity immediately before the investment
by Robin consisted of $3,000,000 of capital stock and $2,600,000 in retained earnings. What is the book
value of Robin's investment in Nest?

a. $1,500,000.
b. $1,680,000.
c. $2,800,000.
d. $3,000,000.

Answer: d

Shares outstanding before new shares are issued $ 300,000


Shares issued to Robin $ 150,000
Total shares outstanding $ 450,000
Percentage owned by Robin equals 150,000/450,000= 33.33%

Stockholders' equity before new shares are issued $ 5,600,000


+lnvestment by Robin $ 3,400,000
=Stockholders' equity after Robin investment $ 9,000,000
x Robin's percentage ownership 33.33%
=Book value of Robin's interest $ 3,000,000

20. Pogi Corporation paid P 100,000 cash for the net assets of Ganda Corporation which consisted of the
following:

Book Value Fair Value


Current Assets P 98,000 P 120,000
Property and Equipment P 350,000 P 400,000
Liabilities assumed P 100,000 P 110,000
The property and equipment acquired in this business combination should be recorded at what amount?

a. P 100,000
b. P 80,000
c. P 350,000
d. P 400,000

Answer: d

The Property and equipment should be recorded at its Fair Value of P 400,000

21.Ice, a private limited company, has arranged for Cream, a public limited company, to acquire it as a means
of obtaining a stock exchange listing. Cream issues 15 million shares to acquire the whole of the share capital
of Ica (6 million shares). The fair value of the net assets of Ice and Cream are P30 million and P18 million
respectively. The fair value of each of the shares of Ice is P6 and the quoted market price of Creams shares is
P2. The share capital of Man is 25 million shares after the acquisition. Calculate the value of goodwill in the
above acquisition.
a. P16 million c. P10 million
b. P12 million d. P 6 million

Ans: d

Consideration transferred (4 million shares* x P6) P24,000,000


Less: BV of SHE Cream: P18 million x 100% 18,000,000
Allocated excess P 6,000,000
Less: Over/Under valuation of Assets and Liabilities
(book value same fair value) 0
Goodwill P 6,000,000

22.Red, Inc. has 100,000 shares of P2 par value stock outstanding. Velvet Corporation acquired 30,000 shares
of Reds shares on January 1, 20x5 for P120,000 when Reds net assets had a total fair value of P350,000. On
July 1, 20x8, Velvet agreed to buy an additional 60,000 shares od Red from single stockholder for P6 per
share. Although Reds shares were selling in the P5 range around July 1, 20x8, Velvet forecasted that
obtaining control of Red would produce significant revenue synergies to justify the premium price paid. If Reds
net identifiable assets had a fair value of P500,000 on July 1, 20x8, how much goodwill on full fair value basis
should Velvet report in its post-combination consolidated balance sheet?
a. P 0 c. P 90,000
b. P60,000 d. P100,000

Ans: b

(60%) Fair value of consideration given


Shares: 60,000 shares x P6, fair value P 360,000
(30%) Fair value on previously held equity interest
30,000 shares x P5, fair value 150,000
(10%) Fair value of non-controlling interest
(100,000 60,000 30,000) x P5, fair value 50,000
(100%) Fair value of Subsidiary P 560,000
Less: Fair value of Net Assets (SHE of Subsidiary) 500,000
Goodwill (Full/Gross-up) P 60,000

23.Aquino Corp. acquired all the assets and liabilities of Binay Corp. by issuing shares of its common stock on
January 1, 2016. Partial balance sheet data for the companies prior to the business combination and
immediately following the combination is provided:
Aquino Binay
Book Value Book Value Combination
Cash. P 130,000 P 50,000 P 180,000
Accounts receivable.. 144,000 40,000 188,000
Inventory.. 66,000 90,000 176,000
Plant and equipment, net.. 800,000 300,000 1,300,000
Goodwill ?

Total Assets. P1,140,000 P480,000 P ?

Accounts payable P 100,000 P 50,000 P 150,000


Bonds payable. 500,000 200,000 700,000
Common stock, P2 par.. 200,000 50,000 320,000
Additional paid-in capital 130,000 40,000 490,000
Retained earnings.. 210,000 140,000 ?

Total Liabilities and Equities......... P1,140,000 P480,000 P ?

What number of shares and in what price did Aquino issue for this acquisition, as well as the amount of
goodwill to be reported by the combined entity immediately following the combination?
a. P80,000; P8; P450,000
b. P60,000; P8; P454,000
c. P40,000; P6; P456,000
d. P20,000; P6; P460,000
ANSWER: (b)
Common stock combined P320,000
Common Acquirer Aquino.. 200,000
Common stock issued P120,000
Divided by: Par value of common stock.. P 2
Number of Aquino shares to acquire Binay 60,000 (b)

Paid-in capital books of Aquino (P200,000 + P130,000) P330,000


Paid-in capital in the combined balance sheet
(P320,000 + P490,000).. 810,000
Paid in capital from the shares issued to acquire Binay. P480,000
Divided by the number of shares 60,000
Fair value per shares when sock is issued.............. P 8 (
b
)

Net identifiable assets of Aquino before acquisition:


(P130,000 + 144,000 + 66,000 + 800,000 -
100,000 500,000)........... P540,000
Net identifiable assets in the combined balance sheet:
(P180,000 + 188,000 + 176,000 + 1,300,000 -
150,000 700,000.. 994,000
Fair value of the net identifiable assets held by Binay
at the date of acquisition P454,000 (
b
)

24. Richard, Inc. is to acquire Raymond Corp. by absorbing all the assets and assuming all the liabilities to the
latter in exchange for shares of the formers stock. Below are the balance sheets of the two companies with the
corresponding appraised value increment for Raymond Corp.
Richard Raymond
Assets, per books P2,000,000 P1,250,000
Assets, appraised increase 150,000

Liabilities P 750,000 P 400,000


Common stock (No par; P100 par) 1,000,000 500,000
Additional paid-in capital. 350,000 150,000
Retained earnings (deficit). (100,000) 200,000

Total Equities P2,000,000 P1,250,000

The parties agree to use the appraised values, against which the fair market value of the shares will be
matched. Richard, Inc.s common stock is currently selling at P100 per share. The number of share to
be issued by Richard, Inc. is:
a. 20,000
b. 15,000
c. 13,000
d. 10,000
ANSWER: (d)
Assets at appraised value (P1,250,000 + P150,000) P1,400,000
Less: Liabilities. 400,000

Net assets at appraised values. P1,000,000


Divided by: Current selling price per share.. 100

Number of shares issued. 10,000

25. On July 1, 2016, Parent Ltd. acquired all the issued share capital of Sub Ltd. giving in exchange of
100,000 shares in Parent Ltd. these having a fair value of P5 per share. At acquisition date, the balance
sheets of Parent Ltd. and Sub Ltd. and the fair values of Sub Ltd's assets and liabilities, were as follows:

Parent Ltd. Sub Ltd.


Carrying Amount Carrying Amount Fair Value
EQUITY AND LIABILITIES
Equity
Share Capital P550,000 P300,000
Retained earnings 350,000 140,000
Total equity P900,000 P440,000
Liabilities
Provisions P 30,000 P 60,000 P 60,000
Payables. 27,000. 34,000. 34,000
Tax liabilities 10,000 6,000 6,000
Total liabilities P 67,000 P100,000
Total equity and liabilities P967,000 P540,000

ASSETS
Land P120,000 P150,000 P170,000
Equipment 620,000 480,000 330,000
Accumulated depreciation (180,000) (170,000)
Investment in subsidiary 500,000
Inventory. 92,000 75,000 80,000
Cash 15,000 5,000 5,000
Total Assets P967,000 P540,000

At acquisition date, Sub Ltd.has an unrecorded patent with a fair value of P20,000 and a contingent liability of
with a fair value of P15,000. The tax rate is 30%.

The amount of goodwill acquired on July 1, 2016:

a. P25,000
b. P15,000
c. P10,000
d. Zero

ANSWER: A

Consideration transferred (100,000 shares x P5) P500,000


Less. Book value of SHE - S: P440,000 x 100 440,000
Allocated excess P 60,000
Less: Over/Under validation of Assets and Liabilities
Increase in Land: P20,000 x 100% x 70% P14,000
Increase in Equipment: P20,000 x 100% x 70% 14,000
Increase in Inventory: P5,000 x 100% x 70% 3,500
Increase in Parent: P20,000 x 100% x 70% 14,000
Increase in Provision: P(15,000) x 100% x 70% (10,500) 35,000
Goodwill P 25,000

26. Oh January 1, 2016, Park Corporation and Stand Corporation and their condenser balance sheet are
as follows:
Park Corp. Strand Corp.
Current Assets P 70,000 P 20,000
Non-current Assets 90,000 40,000
Total Assets P160,000 P 60,000
Current Liabilities P30,000 10,000
Long-term Debt 50,000 -
Stockholders' Equity 80,000 50,000
Total Liabilities and Equities P160,000 P60,000

On January 2, 2016, Park Corporation borrowed P60,000 and used the proceeds to obtain 80% of the
outstanding common shares of Strand Corporation. The acquisition price was considered proportionate to
Stand's fair value.

The P60,000 debt is payable in 10 equal annual principals, plus interest, beginning December 31, 2016. The
excess fair value of the investment over the underlying book value of the acquired net assets is allocated to
inventory (60%) and to goodwill (40%).

On the consolidated balance sheet as of January 2, 2016, what should be the amount of each of goodwill using
proportionate basis (partial)?
a. P 0
b. P8,000
c. P10,000
d. P20,000

ANSWER: B

Fair Value of consideration given P60,000


Less: Book value of net assets (P50,000 x 80%) 40,000
Allocated excess P20,000
Less: Over/Under validation of Assets and Liabilities
Increase in Inventory (P20,000 x 60% = P12,000 / 80%
= P15,000 increase in inventory x 80% 12,000
Goodwill (partial). P 8,000

27. On June 12, 2015 Don Company purchases 8,000 shares of Sam Company for P68 per share.
Just prior to the purchase, Sam Co. has the following statement of financial position.

ASSETS Liabilities & Equity


Cash P 20,000 Current liabilities P 250,000
Inventory 280,000 Common Stock, P5 par 50,000
Equipment 400,000 APIC 130,000
Goodwill 100,000 Retained Earnings 370,000
Total Assets P 800,000 Total Liabilities and EquityP 800,000

On June 12,2015 Sams inventory has a fair value of P450,000 and that the equipment is worth
P600,000. What is the amount of non controlling interest in the consolidated statement of financial
position on the date of acquisition?
a. P128,000
b. P150,000
c. P164,000
d. P120,000

ANSWER: C
Rationale
Cash P 20,000
Inventory 450,000
Equipment 600,000
Total Assets 1,070,000
Liablities (250,000)
Net Assets P820,000
X 20%
NCI P164,000

28. Using the data in the preceding number what is the amount of goodwill (gain on acquisition) to be
reported in the consolidated statement of financial position on the date of acquisition?
a. P98,000
b. P100,000
c. P112,000
d. P106,000

ANSWER: C
Rationale
Acquisition Cost (8000 x 68) P544,000
NCI 164,000
Total P708,000
Less: Net Assets P820,000
Gain on Acquisition (P112,000)

29. The balance sheets of Min Ltd. and Kim Ltd on June 30,2017 were as follows:
Min Ltd Kim Ltd
Current assets P 600 P 800
Non-current assets 1 200 2 900
Total assets P1 800 P3 700

Current Liabilities P 250 P 700


Non-current liabilities 450 1 000
P700 P 1 700
Share Capital
100 shares P 300
60 shares P 600
Retained Earnings 800 1 400
P1 100 P2 000
Total equity and liabilities P1 800 P3 700

On July , 2017, Min Ltd acquired all the issued shares of Kim Ltd giving in exchange 21/2 Min Ltd shares for
each ordinary share of Kim Ltd. Min Ltd thus issued 150 shares to acquire the 60 shares issued by Kim Ltd.

The fair value of each ordinary share of Kim Ltd on July 1,2017is P40, while the quoted market price of Min
Ltds ordinary shares is P16. The fair values of Min Ltds identifiable assets and liabilities at acquisition date are
the same as their carrying amounts except for the non-current assets whose fair value was P1 500. The tax
rate is 30%.

The amount of goodwill acquired on July 1,2016 is


a. P1 160 c. P400
b. P 856 d. P360
Solution:
ANS: D
Consideration transferred (40 shares* x P40) P1 600
Less: Book value of SHE-Pedro Ltd (P300 + P800) x 100% 1 100
Allocated Excess P 500
Less: Over/Under valuation of Assets and Liabilities:
Increase in Non-current assets: [(P1 500 P1 300) x
100% x 70% 140
Goodwill P 360

100%
Min Ltd Kim Ltd
Currently issued 150 60% 60 60%
Additional shares issued 100 40% 40 / 40%
Total shares 250 100

**150/250

Min Ltd issues 21/2 shares in exchange for each ordinary share of Kim Ltd. all of Min Ltds shareholders
exchange their shares for Min Ltd. Min Ltd therefore issues 150 shares (60 x 21/2) for the 60 shares in Kim Ltd.

Min Ltd is now the legal parent of the subsidiary, Kim Ltd. however, analyzing the shareholding in Min Ltd
shows that it consists of the 100 shares existing prior to the merger and 150 new shares held by former
shareholders in Kim Ltd. In essence, the former shareholders of Kim Ltd now control both entities Min Ltd and
Kim Ltd. The former Kim Ltd shareholders have a 60% interest in Min Ltd[150/(100+150)]. The IASB argues
that there has been a reverse acquisition, and that Kim Ltd is effectively the acquirer of Min Ltd.

30. On July 1,2016, Naly Co. Acquired all the issued share capital of Lito Co. giving in exchange of 120,000
shares in Naly Co. these having a fair value of P5 per share. At acquisition date, the balance sheets of Naly
Co. And Lito Co. And the fair values of Lito Co.s assets and liabilities were as follows:
Naly Co. Lito Co.
Carrying Amount Carrying Amount Fair Value
ASSETS
Cash P 15,000 P 5,000 P 5,000
Inventory 92,000 75,000 80,000
Investment in Subsidiary
(Shares in Lito Co.) 500,000
Equipment 620,000 480,000 330,000
Accumulated depreciation (180,000) (170,000)
Land 120,000 150,000 170,000
TOTAL ASSETS P967,000 P540,000

LIABILITIES AND EQUITY


Liabilities
Payables P 27,000 P34,000 P34,000
Tax liabilities 10,000 6,000 6,000
Provisions 30,000
Total liabilities P 67,000 P 40,000
Equity
Share capital P550,000 P360,000
Retained earnings 350,000 140,000
Total equity P900,000 P500,000
TOTAL EQUITY AND LIABILITIES P967,000 P540,000

At acquisition date, Lito Co. Has an unrecorded patent with a fair value of P20,000 and a contingent liability
with a fair value of P15,000. The tax rate is 30%.

The amount of goodwill acquired on July 1,2016:


a. P65,000 c. P50,000
c. P45,000 d. Zero
Solution:
ANS: A
Consideration transferred (120,000 shares x P5) P600,000
Less: Book value of SHE-Lito P500,000 x 100% 500,000
Allocated excess P100,000
Less: Over/Under valuation of Assets and Liabilities:
Increase in Land: P20,000 x 100% x 70% P14,000
Increase in Equipment: P20,000 x 100% x 70% 14,000
Increase in Inventory: P5,000 x 100% x 70% 3,500
Increase in Patent: P20,000 x 100% x 70% 14,000
Decrease in Provision : (P20,000) x 100% x 70% ( 10,500) 35,000
Goodwill P 65,000

The net fair value of the subsidiary could be calculated by revaluing the assets and liabilities of the subsidiary
from the carrying amounts to fair value, remembering that under PAS No. 12 Income Taxes revaluation of
assets requires a recognition of the tax effect of the revaluation because there is a difference between the
carrying amount and the tax base caused by the revaluation.

31. On January 1, 2016, Park Corporation and Strand Corporation and their condensed balance sheet
are as follows:
Park Corporation Strand Corporation
Current Assets.. P70,000 P20,000
Non-current Assets.. 90,000 40,000
Total Assets.. P160,000 P60,000

Current Liabilities. P30,000 P10,000


Long term debt. 50,000 -
Stockholders Equity 80,000 50,000
Total Liabilities and Equities.. P160,000 P60,000

On January 2, 2016, Park Corporation borrowed P60,000 and used the proceeds to obtain 80% of the
outstanding common shares of Strand Corporation. The acquisition price was considered proportionate
to Strands fair value. The P60,000 debt is payable in 10 equal annual principal payments, plus
interest, beginning December 31,2016. The excess fair value of the acquired net assets is allocated to
inventory (60%) and to goodwill (40%).

On a consolidated balance sheet as of January 2, 2016, what should be the amount of goodwill using
proportionate basis (partial)?

a. P0
b. P8,000
c. P10,000
d. P20,000
Answer: B

Fair value of consideration given P60,000

Less: Book value of net assets (P50,000 x 80%) 40,000

Allocated excess P20,000

Less: Increase in inventory (P20,000x60%)/80%

=P15,000 increase in inventory x 80% 12,000

Goodwill (partial) P8,000

32.Pita Company acquires a controlling interest in Soda Company in the open market for P120,000.
The P100 par value capital stock of Soda Company at the date of acquisition is P100,000 and its
retained earnings amounts to P50,000. The market value per share of Soda Company is P150 per
share. In the consolidated statement of financial position, non-controlling interest would show a
balance of:
a. P80,000
b. P5,000
c. P30,000
d. P35,000
Answer: C

Controlling (Parent) interest:

Shares acquired (P120,000 / P150) 800 shares

Divided shares outstanding (P100,000 /P100) 1,000

Parents interest 80%

Minority interest in net assets of subsidiary (P120,000 / 80% x 20%) P30,000

33.AA Company acquired all the issued share capital of BB Company on March 31,2016 giving in exchange of
100,000 shares in AA Co. These having a fair value of P5 per share. At acquisition date, the balance sheets of
AA Co and BB Co. And fair values of BB Co.s assets and liabilities,were as follows:

AA Co. BB Co.

Carrying Amount Carrying Amount Fair Value

Assets:

Cash .................................................................. P 15,000 P 5,000 P 5,000

Inventory........................................................... 92,000 75,000 80,000

Investment in Subsidiary

(Shares in BB Co.) ........................................ 500,000

Equipment ...................................................... 620,000 480,000 330,000

Accumulated depreciation ............................. (180,000) (170,000)

Land .............................................................. 120,000 150,000 170,000

Total Assets.................................................. P 967,000 P 540,000

Liabilities and Equities:

Liabilities

Tax Liabilities ............................................. P 10,000 P 6,000 P 6,000


Payables....................................................... 27,000 34,000 34,000
Provisions................................................. 30,000 60,000 60,000

Total Liabilities .......................................... P 67,000 P100,000

Equity
Share Capital ............................................. P 550,000 P300,000
Retained Earnings ..................................... 350,000 140,000
P 900,000 P440,000

Total Liabilities and Equity ........................... P 967,000 P540,000

At acquisition date, BB Co. Has an unrecorded patent with a fair value of P20,000 ,contingent liability with a fair
value of P15,000, and one of the payables is a dividend payable of P8,000. AA Co. acquires the shares of BB
Co. On a cum div basis or dividends-on arrangement.The tax rate is 30%. The amount of goodwill acquired
on March 31,2016:

A.P 16,000
B. P 17,000
C.P 18,000
D.P 19,000

Answer: B

Consideration transferred[(100,000 shares x P5) - P8,000] ................................................. P 492,000


Less:Book Value of Shareholders Equity-BB Co:
P440,000 x 100% .................................................................................................. 440,000

Allocated Excess................................................................................................................. P 52,000


Less: Over/Under valuation of Assets and Liabilities:
Increase in Land: P20,000 x 100% x 70%..........................P 14,000
Increase in Equipment: P20,000 x 100% x 70%.................. 14,000
Increase in Inventory: P5,000 x 100% x 70%...................... 3,500
Increase in Patent:P20,000 x 100% x 70%...........................14,000
Increase in Provision: P(15,000) x 100% x 70%.................(10,500) 35,000
Goodwill .................................................................................................................... P 17,000 (B)

34.What would be the effect on the consolidated financial statements if an unconsolidated subsidiary is
accounted for by the cost method of accounting, but consolidated financial statements are prepared for other
subsidiaries?

a. All the unconsolidated subsidiary's ledger account balances would be included individually in the
consolidated financial statements.

b. Consolidated net income would not include any amounts for the unconsolidated subsidiary.

c. Consolidated net income would be the same as if the subsidiary had been included in the consolidation.

d. Dividend revenue from the unconsolidated subsidiary would be included in consolidated net income.

Answer: D

35. Under SFAS 141R, what value of the assets and liabilities is reflected in the financial statements on the
acquisition date of a business combination?

a. Carrying value
b. Fair value
c. Book value
d. Average value

Answer: b

36.The parent company owned 65% of subsidiarys net assets. On the consolidated statement of financial
position of the combined entity, the retained earnings has amount equal to:

a. The subsidiarys retained earnings.


b. The 65% of the subsidiarys retained earnings plus parent companys retained earnings.
c. The 100% of the subsidiarys retained earnings plus parent companys retained earnings.
d. The parent companys retained earnings.

ANSWER: (d)

Only the parent companys retained earnings will appear on the consolidated statement of financial
position.

37.Which is not true about the working paper for consolidated statement of financial position on the date of
acquisition?
a. The amounts in the consolidated column reflects the financial position of single economic entity comprising
two legal entities, after eliminating all intercompany balances
b. Consolidated retained earnings include only the retained earnings of the parent company
c. The elimination entry is recorded in the parent and subsidiarys accounting records
d. The consolidated paid-in capital amounts are those of the parent company only.
Ans. C

38.On December 31,2013, Palo Company paid P990,000 for 99% of the outstanding coomon stock of Sota
Company. The remaining 1% was held by a stockholder who was unwilling to sell the stock. Sotas net assets
had a book value of P850,000 and a fair market value of P900,000 when it was acquired by Palo. If Sota uses
push-down accounting, the non-controlling interest should be reported at:
a. P8,500
b. P9,000
c.P9,900
d. P10,000

Ans. B
Solution:
P900,000 x 1%= P9000
39. Rizal Corporation paid P 100,000 cash for the net assets of Bonifacio Corporation which consisted of the
following:

Book Value Fair Value


Current Assets P 98,000 P 120,000
Property and Equipment P 350,000 P 400,000
Liabilities assumed P 100,000 P 110,000

The property and equipment acquired in this business combination should be recorded at what amount?

a. P 400,000
b. P 80,000
c. P 350,000
d. P 100,000

Answer: A

40. The Property and equipment should be recorded at its Fair Value of P 400,000
On January 2, 2011, Bulalo Co. purchased 75% of Pares Cos outstanding common stock. On that date, the
fair value of the 25% noncontrolling interest was P35,000. During 2011, Pares had net income of P20,000.
Selected balance sheet data at December 31,2011, is as follows:

Bulalo Pares
Total assets P420,000 P180,000
Liabilities P120,000 P60,000
Common stock 100,000 50,000
Retained Earnings 200,000 70,000

During 2011 Bulalo and Pares paid cash dividends of P25,000 and P5,000 respectively, to their shareholders.
There were no other intercompany transactions.

In Bulalos December 31,2011 consolidated balance sheet, what amount should be reported as noncontrolling
interest in net assets?

a. P30,000
b. P35,000
c. P38,750
d. P40,000

ANSWER: C

Fair value of noncontrolling interest P35,000

Plus: Share of net income (25% x 20,000) 5,000


Less: Share of dividends (25% x 5,000) (1,250)

Noncontrolling interest P38,750

SUBSIQUENT DATE OF ACQUISITION


1. Watkins Inc. acquires all of the outstanding stock of Guen Corporation on January 1, 2016. At that date,
Guen owns only three assets and has no liabilities:

Book Value Fair Value


Inventory P40, 000 P50, 000
Equipment 80, 000 75, 000
Building 200, 000 300, 000

If Walkins pays P450, 000in cash for Guen, what amount would be represented at the subsidiarys building in a
consolidation at December 31, 2018, assuming the book value at that date is still P200. 000?
a. 200, 000 c. 285, 000
b. 255, 000 d. 300, 000

ANSWER:
Building, book value P200, 000
Increase to fair value (300, 000- 200, 000) 100, 000
Amortization of allocated excess (100, 000/20 x 3 years) (15, 000)
Consolidated building, 12/31/2018 P285, 000
2. On January 1, 2016, Harry Inc. reports net assets of P880, 000 although a patent (with a 10-year life) having
a book value of 330, 000is now worth P400, 000. Newt Corporation pays P840, 000 on that date for an 80%
ownership in Newt. On December 31, 2017, Harry reports total expenses of P621, 000 while Newt reports
expenses of P714, 000. What is the consolidated total expense balance on December 31, 2017?
a. 1, 197, 800 c. 1, 342, 000
b. 1, 335, 000 d. 1, 349, 000

ANSWER:
Harry expense P621, 000
Newt expense P714, 000
Amortization of allocated excess
(400, 000 330,000) / 10 years 7, 000 721, 000
Consolidated total expense for 2017 P1, 342, 000
3. Keep,Inc., a calendar year corporation, acquires 70% of George Company on September 1, 20x4 and an
additional 10% on April 1, 20x5. Total annual amortization of P6,000 relates to the first acquisition. George
reports the following figures for 20x5:

Revenues P500,000
Expenses 400,000
Retained earnings, 1/1/20x5 300,000
Dividends paid 50,000
Common Stock 200,000
Without regard for this investment,Keefe earns P300,000 in net income during 20x5. All net income is earned
evenly throughout the year. What is the controlling interest in consolidated net income for 20x5?

a. P371,500
b. P372,850
c. P373,300
d. P394,000

ANSWER: B

Net income from own operations;


Parent Keefe P 300,000
Subsidiary - George (P500,000 P400,000) 100,000
P 400,000
Less: Amortization of allocated excess 6,000
Impairment of goodwill (if any) 0
Consolidated/Group Net Income P 394,000
Less: Non-controlling interest in Net Income
Subsidiary net income from own operations:
1/1/20y0 - 4/1/20y0 (3 months):
P100,000 x 3/12 = P25,000 x 30% P 7,500
4/1/20y0 12/31/20y0 (9 months):
P100,000 x 9/12 = P75,000 x 20% 15,000
Total P 22,500
Less: Amortization of allocated excess:
1/1/20y0 4/1/20y0 (3 months)
P6,000 x 3/12 = P1,500 x 30% 450
4/1/20y0 12/31/20y0 (9 months)
P6,000 x 9/12 = P4,500 x 20% 900
Impairment of goodwill (if any):
First 3 months: P 0 x 30% 0
Remaining 9 months: P 0 x 20% 0 21,150
CNI attributable to the controlling interest (CI-CNI)/ Profit
attributable to equity holders of parent P372,850

4.On April 1, Narz Inc. exchages P430,000 fair value consideration fot 70% of the outstanding stock of Anne
Corporation. The remaining 30% of the outstanding shares continued to trade at a collective fair value of
P165,000. Annes identifiable assets and liabilities each had book values that equated their fair values on April
1 for a net total of P500,00. During the remainder of the year, Anne generates revenues of P600,000 and
expenses of P360,000 and paid no dividends. On a December 31 consolidated balance sheet, what amount
should be reported as a non-controlling interest on a full fair value basis?

a. P219,000
b. P237, 000
c. P287,000
d. P250,000
ANS: A

Book value of Stockholders Equity of Subsidiary 4/1/16 P500,000


Add: Net income for 9 months
(P600,000-P360,000= P240,000 x 9/12) 180,000
Book value of Stockholders Equity of Subsidiary 12/31/16 P680,000
Add: Adjustments to reflect fair value 0
Fair value of Stockholders Equity of Subsidiary 12/31/16 P680,000
Multiplied by: Non-controlling interes percentage 30%
Non-controlling interest (partial goodwill) P204,000
Add: Non-controlling interest in Full Goodwill
(P95,000 full -P80,000 partial) 15,000
Non-controlling Interest (full) P219,000

Partial Goodwill
Fair value of Subsidiary:
Fair value of consideration transferred: Cash P430,000
Less: Book value of Net Assets (Stockholders Equity-
Subsidiary): (P500,000x 70%) 350,000
Partial Goodwill P80,000

Full-goodwill
(70%) Fair value of consideration transferred: Cash P430,000
(30%) Fair value of non-controlling interests 165,000
(100%) Fair value of subsidiary P595,000
Less: Book value of Net Assets
(Stockholders Equity- Subsidiary) 500,000
Goodwill (Full/Gross-up) P95,000

5.On January 3, 2016, Ali Company acquired 80% of Frazer Corp.s common stock for P344,000 in cash. At
the acquisition date, the book values and fair values of Frazers assets and liabilities were equal, and the fair
value of the non-controlling interest was equal to 20% of the book value of Frazer. The stockholders equity
accounts of the two companies at the acquisition date are:

Ali Frazer
Common stock, P5 par value P500,000 P200,000
Additional paid in capital 300,000 80,000
Retained Earnings 350,000 150,000
Total Stockhoders Equity P1,150,000 P430,000

Non-controlling interest was assigned income of P11,000 in Alis consolidated income statement in
2016.

What will be the amount of net income reported by Frazer Corp. in 2016?

a. P44,000
b. P55,000
c. P66,000
d. P36,000

ANS: B
Full-goodwill Presentation (work back approach)
Non-controlling interest in net income P11,000
Divided by: Non-controlling interest 20%
Subsidiary-Frazer Net Income from his own operations P55,000

6. At the end of 2016, Micah Companys stockholders equity includes common stock of 500,000 and additional
paid in capital of 300,000.Paper purchased a 70% interest in Slick Company on January 1, 2016, when the
non-controlling interest in Slick had a fair value of 90,000. No differential arose from the business combination.
During 2016, Slick report net income of 20,000 and declares dividend of 5,000. The 2016 consolidated balance
sheet includes retained earnings of 630,000 (controlling interest portion).

Determine the consolidated equity on December 31, 2011:


a. 1,430,000
b. 1,457,000
c. 1,524,500
d. 1,526,000

ANS:C
Consolidated Equity:
Attributable to Equity Holders of Parent/ Controlling Interest:
Common stock 500,000
Additional paid-in capital 300,000
Retained earnings 630,000
--------------
Equity Holders of Parent/Controlling Interest 1,430,000
Non- Controlling interest:
(90,000+(20,000-5,000) X 30% 94,500
--------------
Consolidated Equity 1,524,500

7. Erhlyns Company acquired an 80% interest in the Aira Company when Airas equity comprised share capital
of 100,000 and retained earnings of 500,000. Airas current statement of financial position shows share capital
of 100,000, a revaluation reserve of 400,000 and retained earnings of 1400,000.

What figure in respect of Airas retained earnings should be included in the consolidated statement of financial
position?
a. 720,000
b. 1,440,000
c. 1,040,000
d. 1,520,000

ANS:A
This is the parent companys share of the post acquisition retained earnings of the subsidiary. This is
determined by deducting (i) the parent companys share of the retained earnings of the subsidiary of the date of
acquisition from (ii) the parent companys share of the retained earnings of the subsidiary at the end of the
current reporting period.

Airas retained earnings, date of acquisition 500,000


Less: Airas retained earnings, end of the current reporting period 1,400,000
900,000
X: Controlling interest % 80%
Airas retained earnings included in the consolidated financial position 720,000

8. Beta Company acquired 100 percent of the voting common shares of Standard Video Corporation, its bitter
rival, by issuing bonds with a par value and fair value of P150,000. Immediately prior to the acquisition. Beta
reported total assets of P500,000, liabilities of P280,000, and stockholders' equity at P220,000. At that date,
Standard Video reported total assets at P400,000, liabilities of P250,000., and stockholders' equity of
P150,000. Included in Standard's liabilities was an account payable to Beta in the amount at P20,000 which
Beta included in its accounts receivable.

Based on the preceding information: (1) what amount of total assets did Beta report in its balance sheet
immediately after the acquisition: (2) what amount of assets was reported in the consolidated balance sheet
immediately after the acquisition?

a. (1) P650,000; (2) P650,000


b. (1) P650.000; (2) P800,000
c. (1) P800,000; (2) P650,000
d. (1) P800.000: (2) P800000
Answer: B
Solution
Beta's Balance Sheet
Total assets of Beta Company before issuance of shares P 500,000
Add: Investment in Subsidiary (at fair value) 150,000
Total asset in the balance sheet of Beta Company P 650,000

Consolidated Balance sheet:

Beta's(parent) assets P 650,000


Standard Video's(subsidiary) assets 400,000
Total assets before eliminations P 1,050,000
Investment in subsidiary (150,000)
A/R from Standard (20,000)
Consolidated Total assets P 880,000

9. Montero Company is contemplating the purchase of the net assets of Toyota Company for P800,000 cash.
To complete the transaction, direct acquisition costs are P15,000. The balance sheet of Toyota Company on
the purchase date is as follows:

Toyota Company
Balance Sheet
December 31, 2014
Assets Liabilities and Equity
Current assets P 80,000 Liabilities P100,000
Land 50,000 Common Stock, P10 par 100,000
Building 450,000 Paid-in capital 150,000
Acc.depreciation (200,000) Retained earnings 230,000
Equipment 300,000
Acc. depreciation (100,000)
Total P580,000 Total P580,000
The following fair values were obtained for Toyotas assets and liabilities:
Current assets P100,000
Equipment P275,000
Land 75,000
Liabilities 102,000
Building 300,000
Determine the increase in assets that resulted from the business combination.
A. P 887,000
B. P 902,000
C. P 917,000
D. P 747,000
Answer: A
Solution
Fair value of assets acquired P750,000
Goodwill (800,000 - 648,000) 152,000
OPC (15,000)
Increase in Assets P887,000

10 . DMCI Company acquired 80% capital interest of STONERICH Company. DMCI paid P1,240,000 for the
80% interest and paid P88,000 for legal assistance (related to the acquisition). STONERICH net assets valued
at P1,200,000 composed of capital stock, P600,000; additional paid-in capital of P180,000, and retained
earnings of P420,000. At the time of acquisition, STONERICH building is undervalued by P100,000 and has
still a remaining life of 30 years. Any other excess is allocated to goodwill. STONERICH Company reported net
income of P140,000 and paid dividends of P20,000 during the year.

How much is the income from investment under the equity method?
a. P 109,333 b. P 112,000 c. P 99,733 d. P 108,667

Answer: A.
Cost of investment (1,240,000 + 88,000) P 1,328,000
Book value of investment (1,200,000 x 80%) 960,000
Excess of cost over book value P 368,000
Income from investment:
Share (140,000 x 80%) P 112,000
Amortization allocated to building
(100,000 x 80%/30 yrs.) 2,667
P 109,333

11. Western Company, buys all of the outstanding stock of Abenson Company on January 1, 2014. Annual
excess amortizaton of P12,000 results from this purchase transaction. 0n the date of the takeover, Western
reported retained earnings of P400,000 while Ahenson reported a P200,000 balance. Western reported income
of P40,000 in 2014 and P50,000 in 2015 and paid 10,000 in dividends each year. Abenson reported net
income of P20,000 in 2014 and P30,000 in 2015 and paid P5,000 in dividends each year.
Assume that Western's reported income does indude income derived from the subsidiary.

If the parent uses the cost method of accounting investment in subsidiary, what are the consolidated retained
earnings on December 31, 2015?

a. P470,000 b. P510,000 c. P446,00 d. 486,000

Answer: D.
Western's retained eamlngs at the date of takeover P400,000
Add: Reponed net income of Wetem: 2014 and 2015 (40,000+50,000) 90,000
Less: Dividends paid for 2 years (10,000 x 2) ( 20,000)
Add: Undistributed nt Income of Abenson for 2 years
(20,000 + 30,000-5,000-5,000) 40,000
Annual excess amortization for 2 years (12,000 X 2) 24,000
Consolidated Retained Earnings, December 31,2015 P486,000
12. Coco Company 's CI during the year was P 250,000, it declared dividends of P 90,000 and the depreciation
and amortization of current fair value excess was P 50,000. If it was acquired last year by Nut Company as its
wholly owned subsidiary, the NCI in CI of subsidiary under the cost method of accounting for the current year
is:

a. P 150,000 b. P 160,000 c. P 200,000 d. P -0-

Answer: d. P -0-

Because there is no NCI in a wholly owned subsidiary.

13. Alonte Corporation holds 80% of Ronnie Company and uses the cost method in accounting for its
investment. During 2015, Ronnie Company reported CI of P 80,000 and paid dividends of P 60,000. There was
no purchase difference at the time of investment. What amount of Consolidated CI attributable to parent will be
reported in 2015?

a. P 144,000 b. P 164,000 c. P 156,000 d. P 80,000

Answer: b. P 164,000

Solution:
Alonte CI P 140,000
Parent's share of Subsidiary Net Income
( P 80,000 x 80% ) 64,000
Less: Dividends Received by Alonte
( P 50,000 x 80% ) 40,000
Consolidated CI Attributable to Parent P 164,000

14. On January 1, two years ago, Pab Corporation purchased all of the outstanding common stock of Shaw
Company for P220,000 cash. On that date, Shaw's net assets had a book value of P148,000. Equipment with
an 8-year life was undervalued by P20,000 in Shaw's financial records. Shaw has a database that is valued at
P52,000 and will be amortized over ten years. Shaw reported net income of P25,000 in the year of acquisition
and P32,500 in the following year. Dividends of P2,500 were declared and paid in each of those two years.

The third year of operations is now complete. For each of the two companies, selected account balances as of
December 31 for this third year are as follows:

What is consolidated retained earnings at January 1 of the third year if the parent company uses the initial
value method?

a. P191,100
b. P192,500
c. P187,000
d. P134,600

Answer: D

Pab Company' s retained earnings at january 1 of third year. P150,000


Additional equity: first year (P25,000 P2,500). 22,500
Additional equity: second year (P32,500 P2,500). 30,000

Amortization for 2 years (P7,700/2years). (15,400)


Consolidated Retained Earnings at january 1 of 3rd year P187,100

15. Peter, Inc. owns 100% of The Rock Company. The book value of the Goodwill is P300,000. When Peter
made its investment, The Rock had a fair value of P2,800,000. Today, the value of The Rock has fallen to
P2,250,000. An appraisal of The Rock's net assets reveals a fair value of P2,075,000. How much "impairment"
should Peter record related to its investment in The Rock?

a. P550,000
b. P175,000
c. P0
d. P125,000

Answers: D

Fair value of Pink' s investment P 2,250,000


Fair value of the Red' s Assets. P 2,075,000
Goodwill P 175,000
Carrying amount of Goodwill. (300,000)
Goodwill Impairment. ( P 125,000 )

16. Leslie Products, Inc purchased 60% of the stock of Edz Cream Company on Jan. 2, 2016 for P180,000. On
that date Edz reported retained earnings of P100,000 and had P200,000 of stock outstanding. Leslies retained
earnings was P400,000 at acquisition. Leslie accounts for its investment in Edz under the cost method. The
companies recorded the following results for 2016 and 2017:

Leslie Edz
2016: CI P70,000 P35,000
Dividends paid P25,000 P30,000
2017: CI P90,000 P40,000
Dividends paid P30,000 P15,000

What amount of consolidated CI attributable to parent and consolidated retained earnings will be reported in
2017?

a. P121,000 and P523,000, respectively


b. P105,000 and P523,000, respectively
c. P105,000 and P525,000, respectively
d. P121,000 and P533,000, respectively

Ans. C
Consolidated net income 2017
Net income Leslie P 90,000
Dividend income (P15,000 x 60%) (9,000)
Edz net income 40,000
MINIS (P40,000 x 40%) (16,000)
Consolidated net income attributable to parent 2017 P105,000

Consolidated retained earnings 2017


Retained earnings, Jan. 2, 2016- Leslie P 400,000
Consolidated net income attributable to parent 2016:
Net income Leslie P70,000
Dividend income (P30,000 x 60%) (18,000)
Edz net income 35,000
MINIS (P35,000 x 40%) ( 14,000) 75,000
Dividends paid, 2016 Leslie (25,000)
Consolidated retained earnings, Dec. 31, 2016 P450,000
Consolidated net income attributable to parent 2017 105,000
Dividends paid. 2017 Leslie (30,000)
Consolidated retained earnings, Dec. 31, 2017 P525,000

17. For the year ended Feb. 28, 2016, S Company, the 90% owned purchased subsidiary of P Corporation,
declared a dividend of P100,000 and had CI of P300,000. Also for that year, amortization of the current fair
value differences of Ss identifiable net assets was P60,000. The balance of NCI in CI of Subsidiary account on
Feb. 28, 2016, is:
a. P24,000
b. P21,000
c. P24,900
d. P20,000

Ans. A
Ss net income P300,000
Amortization of allocated difference ( 60,000)
Adjusted net income of S P240,000

Minority interest in net income of subsidiary (P240,000 x 10%) P 24,000

18. On January 1, 2015, Wilhelm Corporation acquired 90% of Kaiser Companys voting stock, at underlying
book value. The fair value of the non-controlling interest was equal to 10% of the book value of Kaiser at that
date. Wilhelm uses the equity method in accounting for its ownership of Kaiser. On December 31, 2016, the
trial balances of the two companies are as follows:
DebitCreditDebitCredit
Current assetsP200, 000P140, 000
Depreciable assets350, 000250, 000
Investment in Kaiser Company162, 000
Depreciation expense27, 00010, 000
Other expenses95, 00060, 000
Dividends declared20, 00010, 000
Accumulated depreciationP118, 000P80, 000
Current liabilities100, 00080, 000
Long-term debt100, 00050, 000
Common stock100, 00050, 000
Retained earnings150, 000100, 000
Sales 250, 000
Income from Subsidiary36, 000
P854, 000P854, 000P470, 000P470, 000

Based on the preceding information, what amount would be reported retained earnings in the consolidated
balance sheet prepared at December 31, 2016?
a. P424, 000
b. P314, 000
c. P294, 000
d. P150, 000
Ans. : C
Solution:
Parents (Wilhelm) Retained earnings, 1/1/2015*P150, 000
Add: CNI attributable to the controlling interest (CNI CI)/
Profit attributable to equity holders of parent164, 000
Less: Dividends Parent (Wilhelm)20, 000
Parents (Wilhelm) Retained earnings, 12/31/3015
(Equity method) or Consolidated Retained EarningsP294, 000

Net income from own operations:


ParentP128, 000
Subsidiary40, 000
P168, 000
Less: Amortization of allocated excess
Impairment of full-goodwill (if any)
Consolidated/Group Net Income
Less: Non-controlling interest in Net Income:
Subsidiary income from own operationsP40, 000
Less: Amortization of allocated excess
Impairment of full-goodwill (if any)
P40, 000
X Non-controlling interests10%4, 000
CNI attributable to the controlling interest (CNI NI)/
Profit attributable to equity holders of parentP164, 000

Net income from own operations


Parent Subsidiary
SalesP250, 000P110, 000
Other expenses95, 00060, 000
Depreciation expense27, 00010, 000
Net income from own operationsP128, 000P40, 000

*It should be noted that since equity method was used, the retained earnings on January 1, 2015 is also
considered as the consolidated retained earnings.

19. On January 1, 2016, Plimsol Company acquired 100% of Shipping Corporations voting shares, at
underlying book value. Plimsol uses the cost method in accounting for its investment in Shipping. Shippings
retained earnings were P75, 000 on the date of acquisition. On December 31, 2016, the trial balance data for
the two companies are as follows:
DebitCreditDebitCredit
Current assetsP100, 000P75, 000
Depreciable assets (net)200, 000150, 000
Investment in Shipping Company125, 000
Depreciation expense20, 00015, 000
Other expenses60, 00045, 000
Dividends declared25, 00015, 000
Current liabilities40, 00025, 000
Long-term debt75, 00050, 000
Common stock100, 00050, 000
Retained earnings150, 00075, 000
Sales150, 000100, 000
Dividends income15, 000
P530, 000P530, 000P300, 000P300, 000

Based on the information provided what amount of retained earnings will be reported in the consolidated
balance sheet prepared on December 31, 2016?

a. P310, 000
b. P235, 000
c. P225, 000
d. P210, 000

Ans. B
Solution:
Parents (Plimsol) Retained earnings, 1/1/2016*P150, 000
Add: CNI attributable to the controlling interest (CNI- NI)/
Profit attributable to equity holders of parent**110, 000
Less: Dividends Parents (Plimsol)25, 000
Parents (Plimsol) Retained earnings, 12/31/2016
(Equity method) or Consolidated Retained EarningsP235, 000

Or alternatively. Consolidated Retained Earnings can also be determined by using the Retained Earnings of
Parent on December 31, 2016 under cost method (model). Then adjust the Retained Earnings of Parent under
the cost method (model) on December 31, 2016 from cost to equity method from the date of acquisition to
arrive at Retained Earnings of Parent on December 31, 2016 under the equity method, which will eventually be
considered as the Consolidated Retained Earnings on December 31, 2016. Thus, the computation is as
follows:

Parents (Plimsol) Retained Earnings, 1/1/2016*P150, 000


Add: Reported Net Income of Parent 2016 (cost model):
Net income of Parent from its own operationsP70, 000
Add: Dividend Income15, 00085, 000
Less: Dividends declared Parent (Plimsol)25, 000
Parents (Plimsol) Retained Earnings, 12/31/2016 (cost model)P210, 000
Retroactive adjustment to convert cost model to equity method since the date of acquisition for purposes of
consolidation / Parents share of adjusted net increase is subsidiarys retained earnings:
Retained earnings Subsidiary, 1/1/2016P75, 000
Retained earnings Subsidiary, 12/31/2016:
Retained earnings Subsidiary, 1/1/2016P75, 000
Add: Net income of Subsidiary 201640, 000
Less: Dividends declared15, 000100, 000
Increase in retained earnings or cumulative net income less cumulative dividends25, 000
Parents (Plimsol) Retained Earnings (equity method) or Consolidated Retained Earnings, 12/31/2016P235,
000

*It should be noted that on the date of acquisition, the retained earnings of parent is considered also as the
consolidated retained earnings regardless of the method (cost or equity) used, but not on subsequent years.
**Net income from own operations:
ParentP70, 000
Subsidiary40, 000
P110, 000
Less: Amortization of allocated excess
Impairment of full-goodwill (if any)
Consolidated/Group Net IncomeP110, 000
Less: Non-controlling interest in Net Income:
Subsidiary net income from own operations
Less: Amortization of allocated excess
Impairment of full-goodwill (if any)
P40, 000
X Non-controlling interest 0%
CNI attributable to the controlling interest (CN NI) / Profit attributable to equity holders of parentP110, 000

Net income from own operations


ParentSubsidiary
SalesP150, 000P100, 000
Other expenses60, 00045, 000
Depreciation expense20, 00015, 000
Net income from own operationsP70, 000P40, 000

20. Jay Corporation holds 70 percent of Shane Company and uses the cost method in accounting for its
investment. During 2015, Shane Company reported CI of P70,000 and paid dividends of P40,000. Jay reported
CI (including dividend income) of P130,000 and paid dividends of P50,000. There was no purchase difference
at the time of investment. What amount of consolidated CI attributable to parent will be reported for 2015?

a. P151,000
b. P172,000
c. P102,000
d. P 70,000

Ans. B
Net income Pablo P130,000
Dividend income (P40,000 x 70%) (28,000)
Sitos net income 70,000
MINIS (P70,000 x 30%) (21,000)
Consolidated net income attributable to parent P151,000

21. Denver Co. acquired 60% ot the common stock of Kailey Corp. on September 1, 20x4. For 20x4, Kailey
reported revenues of P800,000 and expenses at P620,000. The annual amount of amortization related to this
acquisition was P15,000. Denver Co. accounts for its consolidations according PFRS 3.
In consoiidation. the totai amount of expenses related to Kailey and to . Denvers acquisition at Kailey tor 20x4
is determined to be
A. P206,667
B. P211,667
C. P620,000
D. P635,000

ANSWER: B
As a general rule, if problem is silent it is assumed that expenses are generated evenly throughout the
year, thus:
Expenses (9/1/20x4-12/31/20x4): P620,000 x 4/12 P206,667
Amortization of allocated excess: P15,000 x 4/12 5,000
P 211,667

22. Pelican Corporation acquired a 30% interest in Crustacean Incorporated at book value several years ago.
Crustacean declared $100,000 dividends in 2005 and reported its income for the year as follows:

Income from continuing operations $ 700,000


Loss on discontinued division (100,000)
Net income $ 600,000

Pelicans Investment in Crustacean account for 2005 should increase by

a. S 150,000
b. S 160,000
c. S 180,000
d. S 210,000

Answer: a

Pelicans share of income


equals $600,000x 30% = $ 180,000
Pelicans share of
dividends = $100,000 x 30% $ (30,000)
Increase in investment account $ 150,000

23. Xing Corporation owns 80 percent of the voting common shares of Adams Corporation. Noncontrolling
interest was assigned $24,000 of income in the 2009 consolidated income statement. What amount of net
income did Adams Corporation report for the year?

a. $ 150,000
b. $ 96,000
c. $ 120,000
d. $ 30,000

Answer: a

NCI share in income $ 24,000


Divided by NCI: 20%
Adams Net Income $ 120,000

Teri Corporation acquired 80% of Yaki Companys stock on January 1, 20x5. At the acquisition date, Yaki had
the following account balances:

Book Value Market Value Remaining Life


Cash and Receivables P 30,000 P 30,000 3 months
Inventory 100,000 120,000 5 months
Plant Assets (net) 250,000 290,000 8 years
Liabilities 150,000 160,000 5 years
Common Stock 10,000
Retained Earnings 220,000

Yaki has income of P80,000 and pays dividends of P20,000 during 20x6. Assuming there is no goodwill
impairment, what is the amount of income allocated to the non-controlling interest for 20x6?
a. P15,400
b. P19,400
c. P14,600
d. P18,600
Ans: c

Net Income P80,000


Less: Plant Assets P290,000 (MV)
250,000 (BV)
P 40,000/8 yrs (5,000)
Liabilities P160,000
150,000
P 10,000/5 yrs (2,000)
Consolidated Income P73,000
NCI rate 20%
Income allocated to NCI P14,600

SD 2.
French Industries acquired an 80% interest in Fries Company by purchasing 24,000 of its 30,000 outstanding
shares of common stock at book value of P105,000 on January 1, 20x4. Fries reported net income in 20x4 of
P45,000 and in 20x5 of P60,000 earned evenly throughout the respective years. French received P12,000
dividends from Fries in 20x4 and P18,000 in 20x5. French uses the equity method to record its investment.
What is the balance of Frenchs Investment account in Fries account at December 31,20x5?
a. P105,000
b. P138,600
c. P159,000
d. P165,000

Ans: c

Investment (1/1/20x4) P105,000


Add: Share in net income 20x4 (P45,000 x 80%) 36,000
Less: Dividends received 12,000
Investment (12/31/20x4) P129,000
Add: Share in net income 20x5 (P60,000 x 80%) 48,000
Less: Dividends received 18,000
Investment (12/31/20x5) P159,000

26.On January 1, 2016, Speed Co. purchased 75% of the common stock of Slow Co. for P632,000. On
this date, Slow Co. had common stock, other paid-in capital, and retained earnings of P80,000,
P240,000, and P380,000, respectively. Speed Co.s common stock amounted to P1,000,000 and
retained earnings of P400,000.
On January 1, 2016, the only tangible assets of Slow Co. that were undervalued were inventory and
equipment. Inventory, for which FIFO is used, was worth P10,000 more that cost. The inventory was
sold in 2016. Equipment, which was worth P30,000 more than book value, has a remaining life of 8
years, and straight-line method is used. Any remaining excess is full-goodwill with an impairment for
2016 amounting to P6,000. Slow Co. reported net income of P 100,000 and paid dividends of P40,000.
Compute the Consolidated Net Income Attributable to Controlling Interest and Non-controlling interest
respectively using Full-Goodwill:

a. P250,812.50; P21,937.50
b. P254,812.50; P20,937.50
c. P250,000.00; P21,000.00
d. P255,000.00; P25,000.00
ANSWER: (a)

Net income from own operations:


Speed [P200,000 (P20,000 x 75%)] P 185,000
Slow. 100,000
P 285,000
Less: Amortization of allocated excess.. 6,250
Impairment of full-goodwill.. 6,000
Consolidated/Group Net Income. P 272,750
Less: Non-controlling interest in Net Income
Slow net income from own
operations. P100,000
Less: Amortization of allocated excess. 6,250
Impairment of full-goodwill 6,000
87,750

x: Non-controlling interest 25% P 21,937.50

CNI attributable to the controlling interest P250,812.50

27. On January 1, 2016, Steven Corp. acquired 80% of Kevin Corp. in exchange for 2,700 shares of
P10 par common stock having a market value of P60,300. Steven Corp. and Kevin Corp. condensed
balance sheets were as follows:

Steven Corporation and Kevin Corporation


Balance Sheets at January 1, 2016
(before combination)

Steven Kevin
Corp. Corp.
Assets:
Cash. P15,450 P18,700
Accounts receivable (net). 17,100 4,550
Inventories 11,450 8,050
Building. 89,500 20,000
Patents.. - 5,000
Total assets P133,500 P56,300
Liabilities and Equities:
Accounts payable. P 2,000 P 3,300
Bonds payable, 10%......................................................... 50,000 -
Common stock, P10 par.. 50,000 25,000
Additional paid-in capital. 7,500 7,500
Retained earnings. 24,000 20,500
Total liabilities and equities... P133,500 P56,300

At the date of acquisition, all assets and liabilities of Kevin Corp. have book value approximately equal
to their respective market values except the following as determined by appraisal as follows:

Inventories (FIFO method).. P 8,550


Building (net remaining life 4 years) 24,000
Patents (remaining life 10 years) 6,500
Goodwill (no impairment).
In what amount of partial goodwill on January 1, 2016:
a. P13,000
b. P13,100
c. P12,900
d. P12,100

ANSWER: (b)

Consideration given P60,300


Less: Book Value of SHE Kevin Corp. 1/1/2016:
C/S K: P25,000 x 80%........................................ P20,000
APIC K P7,500 x 80%......................................... 6,000
RE K P 20,500 x 80%.......................................... 16,400 42,400
Allocated Excess. P17,900
Less: Over/Under valuation of Assets and Liabilities:
*Increase in Inventories (P500 x 80%).. P400
*Increase in Building (P4,000 x 80%). 3,200
*Increase in Patents (P1,500 x 80%).. 1,200 4,800

Goodwill partial P13,100

Book Values Fair Values Increase


*Inventories P 8,050 P 8,550 P 500
Building. 20,000 24,000 4,000
Patents.. 5,000 6,500 1,500
P6,000

28. The Pony Company acquired all of the outstanding stock of Stag Company on January I, 2011, for
P206,000 in cash. Stag had a book value of only P140,000 on that date. However, equipment (having an eight-
year life) as undervalued by P40,000 on Stags financial records. A building with a 20-year life was overvalued
by 10,000. Subsequent to the acquisition, Stag reported the following:

CI Dividends Paid
2011 50,000 10,000
2012 50,000 40,000
2013 30,000 20,000

In accounting for this investment Pony has used the cost method. Selected accounts taken from the financial
records of these two companies as of December 31, 2013, are as follows:

Pony Company Stag Company


Revenues - Operating P310,000 P104,000
Expenses 198,000 74,000
Equipment (net) 320,000 50,000
Buildings (net) 220,000 68,000
Common stock 290,000 50,000
Retained Earnings, 12/31/2013 Balance 410,000 160,000

What amount should be reporter as consolidated retained earnings at December 31, 2013?
a. P136,500
b. P137,500
c. P142,000
d. P122,000

ANSWER: B

Consolidated Net Income


Net income from own operations - Pony
(P310,000 P198,000) P 112,000
Net income from own operations - Stag
(P104,000 P74,000) 30,000
Amortization: Equipment (P40,000/8) P5,000
Buildings (P10,000/20) (500) ( 4,500)
Consolidated net income P 137,500

29. Reyes Corporation holds 70% of Oyama Company, and uses the cost method in accounting for its
investment. During 2016, Oyama Company reported CI of P80,000 and paid dividends of P50,000.
Reyes reported CI (including dividend income) of P250,000 and paid dividends of P50,000. There was
no purchase difference at the time of investment. What amount of consolidated CI attributable to parent
will be reported for 2016?

a. P295,000
b. P271,000
c. P215,000
d. P80,000
Answer: B

Net income from own operations - Reyes P250,000

Less: Dividend income (P50,000 x 70%) (35,000)

Oyamas net income 80,000

NCI in Oyamas net income (P80,000 x 30%) (24,000)

Consolidated net income attributable to parent P271,000


30. Fave Inc., purchased 80% of Gade Companys outstanding common stock for P280,000, P80,000
above the underlying book value on January 2,2015. The fair value of Gades net assets approximated
book value. On the December 31 2015 consolidated statement of financial position, NCI should be
reported at:
a. P52,000
b. P46,000
c. P70,000
d. P64,000

ANSWER: C
Rationale
Consideration Transferred P280,000
Divided by: % of ownership 80%
Total FV of Gades assets P350,000
Multiplied by: % of NCI 20%
NCI P70,000
31.If a wholly owned subsidiarys CI was P150,000, the subsidiary declared dividends of P80,000 and the
depreciation and amortization of current fair value excess was P20,000, the NCI in CI of subsidiary under
the cost method of accounting is:
a. P100,000
b. P70,000
c. P-0-
d. P130,000

ANSWER: C
Zero because there is no NCI in a wholly owned subsidiary.

32. on January 1,2016, Bullet Company acquired 80 percent of Electric Companys common stock for P300,000
cash. At that date, Electric reported common stock outstanding of P200,000 and retained earnings of P100,000,
and the fair value of the noncontrolling interest was P75,000. The book values and fair values of Electrics assets
and liabilities were equal, except for other intangible assets which had a fair value P75,000 greater than book
value and a 6-year remaining life. Electric reported the following data for 2015 and 2016:
Year Net Income Comprehensive Income Dividends Paid
2016 P 25,000 P30,000 P 5,000
2017 40,000 45,000 10,000
Bullet reported separate net income from own operations of P130,000 and paid dividends of P30,000 for both
years.
What is the amount of consolidated comprehensive income reported for 2016?
a. P145,000 c. P118,500
b. P147,500 d. P130,000
Solution:
ANS: B
Fair value of consideration given P300,000
Fair value of noncontrolling interest 75,000
Fair value of the Subsidiary P375,000
Less: Book Value of Stockholders equity of subsidiary
(P200,000 + P100,000) 300,000
Allocated excess P 75,000
Less: Over/Under valuation of Assets and Liabilities:
Increase in Intangible Assets 75,000
Amortization of allocated excess: P75,000/6years P 12,500

Note: Since the allocated excess is attributable to undervalued intangible asset, it does not make sense, whether
partial or full goodwill method is used.

Consolidated Comprehensive Income:


Net income from own operations:
Parent-Bullet P130,000
Subsidiary-Electric 25,000
P155,000
Less: Amortization of allocated excess 12,500
Impairment of full-goodwill (if any) 0
Consolidated/Group Net Income P142,500
Add: Other Comprehensive Income
(P30,000 P25,000) 5,000
Consolidated Comprehensive Income P147,000
33.On March 31,2016, Apple Inc. exchanges P580,000 fair value consideration for 80% of the outstanding stock
of Cherry Inc. The remaining 20% of the outstanding shares continued to trade at a collective fair value of
P165,000. Cherrys identifiable assets and liabilities each had book values that equaled their fair values on March
31 for a net total of P500,000. During the remainder year, Cherry generates revenues of P900,000 and expenses
of P360,000 and paid no dividends. On a December 31 consolidated balance sheet, what amount should be
reported as non-controlling interest on a full-fair value basis?

A.P246,000
B.P181,000
C.P 65,000
D.P165,000

Answer: A

Book value of Stockholders Equity of Subsidiary 3/31/16................................................P 500,000


Add: Net income ( P900,000 - P360,000= P540,000x 9/12) ............................................. 405,000

Book value of SHE of Subsidiary 12/31/16.......................................................................P 905,000


Add: Adjustments to reflect fair value ................................................................................ 0

Fair value of SHE of Subsidiary 12/31/16..........................................................................P 905,000


Multiplied by Non-controlling interests percentage............................................................ 20%

Non-controlling interest (Partial Goodwill)....................................................................... P 181,000


Add:Non-controlling interest in Full Goodwill[P245,000(full)-P180,000(partial)
................................................................ 65,000

Non-controlling Interest (Full)..................................................................................... P246,000 (A)

Computation and allocation of Goodwill:


Total Fair Value Parent Price(80%) NCI Value(20%)
Company Fair Value P745,000 P580,000 P165,000
Fair Value of net assets excluding 500,000 400,000 100,000
goodwill
GOODWILL P 245,000 P 180,000 P 65,000

34.On December 31,2016, Long Companys stockholders equity includes common stock of P1,000,000 and
additional paid-in capital of P600,000. Long Co. Purchased a 90% interest in Short Co. on January 1,2016, when
the non-controlling interest in Short Co. Had a fair value of P230,000. No differential arose from the business
combination . During 2016, Short Co. Reports net income of P20,000 and declares dividend of P5,000. The 2016
consolidated balance sheet includes retained earnings of P630,000(controlling interest portion). The consolidated
equity on December 31,2016:

A.P2,230.000
B.P2,254,500
C.P2,253,000
D.P2,205,500

Answer: B

Consolidated Equity:
Attributable to Parent/Contolling Interest:
Common Stock...............................................................................................P1,000,000
Additional paid-in capital.............................................................................. 600,000
Retained Earnings........................................................................................ 630,000

Equity Holders of Parent/Controlling Interest.............................................P 2,230,000


Non-controlling Interest:
[(P230,000 + (P20,000-P5,000] x 10% ................................................... 24,500

Consolidated Equity...........................................................................................P 2,254,500 (B)

35. On January 1, 2016, Parent Company purchased 80% of the common stock of Subsidiary Company for P316,
000. On this date, Subsidiary Company had a common stock, other paid-in capital, and retained earnings of P40,
000, P120, 000, P190, 000, respectively. Parent Companys common stock amounted to P500, 000 and retained
earnings of P200, 000.
On January 1, 2016, the only tangible assets of Subsidiary that were undervalued were inventory and building.
Inventory, for which FIFO is used, was worth P5, 000 more than cost. The inventory was sold in 2016. Building
which was worth P15, 000 more than book value, has a remaining life of 8 years and straight-line depreciation is
used. Any remaining excess is full-goodwill with impairment for 2016 amounting to P3, 000. Subsidiary Company
reported net income of P50, 000 and paid dividends of P10, 000 in 2016 while the parents reported net income
amounted to P100, 000 and paid dividends of P20, 000.

Determine the Consolidated Net Income Attributable to Controlling Interest/ Profit Attributable to Equity Holders of
Parent.
a. 142, 000 c. 126, 500
b. 132, 125 d. 124, 100
ANSWER:
Full-Goodwill:
Net income from own operations:
Parent (P100, 000- (10, 000 x 80%) P92, 000
Subsidiary 50, 000
P142, 000
Less: Amortization of allocated excess *6, 875
Impairment of full-goodwill (if any) 3, 000
Consolidated Net Income P132, 125
Less: NCI in Net income:
Subsidiary net income from own operation P50, 000
Less: Amortization of allocated excess 6, 875
Impairment of full-goodwill (if any) 3, 000
P40, 125
Multiply by: NCI 20% 8, 025

Consolidated Net Income attributable to controlling interest P124, 100

*Amortization of allocated excess:


Increase in inventory: 5, 000(sold in 2016) P5, 000
Increase in buildings: 15, 000 / 8 years 1, 875
Total P6, 875
36. On January 2, 2013, Pascual Corporation purchased 80% of Suazon Companys P10 par common stock for
P975, 000. On this date, the book value of Suazons net assets was P1, 000, 000. The fair value of Suazons
identifiable net assets and liabilities were the same as their carrying amounts except for plant assets (10 year life),
which were P100, 000 excess of the carrying amount. For the year ended December 31, 2013, Suazon had a
comprehensive income of P190, 000and paid cash dividends totaling P125, 000. In the December 31, 2013
consolidated statement of financial position, non-controlling interest (NCI) should be reported at:
a. 200, 000 c. 254, 750
b. 236,000 d, 182, 750

ANSWER:
NCI, 01/02/2013 (975, 000/80%% x 20% P243, 750
NCI, dividends (125, 000 x 20%) (25, 000)
NCI, adjusted net of subsidiary (190, 000 * 10, 000) x 20% 36, 000
*100, 000/10 years = 10, 000
NCI, December 31, 2013 P254, 750
37. The following are the features of the consolidated working paper for the second year after acquisition except,
a. Elimination of the total equity accounts of subsidiary against the investment account (parents
interest) and NCI.
b. Allocation of excess by adjusting the assets of the subsidiary to fair values.
c. Recognition of NCI in the CI of the parent company, adjusted for amortization and depreciation.
d. Amortization of the allocated excess.
ANSWER: C. Because the recognition of NCI should be in the CI of subsidiary and not the parent company.

38.Which of the following observations is not consistent with the use of push-down accounting?
a. The revaluation capital account is part of the subsidiarys stockholders equity.
b. Eliminating entries related to the differential are needed in the workpapers
c. No differential arises in the consolidation process
d. Revaluation Capital account is eliminated in preparing consolidated statements

Ans: b

39. The method of accounting for investment in subsidiary that is appropriate for the acquisition method of
combination is:
a. The cost method
b. The market value method
c. The equity method
d. The pooling method

Answer: C

40.If the parent company uses the cost method of accounting for a partially owned subsidiary and there are no
intercompany profit or losses eliminated for the computation of consolidated CI, Consolidated retained earnings
is equal to the balance of the parent companys:

a. Retained Earnings
b. Retained Earnings plus the parents share of the balance of the subsidiarys retained earnings
c. Retained earnings plus the parents share of the net increase in the subsidiarys retained earnings
subsequent to the date of acquisition
d. Retained earnings plus the balance of the retained earnings

Ans. C

INTERCOMPANY TRANSACTIONS- INVENTORIES

1.Bruce Company owns 80% of Lee Corp.s common stock. During October 2016, Lee sold merchandise to Bruce
for 100,000. At December 31, 2026, one-half of the merchandise remained in Bruce inventory. For 2016, gross
profit percentages were 30% for Bruce and 40% for Lee. The amount of unrealized intercompany profit in ending
inventory at December 31, 2016 that should be eliminated in consolidation is:

a. 40,000
b. 20,000
c. 16,000
d. 15,000

ANS:B
Sales to Bruce 100,000
Ending Inventory 1\2
Merchandise inventory of Bruce 12/31/2016
inclusive of profits 50,000
GP rate of Lee (the seller) 40%
Unrealized profit in ending inventory 20,000

2. The Maroons Company holds a 70% interest in the Haena Company. At the current year end Maroons holds
inventory purchased from Haena for 270,000 at a cost plus 20%. The groups consolidated statement of financial
position has been drafted without any adjustments in relation to this holding of inventory.

What adjustments should be made to the draft consolidated statement of financial position figures for non-
controlling interest and retained earnings?

Non-Controlling interest Retained Earnings


a. No change Reduce by 45,000
b. No change Reduce by 54,000
c. Reduce by 16,200 Reduce by 37,800
d. Reduce by 13,500 Reduce by 31,500

ANS:D

Ending Inventory of Maroons (parent)- upstream sales 270,000


Mark up of Subsidiary 20/120
Unrealized profit in ending inventory of Maroons (parent company) 45,000

Non Controlling interest (30% X 45,000) 13,500

Controlling interest (70% X 45,000) 31,500

3.X-Beams Inc, owned 70% of the voting common stock of Kent Corp. During 20x4, Kent made several sales of
inventory to X-Beams. The total selling price was P180,000 and the cost P100,000. At the end of the year, 20%of
the goods were still in X-Beams inventory. Kents reported income was P300,000.
What was the non-controlling interest Kents net income?
a. P90,000
b. P85,200
c. P54,000
d. P98,800
e. P86,640
ANSWER: B

Parent Subsidiary
Net Income from own operations: 210,000 90,000
X-Beams (parent) Kent
(subsidiary), 70%:30%
Unrealized Profit in EI of Parent ( 11,200) ( 4,800)
(X-Beams):
P180,000x 20% = P36,000 x
(180-100/180) = P16,000,
70%:30%
Non-controlling Interest in Kents 85,200
Net Income

4.During 2011, Pard Corp. sold goods to its 80% owned subsidiary, Seed Corp. At December 31, one half of
theses goods were included in Seeds ending inventory. Reported 2011 selling expense were P1,100,000 and
P400,000 for Pard and Seed, respectively. Pards selling expenses included P50,000 in freight out costs for
goods to Seed. What amount of selling expenses should be reported in Pards 2011 consolidated income
statement?
a. P1,500,000
b. P1,480,000
c. P1,475,000
d. P1,450,000
ANSWER: D
The requirement is to determine the amount of selling expenses to be reported in Pards 2011 consolidated
income statement. Pards selling expenses for 2011 include P50,000 in freight out costs for goods sold to Seed,
its subsidiary. This P50,000 becomes part of Seeds inventory because it is a cost directly associated with
bringing the goods to a salable condition. None of the P50,000 represents a selling expense for the consolidated
entity, and P1,450,000(P1,100,000+P400,000-P50,000) should be reported as selling expenses in consolidated
income statement.

5. On January 1, 2013, Eron Company purchased 90% of Bessy Company for P400, 000. On that day Bessy
Companys equity consisted of P100, 000 of capital stock and P300, 000 of retained earnings. Assets and
liabilities were fairly valued. In 2013 Bessy had sales of P500, 000 and cost of sales of P300, 000. One half of the
sales were to Eron. Bessys pricing policy has not changed for several years. At January 1, 2013, Erons
inventory contained P40, 000of Bessys merchandise purchased in 2012. Erons December 31, 2013, inventory
included P25, 000 of Bessys merchandise. Both companies use FIFO.

For 2013 Eron had CI from its own operations of P200, 000 and paid dividends of P80, 000. Bessys CI was P75,
000; it paid P30, 000 dividends during the year.

For 2013, consolidated CI attributable to parent is:


a. 270, 900 c. 271, 000
b. 272, 900 d. 271, 900

ANSWER:
Sales 500, 000 Gross Profit Rate(Bessy) = 200, 000/500, 000
Cost of sales 300, 000 = 40%
Gross Pofit 200,000
Net income from own operation Eron P200, 000
Adjusted net income Bessy
Net income from own operation P75, 000
Realized profit beginning inventory (40, 000 x 40%) 16, 000
Unrealized profit ending inventory (25, 000 x 40%) (10, 000) 81, 000
Consolidated Net Income 281, 000
Attributable to NCI (81, 000 X 10%) 8, 100
Attributable to Parent P272, 900

6. Parry Co. owns 80% interest in Starry Co. acquired several years ago. Starry regularly sells merchandise to its
parent at 123% of Starrys cost. Gros profit data of Parry and Starry for the year 2016 are as follows:
Parry Starry
Sales 1, 000, 000 800, 000
Cost of goods sold 800, 000 640, 000
Gross profit P200, 000 P160, 000
During 2016, Parry purchased inventory items from Starry at a transfer price of P400, 000. Parrys December 31,
2015 and 2016 inventories included goods acquired from Starry of P100, 000 and P125, 000, respectively. The
consolidated cost of goods sold of Parry and subsidiary for 2016 was:
a. 1, 024, 000 c. 1, 052, 000
b. 1, 045, 000 d. 1, 056, 000

ANSWER:
Cost of sales of: *Gross profit rate
Parry 800, 000 160, 000/ 800, 000 = 20%
Starry 640, 000
P1, 440, 000
Less: Intercompany sales (400, 000)
Realized profit in beginning inventory (20, 000)
(100, 000 x *20%)
Add: unrealized profit in ending inventory 25, 000
(125, 000 x *20%)
Consolidated cost of goods sold P1, 045, 000
7.Xyril Corp. owns an 80% interest in Erica Corp. acquired several years ago. Erica regularly sells merchandise to
its parent at 125% of Ericas cost. Gross profit data of Xyril and Erica for the year 2016 are as follows:

Xyril Erica
Sales P1,000,000 P800,000
Cost of goods sold 800,000 640,000
Gross Profit P200,000 P160,000
During 2016, Xyril purchased inventory items from Erica at a transfer price of P400,000. Xyrils December 31,
2015 and 2016 inventories included goods acquired from Erica of P100,000 abd P125,000, respectively. The
consolidated sales of Xyril Corp. and subsidiary for 2016 were:

a. P1,800,000
b. P1,425,000
c. P1,400,000
d. P1,240,000
ANS: C
Consolidated Sales
Sales of:
Xyril P1,000,000
Erica 800,000
Total P1,800,000
Less: Intercompany Sales 400,000
Consolidated Sales P1,400,000
8.Using the same information in No. 137, the unrealized profits in the year-end 2015 and 2016 inventories were:

a. P100,000 and P125,000 respectively


b. P800,000 and P100,000 respectively
c. P20,000 and P25,000 respectively
d. P16,000 and P20,000 respectively

ANS: C
Realized Profit in beginning inventory of Xyril (Upstream)
P100,000 x 20% (160/80), GP of Erica P20,000
Unrealized Profit in ending inventory of Xyril
P125,000 x 20%, GP of Erica P25,000

9. Francis Company owns 100 of the capital stock of both Gem Company and Robin Company. Francis
purchases merchandise inventory from Robin Company at 140 of Robin's cost. During 2016, merchandise that
cost Robin P40,000 was sold to Gem. Gem sold all of this merchandise to unrelated customers for 8,200 during
2016. In preparing combined financial statements for 2016 Francis' bookkeeper disregarded the common
ownership of Gem Company and Robin Comapany. By what amount was unadjusted reveneu overstated in thr in
the combined income statement for 2016 and the amount that should be eliminated from cost of good sold in the
combined income statement for 2016?

Overstated Unadjusted Reveneu Cost of Good sold to be eliminated

a. P16,000. P16000
b. 40,000. 40,000
c. 56,000. 56,000
d . 81,200. 56,000
Answer: C
Solution

Overstated Unadjusted Revenue:

When computing combined revenue the objective is to restate the accounts as it the intercompany transaction
had not- occurred. Assuming that there was no sale between Gem and Robin the correct amount of combined
revenue is overstated by the P56,000(40,000x140) intercompany reveneu recognized by Robin(the seller).

Cost of goods sold to be eliminated:

When computing combined cist of goods sold, the objective is to restate the accounts as if the intercompany
transactions had not occurred. Assuming there was no sale between Gem and Robin, the correct amount of
combined cost of goods sold would be P40,000 the original cost of the merchandise to Robin(the seller).
However, Robin, recognized P40,000 for CGS and Gem recognized P56,000(40,000x140) for a total of
P96,000. Thus, 56,000(96,000-40,000) should be eliminated from CGS in he combined income statement for
2016.

Incidentally, the entry for th above items eliminating the intercompany transaction s would be:

Sales 56,000

CGS(or purchases). 56,000

10. Ryan Retail Company sells goods for cash, on normal credit (2/10, n/30). However, on July 1, 2014, the
company sold a used computer for P22,000; the inventory carrying value was P4,40O. The company collected
P2,000 cash and agreed to let the customer make payments on the P20,000 whenever possible during the next
12 months. The company management stated that it had no reliable basis for estimating the probability of default.
The following additional data are available: (a) collections on the instalment receivable during 2014 were P3,000
and during 2015 were P2,000, and (b) on December 1, 2015, Ryan Retail repossessed the computer (estimated
net realizable value P7,000).

Determine the realized gross profit on lnstalment sales for the year 2014.

A. P 1,600
B. P 4,000
C. P 2,400
D. P 5,600
Answer: B
Solution

Total collections made in 2014:

Downpayment P2,000
Installment 3,000 P5,000
Gross profit rate (22,000 - 4,400/ 22,000) x80
Gross profit realized: P4,000
11. Peter Corporation owns 70% of John Conpany's common stock, acquired January 1, 2015. Goodwill from the
Investment is not amortized. John regularly sells merchandise to Peter at 150 percent of John's cost. Peter's
December 31,2015 and 2016 inventories include goods purchased intercompany of P112,500 and P33,000,
respectively. The separate incomes (do not include investment income) of Peter and John for 2016 are
summarized as follows:
Peter John
Sales 1,200,000 800,000
Cost of Sales (600,000) (500,000)
Other Expenses (400,000) (100,000)
Separate Income P200,000 P200,000

Total consolidated income should be allocated to net income to retained earnings and minority interest in the
amounts of:
a. 358,550 and 67,950, respectively
b. 378,550 and 60,000, respectively
c. 366,500 and 60,000, respectively
d. 366,500 and 67,950, respectively

Answer: A.
Separate Income (200,000 + 200,000) 400,000
Add: realized mark-up on beginning inventory (112,500 x 1/3) 37,500
Less: unrealized mark up on ending inventory (33,000 x 1/3) (11,000)
Consolidated net income 426,500
Minority interest income [(200,000+37,500-11,000)x 30%] (67,950)
Net Income to Retained Earnings 358,550

12. Selected information from the separate and consolidated balance sheets and income statements of Pass Inc.,
and its subsidiary, Success Co., as of Dec. 31,2014, and for the year then ended is as follows:
Pass Inc Success co. Consolidated
Balance Sheet accounts
Accounts Receivable 52,000 38,000 78,000
Inventory 60,000 50,000 104,000
Income Statement accounts
Revenues 400,000 280,000 616,000
Cost of Goods Sold 300,000 220,000 462,000
Gross Profit. 100,000 60,000 154,000

Additional Information:
During 2014, Pass sold goods to Success at the same mark-up on cost that Pass uses for all sales. How
much is the correct cost of the merchandise acquired by Success from Pass?
a. 64,000
b. 48,000
c. 24,000
d. 18,000

Answer: B.
Intercompany Sale
Total Sales (400,000+280,000) 680,000
Consolidated Sales (616,000) 64,000
Less: Mark-up (64,000 x 1/4) (16,000)
Cost of merchandise transferred 48,000

13. Evan Corporation owns 75% of the outstanding stock of Jewel Company, acquired at book value during 2012.
During 2015 Jewel Company sold merchandise to Evan for P 150,000 at a gross profit rate of 40%. At the end of
2015, Evan still owed P 75,000 to Jewel for the merchandise. On December 31, 2015 inventory amounting to P
60,000 from the intercompany purchases remained at Evan. The amount of unrealized profit of Evan that should
be eliminated in consolidation is:

a. P 24,000 b. P 36,000 c. P 30,000 d. P 6,000

Answer: a. P 24,000

Solution:
Ending Inventory, Evan P 60,000
Multiplied by Gross Profit Rate 40%
Unrealized Profit, Evan P 24,000

14. Mong Corporation purchased 90% of the stock of Go Company on January 1, 2014. On that date, the book
value of Go's net assets approximated fair value. As a result of the purchase, Mong recognized P 50,000 of
goodwill.

During 2014, Go sold inventory to Mong. On December 31, 2014, Go had unrealized profits on its books of P
8,000. By December 31, 2015, all of the inventory left on Mong's books had been sold to outside parties. During
2015, Mong sold inventory to Go and had P 12,000 of unrealized profits left on its books at the end of 2015. For
2015, Mong reported operating income of P 450,000, and Go reported CI of P 310,000. What is the consolidated
CI attributable to parent for 2015?

a. P 736,200 b. P 717,000 c. P 724,200 d. P 731,800

Answer: c. P 724,200

Solution:
Net Income from own operation- Mong P 450,000
Unrealized Profit in Ending Inventory- downstream ( 12,000)
Realized Separate Net Income- Mong P 438,000
Mong's share of Go's Adjusted Net Income:
Net Income P 310,000
Realized Profit in Beg. Inventory 8,000 318,000
Minority Interest in Subsidiary
( P 318,000 x 10% ) ( 31,800)
Attributable to Parent P 724,200

15. During the fiscal year ended May 31, 2003, Swope Company, the 80%-owned subsidiary of Pone
Corporation, sold merchandise to its parent company at billed prices totaling P360,000, representing a 220%
markup on Swope's cost. On May 31, 2003, Pone's inventories included merchandise totaling P54,000 purchased
from Swopea P12,000 increase over the comparable June 1, 2002, amount.

The total amount to be eliminated for consolidated costs of goods sold of Pone Corporation and subsidiary for the
fiscal year ended May 31, 2003, is:

a. P58,000
b. P60,000
c. P300,000
d. Some other amount

Ans: D
Feedback: (P368,000 x 5/6) + (P348,000 x 1/6) = P358,000

16. Several years ago Ruby Company acquired 70% of Riza company at book value. Relevant data for 2013 are
as follows:

Ruby Riza
CI from its own operations 400,000 250,000
Dividends declared and paid in 2013 270,000 110,000
Merchandise from intercompany sales
in Rubys inventory:
Jan. 1, 2013 40,000
Dec. 31, 2013 70,000
Gross profit rate on sales:
2012 70% 40%
2013 75% 30%

Consolidated CI for 2013 is:


a. 645,000
b. 625,000
c. 517,000
d. 571,000
Ans. A

Net income from own operation Pip P 400,000


Adjusted net income of Sol:
Net income P 250,000
Realized profit in beginning inventory-
Upstream (P40,000 x 40%) 16,000
Unrealized profit in ending inventory-
Upstream (P70,000 x 30%) ( 21,000) 245,000
Consolidated net income - 2008 P 645,000

17. Sailing Company owns 100% of the capital stock of both Twill Corp. and Webb Corp. Twill purchases
merchandise inventory from Webb at 140% of Webbs cost. During 2016, merchandise that cost Webb P40, 000
was sold to Twill. Twill sold all of this merchandise to unrelated customers for P81, 200 during 2016. In preparing
combined financial statements for 2016, Sailings bookkeeper disregarded the common ownership of Twill and
Webb. By what amount was unadjusted revenue overstated in the combined income statements for 2016 and the
amount that should be eliminated from cost of goods sold in the combined income statement for 2016?

Overstated Unadjusted Revenue Cost of goods sold to be eliminated


a. P16, 000 P16, 000
b. P40, 000 P40, 000
c. P56, 000 P56, 000
d. P81, 200 P56, 000

Ans. : C
Solution:

Overstated Unadjusted Revenue:

When computing combined revenue, the objective is to restate the accounts as if the intercompany transaction
had not occurred. Assuming that there was no sale between Twill and Webb, the correct amount of combined
revenue would be P81, 200 sold to unrelated customers. Thus, unadjusted revenue is overstated by the P56, 000
(P40, 000 x 140%) intercompany revenue recognized by Webb (the seller).

Cost of goods sold:

When computing combined cost of goods sold, the objective is to restate the accounts as if the intercompany
transactions had not occurred. Assuming there was no sale between Twill and Webb, the correct amount of
combined cost of goods sold would be P40, 000 the original cost of merchandise to Webb (the seller). However,
Webb recognized P40, 000 for CGS and Twill recognized P56, 000 (P40, 000 x 140%) for a total of P96, 000.
Thus, P56, 000 (P96, 000 P40, 000) should be eliminated from CGS in the combined income statement for
2016.

Incidentally, the entry for the above items eliminating the intercompany transactions would be:

Sales56, 000
Cost of goods sold (Purchase) 56, 000
18.On January 1, 2016 , Joy Company purchased 75% of the outstanding stock of Mae Company at book value.
During 2016 Mae sold inventory items costing P50,000 to Joy for P75,000.Joy resold 60% of this inventory to
outsiders during the year for P100,000. For the year 2016 Joy had CI from its own operations of P200,000 and
paid dividends of P120,000.Mae s CI for the year was P110,000, it paid P40,000 in dividends. What is the
consolidated CI attributable to parent for 2016?

a. P273,000
b. P279,000
c. P300,000
d. P275,000
Ans.D
Net income from own operation Joy P 200,000
Mae s adjusted net income:
Net income P110,000
Unrealized profit in ending inventory-
Upstream (P25,000 x 40%) ( 10,000) 100,000
Consolidated net income P 300,000
MINIS (P100,000 x 25%) (25.000)
Attributable to parent P 275,000

19. On January 1,2016, Kiel Company purchased 90% of Ron Company for P400,000. On that day Ron
Companys equity consisted of P100,000 of capital stock and P300,000 of retained earnings,. Assets and
liabilities were fairly valued.
In 2016 Ron had sales of P500,000 and cost of sales of P300,000.One half of the sales were to Kiel. Rons
pricing policy has not changed for several years.
At January 1.2016, Kiels inventory contained P40,000 of Rons merchandise purchased in 2015.Kiels
Dec.31,2016 inventory included P25,000of Rons merchandise. Both companies use FIFO.
For 2016 Kiel had CI from its own operations of P200,000 and paid dividends of P80,000.Rons CI for
2016 was P75,000. It paid P30,000 dividends during the year.

For 2016.consolidated CI attributable to parent is:

a. P270,900
b. P272,900
c. P273,000
d. P271.900

Ans. B
Gross profit rate of Sit (P200,000 / P500,000) 40%
Net income from own operations Kiel P 200,000
Adjusted net income of Ron :
Net income P 75,000
Realized profit in beginning inventory-
Upstream (P40,000 x 40%) 16,000
Unrealized profit in ending inventory-
Upstream (P25,000 x 40%) ( 10,000) 81,000
Consolidated net income P 281,000
MINIS (P281,000 x 10%) ( 8,100)
Attributable to parent P 272,900

20.GLYSDI Corporation purchased inventory from GBY Corporation for P 120,000 on September 20, 20x4 and
resold 80 percent of the inventory to unaffiliated Companies prior to December 31,20x4, for P140,000. Dresser
produced the inventory sold to GLSDI Corp. for P75,000 owns 70 percent of GBYs voting common stock. The
companies had no other transactions during 20x4.

What amount of Consolidated net income will be assigned to non controlling interest for 20x4?
A. P20,000
B. P30,800
C. P44,000
D. P45,000
E. P69,200

ANSWER: E
Consolidated sales P 140,000
Cost of goods sold (60,000)
Consolidated net income P 80,000
Income to GLYSDIS Non controlling interest:
Sales P 120,000
Reported cost of sales 175,000
Report income P 45,000
Portion realized x .80
Realized net income P 36,000
Portion to non controlling interest x .30
Income to non controlling interest (10,800)
Income to controlling interest P 69,200

21. Kristel Inc. acquired 100% of Gaspard Farms on January 5, 20x3. During 20x3, Kristel sold Gaspard Farms
for P625,000 goods which had cost P425,000. Gaspard Farms still owned 12% of the goods at the end of the
year. In 20x4, Kristel sold goods with a cost of P800000 to Gaspard Farms for P1000000 and Gaspard Farms still
owned 10% of the goods at year end. For 20x4, cost of goods sold was P1,200,000 for Gaspard Farms and
P5,400,000 for Kristel. What was consolidated cost of goods sold for 20x4?
A. P6,600,000
B. P5,596,000
C. P6,596,000
D. P5,625,000
E. P5,620,000

ANSWER: B

Cost of sales
K Company (p) P 5,400,000
G Company (s) 1,200,000
Total 6,600,000
Less: Intercompany sales 1000,000
Realized profit [P625,000x12%=75,000x(625-425)/625] 24,000
Add: Unrealized profit [1000,000x10%=100,000x (1000-800)/1000] _20,000
Consolidated 5,596,000

22. Blue Company owns 80 percent of the common stock of White Corporation. During the year, Blue reported
sales of $1,000,000, and White reported sales of $500,000, including sales to Blue of $80,000. The amount of
sales that should be reported in the consolidated income statement for the year is:

a. $ 500,000.
b. $ 1,300,000.
c. $ 1,420,000.
d. $ 1,500,000.
Answer: c

Blue Sales $ 1,000,000


White Sales $ 500,000
Less: Intercompany Sales $ 80,000
Consolidated Sales $ 1,420,000

23. Pepe Corporation owns an 80% interest in Sisa Company, and t December 31,2012, Pepe's investment in
Sisa under the cost method was equal to 80% of Sisa's stockholders equity. During 2013, Sisa sells merchandise
to Pepe for P 100,000, at a gross profit to Sisa of P 20,000. At December 31, 2013 half of this merchandise is
included to Pepe's inventory. Separate incomes for Pepe and Sisa for 2013 are summarized as follows:

Pepe Sisa
Sales P 500,000 P 300,000
Cost of sales (250,000) (200,000)
Operating Expenses (125,000) (40,000)
CI from own operations P 125,000 P 60,000

In the cosolidated statement of CI for 2013, NCI in CI of subsidiary is:

a. P 12,000
b. P 11,000
c. P 10,000
d. P 14,000

Answer: c

Net income-Sisa P 60,000


Unrealized profit in ending inventory
upstream Adjusted net income Sisa (10,000)
P 50,000
NCI proportionate share 20%
NCI in net income of subsidiary P 10,000

24. Sand Company owns 80% of Wich Corp.s common stock. During October 20x6, Wich sold merchandise to
Sand for P100,000. At December 31, 20x6, one-half of the merchandise remained in Sand inventory. For 20x6,
gross profit percentages were 30% for Sand and 40% for Wich. The amount of unrealized intercompany profit in
ending inventory at December 31, 20x6 that should be eliminated in consolidation is:
a. P40,000 c. P16,000
b. P20,000 d. P15,000

Ans: b

Sales to Sand P100,000


x: Ending Inventory 1/2
Merchandise Inventory of Sand, 12/31/2016 P 50,000
inclusive of profits
x: GP rate of Wich (seller) 40%
Unrealized profit in ending inventory P 20,000
25. On January 1, 2014, Diamond Co. purchased 80% of the outstanding shares of Star Co. by paying
P170,000, the Star Co.s common stock and retained earnings on this date amounted to P75,000 and
P115,000 respectively. Also on this date, an equipment is undervalued by P10,000 with a remaining life of
5 years.
On January 1, 2016, Star Co. had P75,000 of capital stock and P300,000 of retained earnings. Also on
the same date. Diamond Co. had P500,000 of capital stock and P350,000 of retained earnings.
During the year, Diamond Co. sol merchandise to Star Co. for P30,000 and in turn, purchased P20,000
from Star Co. Inter-company sales of merchandise were made at the following gross profit rates:
Sales made by parent. 25% based on cost
Sales made by subsidiary 20% based on sales
On December 31,2016, 30% of all inter-company sales remain in the ending inventory of the purchasing
affiliate.
The beginning inventory of Diamond Co. includes P1,250 worth of merchandise acquired from Star Co.
on which Star Co. reported a profit of P500. While, the beginning inventory of Star Co. also includes
P1,500 of merchandise acquired from Diamond Co. at 35% mark-up.
Using the cost method the following selected results of operations for 2016 were as follows:

Diamond Co. Star Co.


Dividends paid. P30,000 P5,000
Net income from own operations. P50,000 P15,000
Add: Dividend income 4,000
Net income.. P54,000 P15,000
The balance of Investment in Star Co. and the Non-controlling Interest in Net Income as of December 31,
2016, should be:
a. P170,000; P2,360
b. P136,000; P2,360
c. P136,000; P2,460
d. P170,000; P2,460

ANSWER: (d)

Investment in Star Co. = P170,000 since cost model method is in effect.

Net income of Subsidiary. P 15,000


Add: Realized profit in beg. Inventory of Parent
(upstream). 500
Less: Unrealized profit in ending inventory of parent (upstream)
(P20,000 x 30% = P6,000 x 25/125). 1,200
Less: Amortization of allocated excess (P8,000/80%)/5 years 2,000
12,300
Multiplied by: Non-controlling interest 20%
Non-controlling interest in net income P 2,460

26. Large Corp. acquired 75% interest in Small Corp. in 2015. For the year ended December 31, 2015
and 2016, Small Corp. reported net income of P80,000 and P90,000, respectively. During 2015, Small
Corp. sol merchandise to Large Corp. for P10,000 at a profit of P2,000. The merchandise was later resold
to Large Corp. to outsider for P15,000 during 2016. For consolidation purposes, what is the non-
controlling interests share of Smalls net income for 2015 and 2016, respectively?

2015 2016
a. P20,000 P22,500
b. P19,500 P22,000
c. P20,500 P23,000
d. P19,000 P22,000

ANSWER: (b)

2015 2016
Smalls net income from own operation* P80,000 P90,000
Unrealized profit in ending inventory
of Small 2015.. (2,000) (2,000)
P78,000 P88,000
Multiplied by: Non-controlling interest.... 25% 25%
Non-controlling interest in Net Income.. P19,500 P22,000
27. Colton Company acquired 80 percent ownership of Mota Company's voting shares on January 1, 2008, at
underlying book value. The fair value of the noncontrolling interest on that date was equal to 20 percent of the
book value of Mota Company. During 2008, Colton purchased inventory for $30,000 and sold the full amount to
Mota Company for $50,000. On December 31, 2008, Mota's ending inventory included $10,000 of items
purchased from Colton. Also in 2008, Mota purchased inventory for $80,000 and sold the units to Colton for
$100,000. Colton included $30,000 of its purchase from Mota in ending inventory on December 31, 2008.
Summary income statement data for the two companies revealed the following:

What amount of income will be assigned to the noncontrolling shareholders in the 2008 consolidated income
statement?

a. $ 6,200

b. $ 5,400

c. $ 5,800

d. None of the above

ANSWER: B
Reported net income of Mota Company $ 33,000

Unrealized profit on sale to Colton Company

$20,000 x ($30,000 / $100,000) ( 6,000)

Realized net income $ 27,000

Noncontrolling interest's share x 0.20

Income assigned to noncontrolling interest $ 5,400

28. Hunter Company and Moss Company both produce and purchase fabric for resale each period and frequently
sell to each other. Since Hunter Company holds 80 percent ownership of Moss Company, Hunter's controller
compiled the following information with regard to intercompany transactions between the two companies in 2007
and 2008:

Compute the amount of cost of goods sold to be reported in the consolidated income statement for 2008.

a. $184,250

b. $202,250

c. $217,000

d. None of the above

ANSWER: A
29. Czarina Corporation acquired an 80% interest in Grace Corporation in 2015. For the year ended December
31, 2015 and 2016, Grace reported net income of P140,000 and P150,000, respectively. During 2015, Grace sold
merchandise to Czarina Corp. for P20,000 at a profit of P4,000. The merchandise was later resold by Czarina
Corp. to outsider for P30,000 during 2016. For consolidation purposes, what is the non-controlling interests share
of Graces net income for 2015 and 2016, respectively?
a. P27,200; P30,800
b. P32,000 ; P36,000
c. P32,000 ; P30,800
d. P27,200 ; P36,000
Answer: A

2015 2016

Graces net income from own operation P140,000 P150,000

Unrealized profit in ending inventory ( 4,000 ) -

Realized profit in beginning inventory - 4,000

P136,000 P154,000

Multiplied by 20% 20%

Non-controlling interest in Net income P27,200 P30,800

30. On January 1, 2016, CG Company purchased 80% of the stock outstanding of JJ Company at a price that
included P25,000 of excess due to undervaluation of land.

On December 31, 2016, CG Company had in its inventories P22,000 of merchandise purchased from JJ
Company at 125% of cost. On the same date, JJ Companys inventories included P15,000 of merchandise which
were purchased from CG Company at 120% of cost. The NCI reported on the consolidated statement of financial
position at December 31,2016 was P82,420. For 2016, CG Company reported income of P215,600 computed
under the equity method. NCI net income was P26,180.

The net assets of JJ Company as at December 31,2016 is:


a. P281,200
b. P416,500
c. P270,500
d. P831,300
Answer: B

NCI on December 31,2016 consolidated financial position P82,420

NCI in unrealized profit on merchandise sold to CG

(P22,000 x 25/125) x 20% 880

NCI in net assets of JJ as of December 31,2016 P83,300

Divided by: 20%

Net assets of JJ on December 31, 2016 P416,500

On January 1,2015, Blue Company purchased 80% of the outstanding shares of Grey Company by paying
P340,000, the Grey Companys common stock and retained earnings on this date amounted to P150,000 and
P230,000 respectively. Also on this date, an equipment is undervalued by P20,000 with a remaining life of 10
years.

On January 1,2017, Grey Company had P150,000 of capital stock and P300,000 of retained earnings. Also on the
same date, Blue Company had P1,000,000 of capital stock and P700,000 of retained earnings.

During the year, Blue Company sold merchandise to Grey for P60,000 and in turn, purchased P40,000 from Grey
Company. Inter-company sales of merchandise were made at the following gross profit rates:
Sales made by parent 25% based on cost
Sales made by subsidiary 20% based on sales
On December 31,2017, 30% of all inter-company sales remain in the ending inventory of the purchasing affiliate.

The beginning inventory of Blue Company includes P2,500 worth of merchandise acquired from Grey Company
on which Grey Company reported a profit of P1,000. While the beginning inventory of Grey also includes P3,000
of merchandise acquired from Blue Company at 35% mark-up.

Using the cost method the following selected results of operations for 2017 were as follows:
Blue Company Grey Company
Dividends paid
Net Income from own operations
Add: Dividend Income
Net income

32. The stockholders equity of subsidiary on December 31,2017 should be:


a. P450,000 c. P481,600
b. P470,000 d. P484,000
33.} The consolidated stockholders equity on December 31,2017 using proportionate basis (or partial goodwill
approach):
a. P1,911,000 b. P1,905,920
c. P1,906,000 d. P1,740,000

Solutions:
1} ANS: B
Non-controlling interests (in net assets):
Common stock of Subsidiary,12/31/2017
Retained earnings of Subsidiary, 12/31/2017:
Retained earnings of Subsidiary 1/1/2017 P300,000
Add: Net income of Subsidiary 30,000
Less: Dividends of Subsidiary 10,000 320,000
Book value of stockholders equity of
Subsidiary, 12/31/2017 P470,000
Adjustments to reflect fair value of net assets:
Increase in equipment,1/1/2015 20,000
Accumulated amortization (P2,000 x 3 years) ( 6,000)
Fair value of Net Assets/SHE of Subsidiary, 12/31/2017 P484,000
Less: UPEI of Blue2017 2,400
Realized Stockholders Equity Of Subsidiary, 12/31/2017 P481,600
Multiplied by: Non-controlling interest 20%
Non-controlling interest (in net assets) partial goodwill P 96,320
Add: Non-controlling interest on full goodwill
(P25,000 P20,000) 5,000
Non-controlling interest (in net assets) full goodwill P101,320
2} ANS: B
Consolidated Stockholders Equity, 12/31/2017:
Controlling interest/ Parents Interest/ Parents Portion/
Equity Holders of Blue in StockholdersEquity,12/31/2017:
Common stock of Parent P1,000,000
Retained earnings of Parent (equity method),
12/31/2017 809,680
Controlling Interest / Parents Stockholders Equity,
12/31/2017 P1,809,680
Non-controlling interest, 12/31/2017 (partial goodwill) 96,320
Consolidated Stockholders Equity, 12/31/2017 P1,906,000

34. Nokia Co. owns 75% of Samsung Co.s common stock. For the year 2016, income statement of Nokia Co.and
Samsung Co. Is as follows:

Nokia Co. Samsung Co.


Sales....................................................................................P 4,500,00 P1,750,000
Cost of sales...................................................................... 2,800,00 850,000

Gross Profit....................................................................... P 1,700,000 P 900,000


Operating Expense.............................................................. 980,000 645,000

Net Income.........................................................................P 720,000 P 255,000

Intercompany sales for 2016 are upsream and total P650,000. Nokias December 31,2015 and December
31,2016 inventories contain unrealized profits of P98,000 and P120,000,respectively. The consolidated cost of
sales for 2016:

A.P2,978,000
B.P3,672,000
C.P3,022,000
D.P2,798,000

Answer: C

Combined cost of sales(P2,800,000+P850,000)......................................................................P3,650000


Less: Intercompany purchases.................................................................................................. 650,000
Add: Unrealized profit from ending inventory ........................................................................ 120,000
Less:Unrealized profit in beginning inventory......................................................................... 98,000

Consolidated cost of sales.................................................................................................... P3,022,000 (C)

35. One Co. acquired a 60% interest in Two Co.in 2015. For the year ended December 31,2015 and 2016, Two
Co.reported net income of P560,000 and P590,000,respectively. During 2015, Two Co.sold merchandise to One
Co.for P50,000 at a profit of P10,000. The merchandise was later resold by One Co. To outsider for P60,000
during 2016. For consolidation purposes, what is the non-controlling interests share of Twos net income for
2016?

A.P240,000
B.P236,000
C.P232,000
D.P323,000

Answer: A

Twos net income from own operation ...............................................P590,000

P600,000
Multiplied by NCI percentage.......................................................... 40%

Non-controlling interest in Net Income....................................... P 240,000 (A)

*Intercompany: Plant Assets


35. On January 1,2016, Red Inc.sold equipment with a four-year remaining useful life and a book value of
P350,000 to its 65%-owned subsidiary for a price of P530,000. In consolidation working papers for the year ended
December 31,2016, the elimination entry concerning this transaction will include:

A.A debit to equipment for P180,000


B.A credit to depreciation expense for P180,000
C.A debit to cash for P350,000
D.A debit to gain on equipment for P180,000

Answer: D
100% unrealized gain and restore the original book value ,date of sale 1/1/16
Gain on sale ........................................................P180,000
Equipment....................................... P180,000
The entry made in the books of subsidiary on the date of sale:
Equipment..........................................................P 530,000
Cash................................................ P530,000
The entry made in the books of parent on the date of sale:
Cash...................................................................P530,000
Equipment..................................... P350,000
Gain on Sale.................................. 180,000

From consolidated point of view, there should be no gain. Therefore, to eliminate the gain should be debited and
equipment should be reduce accordingly.

For depreciation:
Accumulated depreciation.................................P45,000
Depreciation expense (P180,000/4 years) P45,000

36. Bruce Company owns 80% of Lee Co.s common stock. During October 2016, Lee sold merchandise to Bruce
for 100, 000. At December 31, 2016, one-half of the merchandise remained in Bruce inventory. For 2016, gross
profit percentages were 30% for Bruce and 40% for Lee. The amount of unrealized profit in ending inventory at
December 31, 2016 that should be eliminated in consolidation is:
a. 40, 000 c. 16, 000
b. 20, 000 d. 15, 000
ANSWER:
Sales to Bruce P100, 000
Multiply by: Ending inventory 1/2
Merchandise inventory of Bruce, 12/31/2016 inclusive of profits 50, 000
Multiply by: GP rate of Lee 40%
Unrealized Profit in ending inventory P20, 000
37. Consolidated net income for a parent and its 80 percent owned subsidiary should be computed by eliminating:

a. all unrealized profit in downstream intercompany inventory sales, and unrealized profit in upstream
intercompany inventory sales made during the current year.

b. all unrealized profit in downstream intercompany inventory sales, and the noncontrolling interest's share of
unrealized profit in upstream inventory sales made during the current year.

c. the controlling interest's share of unrealized profit in downstream intercompany sales, and the controlling
interest's share of unrealized profit in upstream sales made during the current year.

d. all unrealized profit in downstream intercompany sales, and the noncontrolling interest's share of unrealized
profit in upstream sales made during the current year.

Answer: A

38.The material sale of inventory items by a parent company to an affiliated company:


a. Enters the consolidated revenue computation only if the transfer was the result of arms length
bargaining.
b. Affects consolidated net income under a periodic inventory system but not under a perpetual inventory
system.
c. Does not result in consolidated income until the merchandise is sold to outside entities.
d. Does not require a working paper adjustment if the merchandise was transferred at cost.
ANS: C

39.Consolidated net income for a parent and its 80 percent owned subsidiary should be computed by eliminating:

a. all unrealized profit in downstream intercompany inventory sales, and unrealized profit in upstream
intercompany inventory sales made during the current year.
b. all unrealized profit in downstream intercompany inventory sales, and the noncontrolling interest's share
of unrealized profit in upstream inventory sales made during the current year.
c. the controlling interest's share of unrealized profit in downstream intercompany sales, and the controlling
interest's share of unrealized profit in upstream sales made during the current year.
d. all unrealized profit in downstream intercompany sales, and the noncontrolling interest's share of
unrealized profit in upstream sales made during the current year.

ANSWER: A

40.A subsidiary made sales of inventory to its parent at a profit this year. The parent, in turn, sold all but 20
percent of the inventory to unaffiliated companies, recognizing a profit. The amount that should be reported as
cost of goods sold in the consolidated income statement prepared for the year should be:

A. the amount reported as intercompany sales by the subsidiary.


B. the amount reported as intercompany sales by the subsidiary minus unrealized profit in the ending inventory of
the parent.
C. the amount reported as cost of goods sold by the parent minus unrealized profit in the ending inventory of the
parent.
D. the amount reported as cost of goods sold by the parent.

ANSWER: B
INTERCOMPANY TRANSACTIONS- PLANT ASSETS

1. On January 1, 2016, Jan Co. purchased 90% equity of Jo Co.. On January 3, 2016, Jo sold equipment
(with original cost of P750,000 and carrying cost of P375,000) to Jan for P540,000. The equipment have
a remaining life of three (3) years and was depreciated using the straight line method by both companies.
In Jan consolidated balance sheet as of December 31, 2016, the cost, accumulated depreciation and
book value should be reported at:

Cost Accumulated Depreciation Net Book Value

a. P750,000 P500,000 P375,000


b. 375,000 375,000 -0-
c. 750,000 750,000 -0-
d. 750,000 500,000 250,000

ANS: D

Cost P750,000
Less: Accumulated Depreciation, 12/31/16
Accumulated Depreciation, 1/1/16 P375,000
Depreciation Expense-2016:
(P750,000-P375,000)/3years 125,000 500,000
Net Book Value, 12/31/16 P250,000

2. Kestrel Company acquired an 80% interest in Reptile Corporation on January 1, 2004. On


January 1, 2005, Reptile sold a building with a book value of $50,000 to Kestrel for $80,000.
The building had a remaining useful life of ten years and no salvage value. The separate
balance sheets of Kestrel and Reptile on December 31, 2005 included the following balances:

Kestrel Reptile
Buildings $ 400,000 $ 250,000
Accumulated Depreciation - 120,000 75,000
Buildings

The consolidated amounts for Buildings and Accumulated Depreciation - Buildings that
appeared, respectively, on the balance sheet at December 31, 2005, were

a. $620,000 and $192,000.


b. $620,000 and $195,000.
c. $650,000 and $192,000.
d. $650,000 and $195,000.
ANSWER:
Combined building amounts $ 650,000
Less: Intercompany gain ( 30,000 )
Consolidated building amounts $ 620,000

Combined Accumulated Depreciation $ 195,000


Less: Piecemeal recognition of gain
( 3,000 )
Consolidated accumulated depreciation
$ 192,000

2. Pied Imperial-Pigeon Corporation acquired a 90% interest in Offshore Corporation in 2003


when Offshore book values were equivalent to fair values. Offshore sold equipment with a
book value of $80,000 to Pied Imperial-Pigeon for $130,000 on January 1, 2005. Pied
Imperial-Pigeon is fully depreciating the equipment over a 4-year period by using the straight-
line method. Offshore reported net income for 2005 was $320,000. Pied Imperial-Pigeons
2005 net income from Offshore was

a. $249,250.
b. $250,500.
c. $254,250.
d. $288,000.

ANSWER:
Pied Imperial-Pigeons share of Rogers income =
($320,000 x 90%) = $ 288,000
Less: Profit on intercompany sale ($130,000 -
$80,000) x 90% = ( 45,000
Add: Piecemeal recognition of deferred profit
($50,000/4 years) x 90% =
11,250
Income from Offshore $ 254,250
4.The Roel Company acquired equipment January 1, 2013 at accost of 800,000, depreciating it over 8 years with
a nil residual value. On January 1, 2016. The Muldon Company acquired 100% of Roel and estimated the fair
value of the equipment at 460,000, with a remaining life of 5 years. This fair value was not incorporated into
Roels book and the depreciation expense continued to be calculated by reference to original cost.

What adjustment should be made to the depreciation expense for the year and the statement of financial position
carrying amount in preparing the consolidated financial statements for the year ended December 31, 2017?

Depreciation Expense Carrying Amount


a. Increase by 8,000 Increase by 24,000
b. Increase by 8,000 Decrease by 24,000
c. Decrease by 8,000 Increase by 24,000
d. Decrease by 8,000 Decrease by 24,000
ANS:D
Fair value adjustments under PFRS 3 par.36 not reflected in the books must be adjusted for on consolidation.

Annual depreciation expense:


Roels depreciation: (800,000-0)/8 years 100,000
Muldons depreciation (460,000/5) 92,000
8,000

Net carrying/book value, 12/31/2017:


Roels book value (800,000-(800,000/8 years X 5yrs) 300,000
Muldons book value (note) (460,000-(460,000/5 X 3years) 276,000
Decrease 24,000

5.. On January 1, 2016. Poe Corporation sold machine for 900,000 to Saxe Corp. its wholly owned subsidiary.
Poe paid 1,100,000 for this machine, which had accumulated depreciation of 250,000 . Poe estimated of 100,000
salvage value and depreciated the machine on the straight line method over 20 years, a policy which Saxe
continued, In Poes December 31, 2016, consolidated balance sheet, this machine should be included in cost and
accumulated depreciation as:

COST ACCUMULATED DEPRECIATION


a. 1,100,000 300,000
b. 1,100,000 290,000
c. 900,000 40,000
d. 850,000 42,500

ANS: A

When preparing consolidated financial statements, the objective is to restate the accounts as if the intercompany
transactions had not occurred. Therefore, the 2016 gain on sale of machine of 50,000 (900,000-(1,100,000-
250,000) must be eliminated, since the consolidated entity has not realized any gain. In effect, the machine must
be reflected on the consolidated balance sheet at 1/1/2016 at Poes cost of 1,100,000 and accumulated
depreciation of 250,000 instead of at a new cost of 900,000.For consolidated statement purposes, 2016
depreciation is based on the original amounts (1,100,000-100,000) X 1/20= 50,000).

Therefore, in the 12/31/2016 consolidated balance sheet, the machine is shown at a cost of 1,100,000 less
accumulated depreciation of 300,000 (250,000+50,000).

6. Peregrine Corporation acquired a 90% interest in Cliff Corporation in 2004 at a time when Cliffs book values
and fair values were equal to one another. On January 1, 2005, Cliff sold a truck with a P45,000 book value to
Peregrine for P90,000. Peregrine is depreciating the truck over 10 years using the straight-line method. Separate
incomes for Peregrine and Cliff for 2005 were as follows:

Sales
Peregrine Cliff
Sales P 1,800,000 P 1,050,000
Gain on sale of truck 45,000
Cost of Goods Sold (750,000) (285,000)
Depreciation expense (450,000) (135,000)
Other expense (180,000) (450,000)
Separate incomes 420,000 225,000

Peregrines investment income from Cliff for 2005 was:

a. P 161,550.
b. P162,000.
c. P166,050.
d. P202,500.

ANSWER: C

Cliff reported income P225,000


Less: Intercompany gain on truck (45,000)
Plus: Piecmeal recognition of gain=P45,000/10years 4,500
Cliff adjusted income 184,500
Majority Percentage 90%
Income from Cliff P166,050

7.Falcon Corporation sold equipment to its 80%-owned subsidiary, Rodent Corp., on January 1, 2005. Falcon
sold the equipment for P110,000 when its book value was P85,000 and it had a 5-year remaining useful life with
no expected salvage value. Separate balance sheets for Falcon and Rodent included the following equipment and
accumulated depreciation amounts on December 31, 2005:

Falcon Rodent
Equipment P 750,000 P300,000
Less: Accumulated Depreciation (200,000) (50,000)
Equipment-net P550,000 P250,000

Consolidated amount for equipment and accumulated depreciation at December 31, 2005 were respectively:
a. P1,025,000 and P245,000
b. P1,025,000 and P250,000
c. P1,025,000 and P245,000
d. P1,050,000 and P250,000

ANSWER: A

Combined equipments amount P1,050,000


Less: Gain on sale (25,000)
Consolidated equipment balance P1,025,000

Combined Accumulated depreciation P250,000


Less: Depreciation on gain (5,000)
Consolidated Accumulated Depreciation P245,000

8. Monica purchased equipment from Its 85% owned subsidiary, Eloisa Company for P150,000 on january 2015
and the equipment and Its accumulated depreciation were carried on the books Eloisa at 200,00 and P100,000,
respectively. Monica estimates a remaining life of the equipment to be 5 years, at which time no salvage value ls
expected to exist. Straight-line depreciation is used. Eloisa reported a net income for 2015 of P150,000.

What portion of the depreciation recorded by Monica must be eliminated for consolidation purpose?

A. 20% of the gain on sale C. 33-1/3% of the gain on sale

B. 40% of the gain on sale D. 66-2/3% of the gain on sale


Answer: A
Solution
Intercompany seiling price P150,000
Net book value carried on the books of Eloisa Company (100 000)
Intercompany gain on sale P50,000
Amortized for 5 years the remaining life (50,000/5 years) P10,000
Portion of the gain eliminated from depreciation (10/50) 20

9. On January 1, 2009, Pili, Inc. purchased 75% of Sili Co. for P500,000. On that date the equity of Sili consisted
of capital stock of P300,000 and retained earnings of P200,000. All assets and liabilities of Sili were fairly valued.
Goodwill, if any, is not amortized.
By January 2,2012, the reatined earnings of Sili had increased to P500,000. For 2012 Sili reported CI of P60,000
and paid dividends of P10,000. For 2013 Sili reported CI of P70,000 and paid dividends of 20,000.
On April 1,2012, Pili sold a land and an old office building on it. Pili's original cost for the land was P20,000;
the office building had a book value of P50,000. Sili paid P35,000 for the land and P40,000 for tge building. It
estimates that the building has a remaining life of 5 years.

For 2013, what is the balance in Pili's equity method Investment in Sili account?
a. P760,250 c. P775,000
b. P829,000 d. P791,500

Answer: D.
Investment in Sili Company stock Equity method
Price paid P500,000
Investment income net of dividends 2007 to 2010:
Increase in earnings (P500,000 P200,000) x 75% 225,000
Investment income, Dec. 31, 2010:
Share of Silis net income (P60,000 x 75%) 45,000
Unrealized gain on sale of land Downstream (15,000)
Unrealized loss on sale of building Downstream 10,000
Realized loss on sale of building (P10,000 / 5) x 75% ( 1,500) 38,500
Investment income, Dec. 31, 2011:
Share of Silis net income (P70,000 x 75%) 52,500
Realized loss (P10,000 / 5) (2,000) 50,500
Dividends received:
2010: (P10,000 x 75%) 7,500
2011: (P20,000 x 75%) 15,000. (22,500)
Investment in Sili Company stock, Dec. 31, 2011 P791,500

10. On January 1, 2014 Jel'z Corporation sells an equipment with a P 30,000 book value to its subsidiary Jom'z
Company for P 40,000. Jom'z intends to use the equipment for 5 years. On December 31, 2015 Jom'z sells the
equipment to an outside party for P 25,000. What amount of gain ( loss ) for the sale of assets is reported on the
consolidated financial statements?

a. loss of P 5,000 b. loss of P 1,000 c. gain of P 7,000 d. gain of P 25,000

Answer: c. gain of P 7,000

Solution:

Equipment Book Value P 30,000


Depreciation ( P 30,000 x 2/5 ) 12,000
Carrying Value P 18,000

Selling Price P 25,000


Carrying Value 18,000
Gain P 7,000

11. Pie Inc. owns 80% of Cake Company's outstanding common stock. Pie reports

cost of goods sold in the current year of P425,000 while Cake Co. reports P260,000. During the current year, Pie
Inc. sells inventory costing P125,000 to Cake Co. for P187,500. 60% of these goods are not resold by Cake
Company until the following year. What is consolidated cost of goods sold ?

a. P685,000
b. P497,500
c. P460,000
d. P535,000

Answer: D
Intercompany gross profit (P187,500 P125,000) P62,500
Inventory remaining at year end 60%
Unrealized Intercompany gross profit at 12/31. P37,500

Cost of good sold Pie Inc. P425,000


Cost of good sold Cake Co. 260,000
Remove intercompany sales 187,500
Deferred unrealized gross profit. 37,500
Consolidated cost of good sold P535,000

12. Presented below are several figures reported for Post Inc. and Mary Co. as of December 31 of the current
year which was the second year of owning Mary.

Two years ago, Post Inc. acquired 80% of Mary Co.'s outstanding common stock on January 1. The entire
difference between the amount paid and the fair value of Mary's net assets is attributed to a previously
unrecorded patent with a fair value of P112,500. The patent is being amortized over 20 years. During the first
year, Mary sold Post inventory costing P60,000 for P70,000. 30% of this inventory was not sold to external parties
until the following year. During the second year, Mary sold inventory costing P90,000 to Post for P115,000. Of this
inventory, 25% remained unsold on December 31 of the second year.
What is the amount of consolidated sales for the second year?

a. P815,000
b. P535,000
c. P608,000
d. P585,000

Answer:B

Post Incorporation's reported sales. P450,000


Mary Corporation's reported sales 250,000
Elimination of Intercompany sales 115,000
Consolidated sales P 585,000

13.On Jan. 1, 2016, Angelica Corporation acquired 90% of Ehrlyn Company. Angelica uses the cost method.
Analysis of data relative to this purchase indicates that goodwill of P50,000 was acquired in this purchase. The
goodwill is unimpaired.

On July 1, 2018, Ehrlyn sold a patent to Angelica. The sale price was P100,000; Angelicas book value was
P50,000. Ehrlyn estimates that the patent has a life of 5 years and no salvage value. It will use straight-line
depreciation.
For 2018, Angelica CI P400,000 from its own operations. Ehrlyn had CI of P100,000.
Consolidated CI for 2018 is:
a. P455,000
b. P450,000
c. P449.500
d. P440,000
Ans. A
Net income from own operations Pipe P400,000
Pipes share of Smokers adjusted net income:
Net income P100,000
Unrealized gain, July 1, 2018 Upstream (50,000)
Realized gain, Dec. 31, 2018 (P50,000/5)x 5,000 55,000
Consolidated net income, Dec. 31, 2018 P455,000
14.Several years ago Parent Corporation acquired 80% of Sub Corporation. Analysis of data relative to this
purchase indicates that goodwill of P60,000 was acquired in this purchase.
On Oct.1, 2016, Sub sold to Parent a used car for P32,000 in cash. Sub had originally paid P55,000 for the car;
on the day of the sale, the car had a book value of P23,000. Parent estimated the remaining life of the car at 3
years.
Parents CI from its own operations was P100,000 in 2016 and P120,000 in 2017. Subs CI was P60,000 in 2016
and P75,000 in 2017.
Consolidated CI attributable to parent for 2016 and 2017 are:
a. P138,000 and P179,400, respectively
b. P138,400 and P195,000, respectively
c. P138,000 and P179,000, respectively
d. P141,400 and P182,400, respectively
Ans. D
2016 2017
Net income from operations Parent P100,000 P120,000
Parents share of adjusted net income of Sub:
Net income P 60,000 P 75,000
Unrealized gain Upstream ( 9,000) -
Realized gain: 2016 (P9,000/3) x 750
2017 (P9,000/3) - 3,000
Adjusted net income P 51,750 P 78,000
Consolidated net income P151,750 P198,000
MINIS (10,350) (15,600)
Attributable to parent P141,400 P182,400

15. On January 1, 2015, Pure Company purchased 80% of the outstanding shares of Sure Company at a cost of
P1, 000, 000. On that date, Sure Company had P400, 000 of capital stock and P600, 000 of retained earnings.

On July 1, 2015, Sure Company sold equipment with a book value of P60, 000 to Pure Company for P80, 000.

For 2015 and 2016, the results of their operations are:


20152016
Pure Co.Sure Co.Pure Co.Sure Co.
Net income from own operationsP400, 000P200, 000P300, 000P150, 000
Dividends paid100, 00050, 00080, 00020, 000

The intercompany gain is included in the net income of Sure Company. The equipment sold is expected to have a
useful life of five years from date of sale.

The non-controlling interests on December 31:

20152016
a. P226, 400 P256, 800
b. P226, 400 P253, 200
c. P226, 000 P252, 400
d. P230, 000 P256, 000

Ans. : B

20152016
Capital stock Sure, December 31P400, 000P400, 000
Retained earnings Sure, December 31:
Retained earnings Sure, January 1P600, 000P750, 000*
Add: Net income200, 000150, 000
TotalP800, 000P900, 000
Less: Dividends50, 000750, 000*20, 000880, 000
Stockholders equity Sure, 12/31P1, 150, 000P1, 280, 000
Less: Unrealized gain (upstream sales)20, 000*18, 000*
P1, 130, 000P1, 262, 000
Add: Realized gain thru depreciation (upstream sales): (P20, 000 / 5 x 6/12)2, 0004, 000
Realized stockholders equity Sure, 12/31P1, 132, 000P1, 266, 000
Multiplied by: Non-controlling interest %20%20%
Non-controlling interests, December 31P226,400P253,200(b)

There is no step-up value in fair value of net assets, so adjustment would not be necessary. The excess of cost
over book represents goodwill which has no bearing in the computation of non-controlling interest (minority
interest).

There was a goodwill arising from acquisition amounting to P200, 000 [P1, 000, 000 (P1, 000, 000 x 80%)], any
impairment losses arising from this goodwill should be ignored for purposes of computing minority interests. But
any adjustments to reflect fair value less amortization should be recognized.

In an upstream sales (subsidiary is the seller) the unrealized gain on sale of equipment amounting to P20, 000
was included in the P200, 000 net income and since it was on intercompany gain, so there is a need to eliminate
such gain from consolidated point of view.
However, in subsequent year 2016, the retained earnings of Sure for January 1, 2016 includes the unrealized
gain of P18, 000 (P20, 000 P2, 000) for reason that P200, 000 net income of 2015 was carried over to 2016,
therefore, there is a need to reflect such deduction of P18, 000 for consolidated point of view.

16. On January 1, 2016, P Company purchased 80% of the outstanding shares of S Company by paying P700,
000. On that date, S Company had P300, 000 capital stock and P500, 000 of retained earnings. An undervalued
asset attributable to building amounting to P75, 000 with a remaining useful life of 25 years. All other assets and
liabilities of S Company had book value approximated their fair market value.

On January 1, 2017, Ps common stock and retained earnings amounted to P1, 000, 000 and P800, 000,
respectively, while S Companys retained earnings is P600, 000.

The 2017 net income and dividends using cost (or initial value) model was as follows:
Net incomeDividends
P CompanyP340, 000P100, 000
S CompanyP150, 000P50, 000

On April 1, 2017, S Company sold equipment with a book value of P30, 000 to P Company for P60, 000. The gain
on sale is included in the net income of S Company indicated above. The equipment is expected to have a
remaining useful life of five years from the date of the sale.

On September 30, 2017, P Company sold machinery with a book value of P40, 000 to S Company for P75, 000.
The gain on the sale is also included in the net income of P Company indicated above. The machinery is
expected to last for ten years from the date of sale.

The non-controlling interest in net income for 2017:

a. P30, 000
b. P25, 500
c. P24, 900
d. P24, 300

Ans.: D
Solution:

Net income of subsidiaryP150, 000


Less: Unrealized gain on sale of equipment (upstream) year of sale30, 000
Amortization of allocated excess3, 000
P117, 000
Add: Realized gain on sale of equipment (upstream) 20174, 500
P121, 500
Multiplied by: Non-controlling interests20%
Non-controlling interests in net incomeP24, 300

FV of Subsidiary:
Consideration transferredP700, 000
Less: Book value of SHE S, 1/1/2016
[(P300, 00 + P500, 000) x 80%]640, 000
Allocated excess60, 000
Less: Over/Under Valuation of Assets and Liabilities:
Increase in buildings: P75, 000 x 80%60, 000
GoodwillP0

Note: As a consequence to determined excess, there is no goodwill (full or partial) arising therefrom:
Amortization of allocated excess: P75, 000 / 25 yearsP3, 000
Upstream Sale of Equipment (date of sale 4/1/2017):
Sales P60, 000
Less: Book value of equipment 30, 000
Unrealized Gain (on sale of equipment) 30, 000
Realized gain on sale of equipment:
2017 P30, 000/ 5 years = P6, 000 x 9/12
(4/1/2017 12/31/2017) P4, 500
2018 P6, 000

Downstream Sale of Machinery ( date of sale 9/30/17):


Sales P75, 000
Less: Book value of machinery 40, 000
Unrealized Gain (on sale of machinery) P35, 000
Realized gain on sale of machinery:
2017:P35, 000 / 10 years = P3, 500 x 3/12
(9/1/2017 12/31/2017) P875
2018 P3500

17. On January 1,2015,Alex Corporation sold equipment to Arse Company, its owned subsidiary, for
P680,000.Alex had paid P1000,000 for this equipment , for which the depreciation to the date of intercompany
sale totalled P360,000.Both companies use the straight line method of depreciation for their depreciable assets.
The equipment had a 10 year life when purchased and an expected salvage value of P100,000. What amount
should be included in the consolidated statement of financial position at December 31, 2015, for the equipment
cost and accumulated depreciation?

a. P1000,000 and P360,000


b. P680,000 and P450,000
c. P1000, 000 and P450,000
d. P680,000 and P360,000

Ans. C
Equipment at original cost P1,000,000

Accumulated depreciation:
Time of sale P360,000
Current depreciation (P900,000 /10) 90,000 P 450,000

18. On January 1, 2016, Les Company purchased 80% of the outstanding stock of Noriel Company at a cost of
P720,000. On the date, Noriel Company had P400,000 of capital stock and P500,000 of retained earnings .
For 2016, Noriel Company reported income of P180,000 and paid dividends of P60,000. All the assets and
liabilities of Noriel Company are at fair market value.
On December 31,2016, Les Company sold equipment to Noriel Company for P75,000 that had a cost of P45,000.
The equipment is expected to have a useful life of 10 years from this date. Les uses the cost method to account
for its investment in Noriel.
The amount of consolidated CI attributable to parent on December 31, 2016 is:

a. P320,000
b. P200,000
c. P314,000
d. P200,000

Ans. C
Net income Po P200,000
Unrealized gain, Dec. 31 DS (30,000)
Net income from own operation Po 270,000
Net income of So 180,000
Consolidated net income, Dec. 31, 2016 P350,000
MINIS (P180,000 x 20%) (36,000)
Attributable to parent P314,000

19. Dalton Corp. owned 70% at the outstanding common stock of Shrugs lnc. On January 1, 20x4. Dalton
acquired a building with a ten-year life for P420,000 No salvage value was anticipated and the building was to be
depreciated on the straight-line basis. On January 1, 20x6. Dalton sold this building to Shrugs for P392000. At
that time, the building had a remaining life of eight years but still no expected salvage value. ln preparing financial
statements for 20x6, how does this transfer affect the calculation of Daltons share at consolidated net income?
A. Consolidated net income must be reduced by P44.800
B. Consolidated net income must be reduced by P50.400
C. Consolidated net income must be reduced by P49.000
D. Consolidated net income must be reduced by P56.000

ANSWER: C
20X6
Unrealized gain on sale of equipment (56,000)
Realized gain on sale of equipment(upstream salaes) 7,000
through depreciation
Net (49,000)

Selling price P 392,000


Less: book value1/1/20x6
Cost 1/1/20x2 P 420,000
Less: accumulated depreciation:P 420,000/10yrs. x 2yrs. 84,000 336,000
Unrealized gain on sale of equipment P 56,000
Realized gain depreciation: P56,000/8yrs. P 7,000

20. On January 1, 20x4, GG Company purchased a computer with an expected economic life of five years.
On January l, 20x6, GG sold the computer, to TLK Corporation and recorded. the following entry:

Cash .................................................................. 39,000


Accumulated Depreciation ................................ 16.000
Computer Equipment .............................. 40,000
Gain on Sale of Equipment ..................... 15,000

TLK Corporation holds 60 percent of GG's voting shares. GG reported net income of P45,000. And TLK reported
income from its own operations of P85,000 for 20x6. There is no change in the estimated economic life of the
equipment as a result of the intercorporote transfer. In the preparation of the 20x6 consolidated income
statement. depreciation expense will be:
A. Debited for P5,000 in the eliminating entries
B. Credited for P5,000 in the eliminating entries
C. Debited for P13,000 in the eliminating entries.
D. Credited for P13.000 in the eliminating entries

ANSWER:
The P39,000 paid to (36 Company will be charged to depreciation expense by TLK Corporation over the
remain'ng 3 years of ownership. As a result. TLK Corporation wil debit depreciation expense for Pl3,000 each
year. (36 Company had charged Pl.000 to accumulated depreciat'on in 2 years, for an annual rate of P8000.
Depreciation expense therefore must be reduced by P5,000 (P13.000 P8,000] in preparing the consolidated
statements
21. Pied Imperial-Pigeon Corporation acquired a 90% interest in Offshore Corporation in 2003 when Offshore'
book values were equivalent to fair values. Offshore sold equipment with a book value of $80,000 to Pied
Imperial-Pigeon for $130,000 on January 1, 2005. Pied Imperial-Pigeon is fully depreciating the equipment over a
4year period by using the straight-line method. Offshore' reported net income for 2005 was $320,000. Pied
lmperial-Pigeon's 2005 net income from Offshore was
a. $249,250.
b. $250,500.
c. $254,250.
d. $288,000.

Answer: c

Pied lmperial-Pigeon's share of Roger's


income = (320,000 x 90%) = $ 288,000
Less: Profit on intercompany sale
(130,000 - 80,000) x 90% = ( 45,000 )
Add: Piecemeal recognition of
deferred profit ($50,000/4 years) x 90% = $ 11,250
Income from Offshore $ 254,250

22.Root Corp. owns 100% of Beer Corp.s common stock. On January 2, 20x5, Root sold to Beer for P40,000
machinery with a carrying amount of P30,000. Beer is depreciating the acquired machinery over a five year life by
the straight-line method. The net adjustments to compute 20x5 and 20x6 Profit Attributable to Equity Holders of
Parent or CNI Attributable to Controlling Interests before income tax would be an increase (decrease) of:

20x5 20x6
a. P( 8,000) P 2,000
b. ( 8,000) 0
c. (10,000) 2,000
d. (10,000) 0

Ans: a

20x5 20x6
Unrealized gain on sale of machinery P(10,000)
Realized gain on sale of machinery
P10,000/5 2,000 P 2,000
Net adjustments P( 8,000) P 2,000

23.The Pine Company owns 65% of The Apple Company. On the last day of the accounting period Apple sold to
Pine a non-current asset for P200,000. The asset originally cost P500,000 and at the end of the reporting period
its carrying amount in Apples books was P160,000. The groups consolidated statement of financial position has
been drafted without any adjustments in relation to this non-current asset.
What adjustment should be made to the consolidated statement of financial position figures for non-current assets
and retained ernings?

Non-current assets Retained earnings


a. Increase by P300,000 Increase by P195,000
b. Reduce by P40,000 Reduce by P26,000
c. Reduce by P40,000 Reduce by P40,000
d. Increase by P300,000 Increase by P300,000

Ans: b

Upstream Sales:
Selling price of non-current assets P200,000
Less: Book/carrying value, date of sale 160,000
Gain on intercompany sale P 40,000

Eliminating Entries would be:


Retained Earnings Parent (65% x P40,000) P26,000
Non-controlling interest (35% x P40,000) 14,000
Non-current asset P40,000
24. Steve Co. is 80% - owned subsidiary of Peter Co., acquired at book value several years ago.
Comparative separate company income statements for these affiliated corporations for 2016 are as
follows:
Peter Co. Steve Co.
Sales. P3,000,000 P1,400,000
Dividend income. 216,000 -
Gain on building. 60,000 -
Income credits. P3,276,000 P1,400,000
Cost of sales.. P2,000,000 P 800,000
Operating expenses.. 600,000 300,000
Income debits.. P2,600,000 P1,100,000
Net income... P 676,000 P 300,000

On January 5, 2016 Peter Co. sold a building with a 8-year remaining useful life to Steve Co. as a gain of
P60,000. Steve Co. paid dividends of P240,000 during 2016.
The Non-controlling interest in net income and CNI Attributable to Controlling Interests for 2016, should
be:
a. P80,000; P640,000
b. P80,000; P647,500
c. P60,000; P647,500
d. P60,000; P640,000

ANSWER: (c)

Profit Attributable to Equity holders of parent 2016


Net income from own operation:
Peter Co.:
Net income. P676,000
Less: Dividend Income. 216,000 P460,000
Steve Co. 300,000
P760,000
Less: Unrealized gain on sale of building.. 60,000
Add: Piecemeal recognition of excess depreciation/
Realized gain thru depreciation (P60,000/8-yrs.) 7,500
P707,500
Less: Amortization of allocated excess.. 0
Non-controlling interests in net income
(P300,000 x 20%). 60,000 (c)
Profit Attributable to Equity Holders of parent P647,500 (c)

25. On January 1, 2013, P Company purchased 80 percent of the outstanding shares of S Company at a cost of
P700,000. On that date, S Company had P300,000 of capital stock and P500, 000 of retained earnings.

For 2013, P Company had CI of P300,000 from its own operations and paid dividends of P 100,000. For 2013, S
Company reported a CI of P 150, 000 and paid dividends of P50,000. All of the assets and liabilities of S
Company had book values approximately equal to their respective market values.
On April 1, 2013, S Company sold equipment with a book value of P30,000 to P Company for P60,000. The gain
on the sale is included in the Cl of S Company indicated above. The equipment is expected to have a useful life of
five years from the date of the sale.

Consolidated CI attributable to parent for 2013 is:

a. P397,200
b. P423,600
c. P400,800
d. P399,600

ANSWER: D

Consolidated Net Income


Net income from own operations - P Company P300,000
Adjusted net income of S Company:
Net income - S P150,000
Unrealized gain, 4/1/11 Upstream ( 30,000)
Realized gain, 12/31/11 (P30,000/5) x 9/12 4,500 124,500
Consolidated net income 424,500
Attributable to NCI (P124,500 x 20%) (24,900)
Attributable to parent P399,600

26. Apex, Inc., and Small, Inc., formed a business combination on January 1, 2007, when Apex acquired a 60
percent interest in the common stock of Small for P372,000. The book value of Small on that day was P350,000.
Patents held by the subsidiary (with a 12-year remaining life) were undervalued within the companys accounting
records by P120,000. Any goodwill indicated by the acquisition price is not amortized.

Intercompany inventory sales between the two companies have been made as follows:

Transfer Price Ending Balance


Year Cost to Apex to Small (at transfer price)
2007 P60,000 P72,000 15,000
2008 70,000 84,000 25,000
2009 80,000 100,000 20,000
2010 100,000 125,000 40,000
2011 90,000 120,000 30,000
2012 120,000 150,000 50,000
2013 112,000 160,000 40,000

Small sold a building to Apex on January 1, 2011, for P80,000. The building had a net book value of P30,000 on
that date and a five-year life. No salvage value was expected for this asset which was being depreciated by the
straight-line method.

The individual financial statements for these two companies as of December 31,2013 and the year then ended
follow:

Apex Small
Inc. Inc.

Sales P 700,000 P300,000


Cost of goods sold ( 460,000) ( 205,000)
Operating expenses ( 170,000) ( 70.000)
Dividend income (from Small) 3,000 -0-
P 73,000 P 25,000

Retained earnings, January 1, 2013 P 690,000 P400,000


CI (above) 73,000 25,000
Dividends paid ( 45,000) ( 5,000)
Retained earnings, December 31, 2013 P 718,000 P 420,000

Cash and receivables P 275,000 P142,000


Inventory 233,000 229,000
Investment in Small Company 372,000 -0-
Buildings (net) 308,000 202,000
Equipment (net) 220,000 86,000
Patents (net) -0- 20,000
Total assets P1,408,000 P679,000

Liabilities P 390,000 P159,000


Common stock 300,000 100,000
Retained earnings, December 31, 2013 713,000 420,000
Total liabilities and equities P1,408,000 P 679,000

What is the consolidated balance of retained earnings, December 31, 2013?

a. P612,000
b. P602,000
c. P620,000
d. None of the above

ANSWER: B

Consolidated Retained Earnings, December 31, 2013


Retained earnings, Jan. 1, 2013 Apex P 690,000
Amortization of patents 2007 to 2013 (P10,000 x 6) ( 60,000)
Unrealized profit on inventory, 2013 - Downstream ( 10,000)
Unrealized gain on sale of building, 12/31/13 - Upstream (P30,000 x 60%) ( 18,000)
Consolidated retained earnings, Dcember 31, 2013. P 602,000

27. Gray Corporation is a 90% owned subsidiary of Green Corporation acquired several years ago at
book value equal to fair value. For the years 2015 and 2016, Proto and Silver report the following:

2015 2016

Greens separate income P250,000 P350,000

Grays net income 100,000 80,000


The only intercompany transaction between Green and Gray during 2015 and 2016 was the January 1,
2015 sale of land. The land had a book value of P20,000 and was sold intercompany for P30,000, its
appraised value at the time of sale.

If the land was sold by Green to Gray and that Gray still owns the land at December 31, 2016, compute
the profit attributable to equity holders of parent for 2015 and 2016.

a. P340,000 ; P422,000
b. P330,000 ; P422,000
c. P340,000 ; P480,000
d. P330,000 ; P480,000

Answer: B

2015 2016

Greens net income from own operation P250,000 P350,000

Unrealized gain on sale of land ( 10,000) -

Total P240,000 P350,000

Grays net income attributable to parent:

2015 (P80,000 x 90%) 90,000

2016 (P60,000 x 90%) 72,000

Total net income attributable to parent P330,000 P422,000

28.On January 1, 2016, King Company purchases 80% of the outstanding stock of Star Company at s
cost of P800,000. On that date, Star Companys shareholders equity amounted to P800,000.

On April 1, 2016, King Company sold equipment with a book value of P50,000 to Star Company for
P90,000. The gain included in the 2016 net income of King Company. The equipment is expected to have
a remaining useful life of five years.

For 2016, the net income from own operations are:


King Company P500,000
Star Company 262,500

King Company used the equity method to account for its investment in Star Company.

For 2016, what is the balance of Investment income account in the books of King Company?
a. P172,000
b. P176,000
c. 271,000
d. P170,000
Answer: B
Share in Stars income (P262,500 x 80%) P210,000

Unrealized gain (P90,000 P50,000) ( 40,000 )

Realized gain (P40,000/5) x 9/12 6,000

Investment Income account balance, 12/31/16 P176,000

29. Basilio Corporation owns 100% of Longalongs Corporations common stock. On January 1,
2016, Basilio sold to Longalong for P440,000 machinery with the carrying amount of P30,000. Longalong
is depreciating the acquired machinery over a five year life by straight-line method. The net adjustments
to compute 2016 and 2017 Profit Atrributable to Equity Holders of Parent or CNI Attributable to
Controlling Interest before income tax would be an increase (decrease):
2016 2017
a. P(8,000) P2,000
b. P(8,000) -0-
c. P(10,000) P2,000
d. P(10,000) -0-

ANSWER; A
Rationale
2016 2017
Unrealized Gain on sale of machinery P(10,000)
Realized Gain on sale of m,achinery (10,000/5) 2,000 P2,000
Net Adjustments P(8,000) P2,000

30. On January 1,2016 Yellow Company purchase 80% of the outstanding shares of Orange Company at
a cost of P1,000,000. On that date, Orange Co. had P400,000 of capital stock and P600,000 of retained
earnings.
On July 1,2016 Orange Company sold an equipment with a book value of P60,000 to Yellow Co. for
P80,000
For 2016 and 2017, the results of their operations are:
2016 2017
Yellow Orange Yellow Orange
Net income from own operations P400,000 P200,000 P300,000 P150,000
Dividends paid 100,000 50,000 80,000 20,000

The intercompany gain is included in the net income of Orange Co.. The equipment sold is expected to
have a useful life of five years from the date of sale.

The non-controlling interest on December 31:

2016 2017
a. P226,400 P256,800
b. P226,400 P253,200
c. P226,000 P252,400
d. P230,000 P256,000
ANSWER:B
Rationale
2016 2017
Capital stock-Orange 12/31 P400,000 P400,000
Retained Earnings, Orange 12/31:
Retained Earnings, Orange 1/1 P600,000 P750,000
Add: Net Income 200,000 150,000
Total P800,000 P900,000
Less: Dividends 50,000 750,000 20,000 880,000
Stockholders Equity- Orange, 12/31 P1,150,000 P1,280,000
Less: Unrealized Gain (Upstream Sales) 20,000 18,000
Total P1,130,000 P1,262,000
Add: Realize gain thru Depreciation
(upstream sales): P20,000/ 5 x 6/12) 2,000 4,000
Realized Stockholders Equity- Orange, 12/31 P1,132,000 P1,266,000
Multiplied by: Non-controlling Interest % 20% 20%
Non-controlling Interests, December 31 P226,400 P253,200

31. On January 1,2016, P Company purchased 80 percent of the outstanding shares of S Company by paying
P700,000. On that date, S Company had P300,000 capital stock and P500,000 of retained earnings. An
undervalued asset attributable to building amounting to P75,000 with a remaining life of 25 years. All other assets
and liabilities of S Company has book value approximated their fair market value.
On January 1,2017, Ps common stock and retained earnings amounted to P1,000,000 AND P800,000,
respectively, while S Companys retained earnings is P600,000.
The 2017 net income and dividends using cost (initial value) model was as follows:
Net Income Dividends
P Company P340,000 P100,000
S Company P150,000 P 50,000

On April 1,2017, S Company sold equipment with a book value of P30,000 to P Company for P60,000. The gain
on the sale is included in the net income of S Company indicated above. The equipment is expected to have a
remaining useful life of five years from the date of the sale.
On September 30,2017, P Company sold machinery with a book value of P40,000 to S Company for P75,000.
The gain on the sale is also included in the net income of P Company indicated above. The machinery is
expected to last for ten years from the date of sale.
The non-controlling interest in Net Income for 2017:
a. P30,000 c. P24,900
b. P25,500 d. P24,300
Solution:
ANS: D
Net income of Subsidiary P150,000
Less: Unrealized gain on sale of equipment (upstream)-
year of sale 30,000
Amortization of allocated excess 3,000
P117,000
Add: Realized gain on sale of equipment (upstream) 2017 4,500
P121,500
Multiplied by: Non-Controlling Interests 20%
Non-controlling interests in net income P 24,300

FV of Subsidiary:
Consideration transferred P700,000
Less: Book value of SHE-S,1/1/2016
[(P300,000 + P500,000) x 80%] 640,000
Allocated excess P 60,000
Less: Over/Under Valuation of A and L:
Increase in Buildings:P75,000 x 80% 60,000
Goodwill P 0
Note: As a consequence to determine excess, there is no goodwill (full or partial) arising therefrom.
Amortization of allocated excess: P75,000 / 25years P3,000
Upstream sale of Equipment (date of sale-4/1/2017):
Sales P60,000
Less: Book Value of equipment 30,000
Unrealized Gain (on sale of equipment) P30,000
Realized gain on sale of equipment:
2017:P30,000/5years=P6,000 x 9/12
(4/1/2017-12/31/2017) P 4,500

2018 P 6,000
Downstream sale of machinery (date of sale-9/30/2017):
Sales P75,000
Less: Book Value of machinery 40,000
Unrealized Gain (on sale of machinery) P35,000
Realized gain on sale of machinery:
2017:P35,000/10years=P3,500 x 3/12
(9/30/2017-12/31/2017) P 875

2018 P 3,500
32. On January 1,2016, Edi Company purchased 80 percent of the outstanding shares of Lyn Company at a cost
of P1,000,000. On that date, Lyn Company had P400,000 of capital stock and P600,000 of retained earnings.

On July 1,2016, Lyn Company sold an equipment with a book value of P60,000 to Edi Company for P80,000.
For 2016 1nd 2017, the results of their operations are:
2016 2017
Edi Co. LynCo. Edi Co. LynCo
Net income from own operations P400,000 P200,000 P300,000 P150,000
Dividends paid 100,000 50,000 80,000 20,000

The intercompany gain is included in the net income of Lyn Company. The equipment sold is expected to have a
useful life of five years from the date of sale.
The Non-controlling interests on December 31:
2015 2016
a. P226,400 P256,800
b. P226,400 P253,200
c. P226,000 P252,400
d. P230,000 P256,000
solution:
ANS: B
2016 2017
Capital stock-Lyn, December 31 P400,000 P400,000
Retained earnings-Lyn, December 31:
Retained earnings-Lyn,January1 P600,000 P750,000
Add: Net Income 200,000 150,000
Total P800,000 P900,000
Less: Dividends 50,000 750,000 20,000 880,000
Stockholders;equity-Lyn,12/31 P1,150,000 P1,280,000
Less: Unrealized gain (upstream sales) 20,000 18,000
P1,130,000 P1,262,000
Add: Realized gain thru depreciation
(upstream sales): P20,000/5x6/12 2,000 4,000
Realized Stockholdersquity-Lyn,12/31 P1,132,000 P1,266,000
Muti[lied by: Non-controlling Interest % 20% 20%
Non-controlling interests, December 31 P 226,400 P 253,000

There is no step-up value of net assets, so adjustment would not be necessary. The excess of cost over book
represents goodwill which have no bearing in the computation of minority interests.

There was a goodwill arising from acquisition amounting to P200,000 [P1,000,000-(80% x P1,000,000)], any
impairment losses arising from this goodwill should be ignored for purposes of computing minority interests. But
any adjustments to reflect fair value less amortization should be recognized.

In an upstream sales, the unrealized gain on sale of equipment amounting to P20,000 was included in the
P200,000 net income and since it was an intercompany gain, so there is a need to eliminate such gain from
consolidated point of view.

However, in subsequent year 2017, the Retained Earnings of Lyn for January 1,2017 includes the net unrealized
gain of P18,000(P20,000-P2,000) for reason that P200,000 net income of 2016 was carried over to 2017,
therefore, there is a need to reflect such deduction of P18,000 for consolidation point of view.

33. On January 1,2016, Red Inc.sold equipment with a four-year remaining useful life and a book value of
P350,000 to its 65%-owned subsidiary for a price of P530,000. In consolidation working papers for the year ended
December 31,2016, the elimination entry concerning this transaction will include:

A.A debit to equipment for P180,000


B.A credit to depreciation expense for P180,000
C.A debit to cash for P350,000
D.A debit to gain on equipment for P180,000

Answer: D

100% unrealized gain and restore the original book value ,date of sale 1/1/16
Gain on sale ........................................................P180,000
Equipment....................................... P180,000
The entry made in the books of subsidiary on the date of sale:
Equipment..........................................................P 530,000
Cash................................................ P530,000
The entry made in the books of parent on the date of sale:
Cash...................................................................P530,000
Equipment..................................... P350,000
Gain on Sale.................................. 180,000

From consolidated point of view, there should be no gain. Therefore, to eliminate the gain should be debited and
equipment should be reduce accordingly.

For depreciation:
Accumulated depreciation.................................P45,000
Depreciation expense (P180,000/4 years) P45,000

34. The Reyes Co.acquired equipment on January 1,2015 at a cost of P1,500,000,depreciating it over 7 years
with a nil residual value. On January 1,2018. The Santos Co.acquired 100% of Reyes and estimated the fair value
of the equipment at P900,000, with a remaining life of 4 years. This fair value was not incorporated into Reyes
books and the depreciation expense continued to be calculated by reference to original cost. The carrying amount
of the statement of financial position in preparing the consolidated financial statement for the year ended
December 31,2019:

A.Increase by P418,000
B.Decrease by P418,000
C.Decrease by P225,000
D.Increase by P225,000A

Answer: B

Net carrying /book value 12/31/17:


Reyes Co.s book value [P1,500,000-(P1,500,000/7 years x 4years)]....................P643,000
Santos Co;s book value [P900,000-(P900,000/4years x 3 years)]....................... 225,000

Decrease........................................................................................................... P418,000 (B)

35.
Kestrel Company acquired an 80% interest in Reptile Corporation on January 1, 2004. On
January 1, 2005, Reptile sold a building with a book value of $50,000 to Kestrel for $80,000.
The building had a remaining useful life of ten years and no salvage value. The separate
balance sheets of Kestrel and Reptile on December 31, 2005 included the following balances:

Kestrel Reptile

Buildings $ 400,000 $ 250,000

Accumulated Depreciation - 120,000 75,000

Buildings

The consolidated amounts for Buildings and Accumulated Depreciation - Buildings that
appeared, respectively, on the balance sheet at December 31, 2005, were

a. $620,000 and $192,000.

b. $620,000 and $195,000.

c. $650,000 and $192,000.

d. $650,000 and $195,000.

ANSWER:
Combined building amounts $ 650,000
Less: Intercompany gain ( 30,000 )
Consolidated building amounts $ 620,000

Combined Accumulated Depreciation $ 195,000


Less: Piecemeal recognition of gain
( 3,000 )
Consolidated accumulated depreciation
$ 192,000

36. The Roel Company acquired equipment January 1, 2013 at accost of 800,000, depreciating it over 8
years with a nil residual value. On January 1, 2016. The Muldon Company acquired 100% of Roel and estimated
the fair value of the equipment at 460,000, with a remaining life of 5 years. This fair value was not incorporated
into Roels book and the depreciation expense continued to be calculated by reference to original cost.

What adjustment should be made to the depreciation expense for the year and the statement of financial position
carrying amount in preparing the consolidated financial statements for the year ended December 31, 2017?

Depreciation Expense Carrying Amount


a. Increase by 8,000 Increase by 24,000
b. Increase by 8,000 Decrease by 24,000
c. Decrease by 8,000 Increase by 24,000
d. Decrease by 8,000 Decrease by 24,000

ANS:D
Fair value adjustments under PFRS 3 par.36 not reflected in the books must be adjusted for on consolidation.

Annual depreciation expense:


Roels depreciation: (800,000-0)/8 years 100,000
Muldons depreciation (460,000/5) 92,000
8,000

Net carrying/book value, 12/31/2017:


Roels book value (800,000-(800,000/8 years X 5yrs) 300,000
Muldons book value (note) (460,000-(460,000/5 X 3years) 276,000
Decrease 24,000

37. Sales of fixed assets between members of an affiliated group may result in the recognition of gain or loss by
the seller.
Statement 1: For each period, adjust depreciation expense and accumulated depreciation to reflect the original
BV of the asset.

Statement 2.For periods subsequent to the year of sale, restore the carrying amount of the asset to its original BV
and eliminate the gain (loss) recorded by the seller.

A.Statement 1 is false; Statement 2 is true

B.Both statements are correct

C.Statement 1 is true; Statement 2 is false


D.Both statements are false

Answer: C

38. Marcelinos Co.:D owned 80% of Sarahs Corp. During 2016, Marcelinos sold to Sarahs land with a book
value of 48,000. The selling price was 70,000. In its accounting records, Marcelinos should.
a. Not recognized a gain on the sale of the land since it was made to a relative party.
b. Recognized a gain of 17,600
c. Defer recognition of the gain until Sarahs sells the land to a third party.
d. Recognize a gain of 22,000

ANS: D
It should be noted that the gain of 22,000 (70,000-48,000) should be recorded in the Marcelinos (Parent)
accounting records, but eliminated in the Consolidated Financial Statement.

39.In reference to the downstream or upstream sale of depreciable assets, which of the following statements is
correct?

a. Upstream sales from the subsidiary to the parent company always result in unrealized gains or losses.

b. The initial effect of unrealized gains and losses from downstream sales of depreciable assets is different from
the sale of non-depreciable assets.

c. Gains, but not losses, appear in the parent-company accounts in the year of sale and must be eliminated by the
parent company in determining its investment income under the equity method of accounting.

d. Gains and losses appear in the parent-company accounts in the year of sale and must be eliminated by the
parent company in determining its investment income under the equity method of accounting.

ANSWER: D

HOME OFFICE,BRANCH AND AGENCY ACCOUNTING

1.Lacoste Philippines has two merchandise outlets, its main store in Manila and in Cebu City branch. For control
purposes, all purchases are made by the main store, and shipmentsto the Cebu City branch are at cost plus 10%.
On January 01, 2016, the inventories of the main store and Cebu City branch were P13,600 and P3,960,
respectively. During 2016, the main office purchased merchandize costing P40,000 and shipped 40% of these to
the Cebu City branch.

At December 31, 2016, the following journal entry was made to prepare the Cebu City branch books for the next
accounting period:
Sales 32,000
Inventory 4,840
Inventory 3,960
Shipments from the main store 17,600
Expenses 10,480
Main store 4,800
[1]what was the actual branch income of 2016 on a cost basis, assuming the use of the provisions of the PAS,
and [2] if the main store has P11,200 worth of inventory on hand at the end of 2016, the total inventory that should
appear on the combined balance sheet at December 31, 2016?

a. [1] P4,800 [2] P15,600 c. [1] P6,320 [2] P15,600

b. [1] 6,320 [2] 15,160 d. [1] 6,480 [2] 16,040

Ans: C

[1] actual branch income:


Sales..32,000
Less: Costs of Goods Sold
Inventory, Jan. 1, at billed price..P 3,960
Shipments from Main store, at billed price 17,600

Cost of goods available for sale, at billed


Price..P 21,560
Less: inventory, December 31, at billed
Price . 4,840

Cost of goods sold at billed price....P 16,720


Multiplied by: cost ratio . 100/110 15,200
Gross ProfitP 16,800
Less: Expenses. 10,480
True Branch Net Income.P 6,320 (c)

[2] Ending Inventory at Cost:


Home office.P 11,200
Branch: (P4,840 x 100/110). 4,400
Combined ending inventory at costP 15,600 (c)

2.Barros Corporations shipments to and from its Brazil City branch are billed at 120% of cost. On December 31,
Brazil branch reported the following data at billed prices: inventory, January 1 of 33,600; shipments received from
home office of 840,000; shipment returned of 48,000; and inventory; December 31, of 36,000.
What is the balance of the allowance for over valuation of branch inventory on December 31 before adjustments?
a. 5,600
b. 137,600
c. 6000
d. 145,600

ANS:B
Inventory, January 1 33,600
Add: Shipments from office, net o returns
(840,000-48,000) 792,000
Cost of goods available for sale 825,600
Multiplied by: Mark up 20/120
Allowance for overvaluation before adjustments 137,600

3.Philippine Overseas Corporation has operated a branch in Jordan for one year . shipments are billed to the
branch at cost. The branch carries its own expenses. The transactionsfor the year are given effect to in the trial
balance below:
Accounts Debit Credit
Cash P4,200
Home office Current P17,500
Shipments From Home Office 67,680
Accounts Receivable 12,800
Expenses 6,820
Sales 74,000
P91,500 P91,500
The branch reported on inventory on December 31, 20x5 of P9,180.
The net profit of the Jordan Branch for 20x5 was:
a. P8,680 c.P6,718
b. P9,180 d. Some other answer.

ANSWER: A
Sales P 74,000
Less: Cost of goods sold:
SFHO P67,680
Less: Inventory, ending 9,180 58,500
Gross profit P 15,500
Less: Expenses 6,820
Net Profit P 8,680

4.The Clark branch of Freeport Corp. submitted the following trial balance as of 30 June 2016:

Debit Credit
Cash P 28,600
Accounts Receivable 173,800
Shipments from home office 462,000
Home-office current P324,500
Sales 369,600
Expenses __ 29,700 ________
Total P694,100 P694,100
Clark reported an ending inventory of P138,600, Shipments are billed at a mark-up of 40% on cost. What is the
real net income of Clark Branch?
a. P70,000
b. P92,400
c. P100,000
d. P108,900

ANS: D

Sales P369,600
Less: Cost of Goods Sold
Shipments from Home Office,
at cost (P462,000 x 100/140) P330,000
Less: Ending Inventory,
at cost (P138,600 x 100/140) 99,000 231,000
Gross Profit P138,600
Less: Expenses 29,700
Real Net Income of the Branch P108,900

5.Tillman Textile Co. has a single branch in Bulacan. On march 1, 2016, the Home Office accounting records
included an Allowance for Overvaluation of Inventories-Bulacan Branch ledger account with a credit balance of
P32,000. During March, merchandise costing P36,000 was shipped to the Bulacan Branch and billed at a price.
On March 31, 2016, the branch prepared an income statement indicating a net loss of P11,500 for march and
ending inventories at a billed prices of P25,000. What is the amount of adjustment for Allowance for Overvaluation
of Inventories to reflect the true branch net income?

a. P39,257 debit
b. P46,000 credit
c. P39,333 debit
d. P46,000 debit
ANS: D
Merchandise Inventory, March 1, 2016 P32,000
Add: Shipments (P36,000/60% = 60,000 x 40%)
Note: Mark-up is based on billed price 24,000
Cost of Goods available for sale P56,000
Less: Merchandise inventory, March 31, 2016
(25,000 x 40%) 10,000
Overvaluation of CGS/Realized the Gross Profit on Branch Sales P46,000

6. As you begin to audit the books of the FIL-EM Company. YOU notice a discrepancy between the balance in the
Investment in Branch (P136,020 Dr.) and the Home Office (P175,400 Cr.) accounts. The followmg Information is
available:

A. The home office bills goods shipped to the branch at 150% of cost. At the beginning of the year, branch
inventory was stated at P7S,000 after the annual physical count, and the home office unrealized profit account
had a credit balance of PS,000. You find that a shipment with a billed value of P60,000 made toward the end of
the prior year had not been recorded by the home office.
B. On December 31 of the year under review, the branch mailed to the home office a check for 925,000 and a
notice that the branch had collected P4,380 on a home office account receivable. These items had not been
recorded by the home office. .
C. The branch was opened during the preceding year and its operating loss of P42,800 for the year was
capitalized by the branch as a start-up costs by the following entry:

Start-up Cost (Intangible Asset) 42,800


Income Summary ' 42,800

The account is not being amortized by the branch, and no entry was made by the home office to record the net
loss.

How much must be the adjusted balance of reciprocal accounts

A. P175,400
B. P192,600
C. P115,400
D. P132,600
Answer: D
Investment ln Branch Home Office
Unreconciled balance P136,020 P175,400
1) Unrecorded shipment 60,000
2) Unrecorded remittance ( 25,000)
2) Accounts collected 4380
3) Net loss ( 42,800) ( 42,800)
Adjusted balance P132,600 P132,600

7. On September 1, 2014, Ricky Company established two branches: Naga and Cebu City branches. The home
office transferred P80,000 worth of cash and P 350,000 worth of inventory to its Naga branch and instructed Naga
to transfer 3/4 of the goods and cash received to Cebu City. In addition, on November 1, 2014, shipments from
home office were received by Naga amounting to P125,000 and the branch paid freight costs amounting to
P6,500. 3/5 of the said shipments were sold to outsiders. On December 1, 2014, Naga transferred half of the
remaining November shipments from the home office to Cebu City, with Cebu City branch paying freight costs of
P 2,500. Had the merchandise been shipped from the home office to Cebu City branch, only P 1,900 worth of
freight would have been incurred. How much is the balance of the Cebu City branch account in the home office
books?
a. 349,400
b. 348,800
c. 206,200
d. 346,900
Answer: D
Solution:
Branch Current - Naga
9/1 430,000
9/1 (322,500)
11/1 125,000
12/1. (26,300)
Balance 206,200
Branch Current - Cebu, City
9/1. 322,500
12/1. 24,400
Balance 346,900

8. The Ray Company was organized in 2014. Shortly after opening its doors to the public t the main store, Ray
Company establish branch in another city. At the end of the second year of operations, the home office received
the following condensed income statement from the branch:

Revenues 140,000
Cost of Goods Sold 110,000
Gross Margin 30,000
Selling and Administrative expenses 25,000

Net Income 5,000


The management at the home office questioned the accuracy of these figures and assigned you the task of
verifying the branch data. Your review of the records uncovered the following facts:

1. The beginning of the year balance in Unrealized Profit to Branch was 6,000
2. During the period, the home office shipped goods to the branch taht had cost the main store P75,000.
However, your review of the branch receiving reports revealed that a number ofshipments from the home office
had been recorded twice by the branch accountant.
3. The branch is billed a uniform 25% above costand receives inventory only from te home office.
4. The branch ending inventory was correctly reported at a billed price of P21,750.
5. When reconciling reciprocal accounts, you found that the branch had not recorded 2,000 og services performed
by the home office and billed to the branch. All other selling and adminitrative expenses were correctly reported
by the branch.

Compute the correct net income of the branch.


a. 31,400
b. 33,400
c. 25,400
d. 11,000

Answer: A.
Correct Net Income
Revenue 140,000
COGS
Beg. Inv. at cost (60,000/25%) 24,000
Add: Shipments at cost 75,000
Less: End. Inv. at cost
(21,750/125%) (17,400) (81,600)
Gross Margin 58,400
Selling and Admin. Exp (25,000+2,000) (27,000)
Net Income 31,400

9. The after closing balances of Denise Corporation's home office and its branch at January 1,2014 were as
follows:
Home Office Branch
Cash P 7,000 P 2,000
Accounts Receivable-net 10,000 3,500
Inventory. 15,000 5,500
Plant Assets, net 45,000 20,000
Branch 28,000 -
Total Assets P 105,000 P 31,000
Accounts Payable P 4,500 P 2,500
Other liabilities 3,000 500
Unrealized Profit- Branch inv. 500 -
Home Office - 28,000
Capital Stock 80,000 -
Retained Earnings 17,000 -
Total Equities P 105,000 P 31,000

A summary of the operations of the home office and branch for 2014 follows:
Home Office sales: P 100,000, including ,P 33,000 to the branch. A standard 10% mark up on cost applies to all
sales to the branch. Branch sales to iys customers totaled P 50,000.
Purchases from outside entities: Home Office, P 50,000; Branch P 7,000
Collections from sales: Home Office P 98,000 (including P 30,000 from branch); Branch Collections; P 51,000
Payments on account; Home Office, P 51,000; Branch P 4,000
Operating expenses paid; Home Office, P 20,000; Branch P6,000
Depreciation on Plant Assets; Home Office, P 4,000; Branch P 1,000
Home Office expenses allocated to the branch, P 2,000

At December 31,2014, the Home Office inventory is P 11,000 and the Branch inventory is P 11,000 and the
Branch inventory is P 6,000 of which P 1,050 was acquired from outside suppliers.

The combined net income amounted to


a. 0
b. 4,550
c. 21,000
d. 25,550

Answer: D.
Sales 117,500
Cost of Sales (20,000+57,000-16,000) (60,450)
Gross Profit 56,550
Operating Expenses (26,000+5,000) (31,000)
Consolidated Net Income 25,550

10. Rea Company operates a branch in Pangasinan. Shipments are billed to the branch at cost. The branch
carries its own accounts receivable, makes its own collections and pays its own expenses. On December 31,
2015, the branch books shows the following balances:

Cash P 6,500
Home Office 33,000
Shipments from Home Office 133,000
Accounts Receivable 23,000
Sales 145,000
Expenses 11,500

The branch inventory on December 31, 2015 is P 16,500. What is the balance of the investment in Branch
account in the books of the Home Office on January 1, 2016?

a. P 33,000 b. P 52,000 c. P 50,000 d. P 53,000


Answer: c. P 50,000

Solution:

Home Office P 33,000


Net Income:
Sales P 145,000
Shipments from Home Office P 133,000
Inventory, End ( 16,500) 116,500
Gross Profit 28,500
Less: Expenses (11,500) 17,000
Investment in Branch P 50,000

11. Capistrano Corporation ships merchandise to its branch at 25% above cost. On its

books the branch shows a beginning inventory of home office merchandise amounting to P 20,000 and shipments
from Home Office of P 115,000. Its ending inventory of Home Office Merchandise is P 10,000. What amount
should the home office, Capistrano Corporation adjust the allowance for overvaluation of branch inventory
account?

a. P 20,000 b. P 25,000 c. P 100,000 d. P 50,000

Answer: b. P 25,000

Solution:
Home Office Merchandise, beg. P 20,000/ 125%= P 16,000 P 4,000
Shipments from Home Office 115,000/ 125%= 92,000 23,000
P 27,000
Less: Home Office Merchandise, end. 10,000/ 125%= 8,000 (2,000)
P 25,000

12. The Best acoustic. bills merchandise shipments its Cavite City branch at 125% of cost. The branch, in turn,
sells the merchandise it receives from home office at 25% above the billing price. On August @, 2016, all the
branch' s merchandise stock was destroyed by fire. The branch record that were recovered showed the following:

Inventory, Jan 1, 2016. P165,000


Shipments received from home office,
January to July(at billed price 110,000
Purchase at cost, from outside sources,
all resold at a 20% mark-up 10,200
Sales 169,000
Sales returns and allowances 3,750

The Best Co. will file an insurance claim. How much is the estimated cost of the merchandise destroyed by fire?

a. P120,000. c. P140,000
b. P130,000. d. P 150,000

Answer: A

Inventory, Jan. 1, at billed price. P165,000


Shipments received from home office at billed price 110,000
Cost of good available for sale at billed price. P275,000
Less: cost of good sold, ffrom home office
at billed price
Sales. P169,000
Less: sales return and allowance. 3,750
Sales price of merchandise purchased
from outsiders (P7,500 x 120%). 9,000
Net sales of merchandise acquired
from home office. P156,250
Multiplied by: Intercompany cost ratio. 100/125 125,000
Inventory, Aug. 1, 2016, at billed price. P150,000
Multiplied by cost ratio. 100/150

Merchandise inventory at cost destroyed by fire. P120,000

13. The home office of Irby Company bills merchandise to branches at 20% above home office cost. Information
taken from the accounting records of Kipp Branch is as follows:

Beginning Inventories (at billed prices) P17,000Shipmnlents from home office P42,500Ending inventories
P20,000Net loss P 1,500

The net income or net loss of Kipp Branch, based on home office cost of branch merchandise, is:

a. P7,900 net income


b. P9,400 net loss
c. P6,400 net income
d. P7,000 net income

Answer: C

Cost of sale P39,500


Mark up%. 20%
Total P 7,900
Less: net loss 1,500
Net income P 6,400

For question no. 14-15


Ruby Corporation shipped inventory to its Licab branch, costing P375,000 plus freight. Ruby bills inventory to its
branches at 120% of original cost, plus the actual amount of shipping charges. At the end of the year, the Licab
branch had resold 50% of the inventory from the home office. Shipping cost paid by Ruby were P2,000.

14. What amount should the inventory will be reported in the branch statement of financial position?
a. P197,500
b. P188,500
c. P377,000
d. P287,500

Ans. B
Cost P375,000
Add: shipping cost P2,000
Total P377,000
Multiply by : 50%
P188,500

15. Using the data above, what amount should the branch inventory from the home office be reported in the
financial position of Ruby Corp. As a whole?
a. P157,250
b. P188,500
c. P198,500
d. P377,000

Ans. A

Cost P375,000
Divided by: 120%
P312,500
Add: shipping cost P2,000
Total P314,000
Less: Resold P157,250
P157,250

16. On July 31, 2013, the home office in Manila establishes a sales agency in Bulacan. The following assets are
sent to the agency:
Cash (working fund to be operated under the imprest system) P22, 000
Samples of merchandise 36, 000

During the month of August, the following transactions occurred:


The sales agency submits sales order of P272, 000, sales per invoice was billed at P268, 000. Cost of sales to
customers is P124, 000.
Collections during the month amount to P58, 200, net of 3% discount.

Home office disbursements chargeable to the agency are as follows:


Furniture P40, 000.
Salaries for the month 21, 600
Annual rent of office space 36, 000

On August 31, the sales agency working fund is replenished. Paid vouchers submitted by the sales agency
amounting to P17, 925. Samples are useful until December 31, 2013, which at this time, are believed to have a
salvage value of 15% of cost. Furniture is depreciated at 18% per annum.

What is the total comprehensive income of the sales agency for the month of August?
a. P91, 425
b. P93, 225
c. P92, 955
d. P58, 425

Ans.: C
Solution:
Sales P268, 000
Sales discount(P58, 200 97%) x 3% 1800
Net sales 266, 200
Cost of expenses:
Cost of sales P124, 000
Salaries 21, 600
Rent expense(P36, 000 x 1/12) 3000
Expenses 17, 925
Samples(P36, 000 x 85%) x 1/5 6, 120
Depreciation(P40, 000 x 18% x 1/12)60 173, 245
Net income P92, 955
17. The account balances shown below were taken from the trial balances submitted to Bon-Apetit Corporation by
its Alabang Branch:
2015 2016
Petty cash fund P1500 P1500
Accounts receivable 43, 800 49,140
Inventory- 37, 170
Sales 173, 180 195, 120
Shipments from home (140% of cost) 107, 450 136, 080
Expenses 51, 260 57, 930
Accounts written off 1, 220 1, 920

All branch collections are remitted to the home office. All branch expenses are paid out of the petty cash fund.
When the petty cash fund is replenished, the branch debits appropriate expense accounts and credits Home
Office Current. The petty cash is counted every December 31, and its composition was as follows:
12/31/1512/31/16
Currency and coinsP580P860
Expense vouchers920640

The branch inventory on December 31, 2016 was P41, 370. The correct branch net income for 2016 was:
a. P3, 390
b. P3, 670
c. P41, 070
d. P41, 350

Ans.: D
Solution:
Sales P195, 120
Less: Cost of goods sold:
Inventory, 1/1/16, at cost (37, 170 x 100/140) 26,550
Add: Shipments, at cost (P136, 080 x 100/140) 97,200
Cost of goods available for sale P123, 750
Less: Inventory, 12/31/16, at cost(P41, 370 x 100/140) 29,550 94,200
Gross profit 100, 920
Less: Expenses(P57, 930 + P1, 920* P280** ) 59, 570
Correct branch net income P41, 350

*Direct write-off was used in recording doubtful accounts since there is no allowance account given in the trial
balance.
**There was P280 reduction on unreimbursed petty cash expense vouchers, incidentally, the entry for the
adjustment would be:
Petty cash 280
Expense 280

18.A branchs ending inventory of merchandise shipped by the home office and purchased from outside vendors
amounts to P100,000. The post closing balance in the unrealized gross profit in branch inventory account is
P12,000 due to the home office practice of shipping merchandise at 20% above cost. The merchandise
purchased from outside vendors contained in the ending inventory of the branch amounts to:

a. 38,000
b. 30,000
c. 14,000
d. 28,000

Ans. D

Unrealized gross profit 12,000/ 20% = 60,000


Unrealized gross profit 12,000
Total Inventory cost 72,000
Branch Ending Inventory 100,000
Less: Inventory cost 72,000
Total branch ending inventory 28,000

19. Items below are taken from the unadjusted trial balance of Anjie Company and its branch on December
31,2015.
Home office books Branch books
Shipment to branch 500,000
Allowance for overvaluation of branch inventory 99,900
Shipments from home office 590,000
Merchandise Inventory, Jan.1 64,500
Merchandise Inventory, Dec.31 48,750
Sales 600,000
Expenses 81,000

It is the companys policy to bill all branches for merchandise shipments at 30% above cost.
How much of the branch inventory on January 1 represents purchases from outsides?

a. 11,700
b. 42,000
c. 12,870
d. 27,600

Ans. D
Inventory, Jan.1 64.500
Less: (99,900-(590,000-500,000) 9,900
(9,900/30%) 33,000
Total Inventory 27,600

20. On December 3, 2016. the home or recorded a shipment of merchandise to its Davao Branch as follows,

Davao Branch ...................................................... 30,000


Shipments to Branch ............................... 25,000
Unrealized Profit in Branch inventory4,000
Cash (for freight charges).1,000

The Davao branch sells 40% of the merchandise to outside entities during the rest of December 2016. The books
at the home office and Kathy Office Supply are closed on December 31 of each year.

On January 5, 2017 , the Davao branch transfer 'half of the original shipment to the Baguio branch, and the
Davao branch pays P500 as the shipment.

At what amount should the 60% of the merchandise remaining unsold at December 31, 2016 be included in (1)
inventory of the Davao branch at Decmbr 31, 2016 and (2) the published balance sheet of Kathy office supply
Company at December 31, 2016 shows inventory at:
A. (1)P15,600(2) P18,000
B. (1)P17,400(2) P15,000
C. (1)P18,000(2)P15,600
D. (1)P18,400(2)P16,000

ANSWER: C
[1] Inventory of Davao branch at December 31, 2016:
Shipment from home office at billed price.. P 29,000
Multiplied by: ending inventory 60%
P 17,400
Add: freight in (1000 x 60%) 600
P 18,000 (c)
[2] inventory at published (external reporting) balance sheet at cost:
Shipment at cost P 25,000
Multiplied by: ending inventory 60%
P 15,000
Add: freight in (1000 x 60%) 600
P 15,600 (c)

21. Macoy starts a branch operation on January 1, 2013. Inventory costing P 72,000 is shipped to this branch at a
transfer price of P 100,000. Freight is an additional P6,000. The branch sells 70 percent of this inventory for P
110,000 and remits P 70,000 in cash to the home office. On Macoy's financial statements for this period, what
appropriate Cost of Goods Sold figure?

a. P 50,400
b. P 54,600
c. P 70,000
d. P 74,200

Answer: b

Shipment to branch, at cost P 72,000


Ending inventory, at cost (P72,000 x 30%) (21 600)
Cost of goods sold P 50,400
Freight (P6000 x P50,400/P72,000) P 4,200
Total P 54,600

22. Power Corporation shipped inventory to its Bacolod branch, costing P 375,000 plus freight. Power bills
inventory to its branches at 120 percent of original cost, plus the actual amount of shipping charges. At the end of
the year, the Bacolod branch had resold 50 percent of the inventory from the home office. Shipping costs paid by
Power were P 2,000. What amount should the inventory be reported in the branch's statement of financial
position?

a. P 187,500
b. P 188,500
c. P 226,000
d. P 377,000

Answer: b

Shipment to branch, at billed price P 375,000


Shipping cost P 2,000
Total cost P 377,000
Sold (50%) P 188,500
Inventory P 188,500
23. Lobster Trading bills its Nueva Ecija branch for shipments of goods at 25% above cost. All the close of
business on October 31, 20x6, a fire gutted the branch warehouse and destroyed 60% of the merchandise stock
stored therein. Therefore, the following data were gathered:

January 1 inventory, at billed price


Shipments from home office to Oct. 31
Not sales to Oct. 31

If undamaged merchandise recovered are marked to sell for P30,000, the estimated cost of the merchandise
destroyed by fire was:
a. P14,400 c. P24,000
b. 21,600 d. 27,500

Ans: b

Inventory, January 1 at billed price P 50,000


Add: Shipments from home office, at billed price 130,000
Cost of Goods Available for Sale at billed price P180,000
Divided by: Cost of Goods Available for sale
at sales price:
Net Sales P225,000
Add: Inventory before the fire:
Undamaged merchandise P 30,000
Divided by: Recovery Percentage 40% 75,000 300,000
Percentage of Billing Price to Selling Price 60%

Estimated cost of merchandise destroyed by fire:


Inventory before the fire at selling price (P30,000/40%) P 75,000
x: % of damaged merchandise 60%
Damaged merchandise at selling price P 45,000
x: % of Billing Price to Selling Price 60%
Damaged Merchandise at Billed Price P 27,000
x: Cost Ratio 100/125
Cost of Merchandise destroyed by fire P 21,600

24.On August 31, 20x6, a fire destroyed totally the rented bodega or stockroom of Adobo Company. The
following are some of the data of the company:

Merchandise Inventory, Dec. 31, 20x5 P110,000


For the period Jan. 1 Aug. 31, 20x6:
Purchases 560,500
Freight In 5,600
Purchases returns 10,200
Sales 695,000
Sales returns and allowances 7,500

Using a 20% gross profit rate, the cost of the merchandise lost in the fire was:
a. P 90,700 c. P88,400
b. 115,900 d. 63,200

Ans: b
Merchandise inventory, 12/31/20x5 P110,000
Add: Net Purchases
Purchases P560,500
Add: Freight-in 5,600
Total P566,100
Less: Purchase returns 10,200 555,900
Cost of Goods Available for Sale P665,900
Less: Cost of Goods Sold:
Net Sales (P695,000 P7,500) P687,500
x: Cost Ratio 80% 550,000
Merchandise inventory, 8/31/20x6
loss due to fire P115,900

25. The Lewis Co. bills merchandise shipments in its Quezon City branch at 120% of cost. The branch, in
turn, sells the merchandise it receives from the home office at 20% above the billing price. On July 31,
2016, all of the branchs merchandise stock was destroyed by fire. The branch records that were
recovered showed the following:

Inventory, January 1, 2016 (at billed price).. P330,000


Shipments received from home office,
January to June (at billed price). 220,000
Purchases, at cost, from outside sources,
All re-sold at a 25% mark-up.. 15,000
Sales.. 338,000
Sales return and allowances... 7,500
The Lewis Co. will file an insurance claim. How much is the estimated cost of that merchandise destroyed
by the fire?
a. P290,792
b. P259,792
c. P232,166
d. P230,166

ANSWER: (c)

Inventory, January 1, at billed price. P330,000


Shipments received from home office at billed price 220,000
Cost of goods available for sale at billed price.. P550,000
Less: Cost of goods sold, from home office at billed price
Sales.. P338,000
Less: Sales return and allowances.. 7,500
Sales price of merchandise purchased
from outsiders (P15,000 x 125%) 18,750
Net sales of merchandise acquired
From home office. P311,750
Multiplied by: Intercompany cost ratio 100/120 259, 792
Inventory, July 31, 2016, at billed price.. 290,208
Multiplied by: Cost ratio. 100/120
Merchandise inventory at cost destroyed by fire.. P232,166

26. Pascual Branch was billed by Home Office for merchandise at 140% of cost. At the end of its first
month, Pascual branch submitted among other things the following data:

Merchandise from Home Office (at billed price) P196,000


Merchandise purchased locally by branch. 80,000
Inventory, December 31 of which P14,000 are of
Local purchase.. 56,000
Net sales for month 360,000
The branch inventory at cost and the gross profit of the branch as far as the home office is concerned are:

Gross Profit Ending Inventory of Branch at Cost


a. P184,000 P154,000
b. P184,000 P176,000
c. P140,000 P176,000
d. P154,000 P140,000

ANSWER: (b)

Net sales P360,000


Less: Cost of good sold:
Purchases P 80,000
Shipments from home office, at cost
(P196,000 x 100/140) 140,000
Cost of goods available for sale P220,000
Less: Inventory, December 31
[(P56,000 P14,000) x 100/140 + 14,000] P 44,000 (b) 176,000
Gross profit P184,000 (b)

27. Ryder Corporation has one branch operation located 500 miles away from the home office. The branch office
sells merchandise which is shipped to it from the home office. The merchandise is transferred at cost but the
branch pays reasonable freight charges. The branch office makes sales and incurs and pays operating expenses.
At the end of the current accounting period the true adjusted balance for the home office account on the branchs
books and the branch office account on the home offices books is P500,000.

The following items may or may not be reconciling items.


The current year is 20x4.
The home office has shipped merchandise to the branch office which cost P1O, 000 and which incurs
P500 freight charges paid by the home office but charged to the branch. This merchandise is received by
the branch on January 5, 20x5.
The branch has transmitted P17, 000 in cash back to the home office as a partial payment on such
purchased merchandise. This cash is received by the home office on January 6th, 20x5.
The branch office returns some defective merchandise to the home office. The cost of the returned
merchandise is P750. The branch office pays P25 of freight costs which will be charged back to the home
office.
On December 1, 20x4, the home office sends a check for P25,000 to replenish the branchs working
capital. The check is received on January 4, 20x5.
The branch pays an advertising expense of P800 that should have been paid by the home office since it
applied to advertising fees incurred by the home office for its own benefit.
The home office allocated P12,000 of general and administrative expenses to the branch. The branch
had not entered the allocation as of the end of the year.
The home office pays insurance premiums on the branch store. The amount paid by the home office is
P1,000 but the branch erroneously records It as P776.00
Compute the unadjusted balances for the branch and home office accounts as of December 31, 20x4.

Home Office Current Branch Current


a. P481,425 P433,701
b. P500,000 P500,000
c. P452,276 P518,575
d. P518,575 P452,276
ANSWER: C

Home Office Books Branch Books


(Branch Current - Dr. Bal) (Home Office Current - Cr. Bal)
Unadjusted balance (SQUEEZED) P518,575 P452,276
Add (deduct) adjustments:
In transit 10,500
Remittance (17,000)
Returns ( 775)
Cash in transit 25,000
Expenses - HO ( 800)
Expenses - Branch 12,000
Error 224
Adjusted balance P500,000 P500,000

28. At December 31, 20x5, the following information has been collected by Maxwell Company's office and branch
for reconciling the branch and home office accounts.

The home office's branch account balance at December 31, 20x5 is P590,000. The branchs home office
account balance is P506,700.
On December 30, 20x5, the branch sent a check for P40,000 to the home office to settle its account. The
check was not delivered to the home office until January 3, 20x6.
On December 27, 20x5, the branch returned P15,000 of seasonal merchandise to the home office for the
January clearance sale. The merchandise was not received by the home office until January 6, 20x6.
The home office allocated general expenses of P28,000 to the branch. The branch had not entered the
allocation at the year-end.
Branch store insurance premiums of P900 were paid by the home office. The branch recorded the
amount at P600.
The correct balance of the reciprocal account amounted to:

a. P575,000
b. P535,000
c. P534,700
d. P501,000

ANSWER: B
Home Office Books Branch Books
(Branch Current - Dr. Bal) (Home Office Current - Cr. Bal)
Unadjusted balance P590,000 P506,700
Add (deduct) adjustments:
Remittance (40,000)
Returns (15,000)
Error by the branch 300
Expenses - Branch 28,000
Adjusted balance P535,000 P535,000

29.Tarlac Branch of Quezon City Company, at the end of its first quarter of operations, submitted the
following statement of comprehensive income:
Sales P300,000

Cost of sales:

Shipments from Home Office P280,000

Local Purchases 30,000

Total P310,000

Inventory at end 50,000 260,000

Gross Margin on Sales P40,000

Expenses 35,000

Comprehensive Income P5,000

Shipments to the Branch were billed at 140% of cost. The branch inventory as at September 30
amounted to P50,000 of which P6,600 was locally purchased. Markup on local purchases is 20% over
cost. Branch expenses incurred by Head Office amounted to P2,500.

On September 30, the inventory at cost and the net income realized by the home office from the Tarlac
branch operation are:

Branch inventory at cost Net income realized

a. P37,600 P72,600
b. P50,000 P55,000
c. P31,600 P 5,000
d. P37,600 P70,100

Answer: D

Acquired from Home Office:

Billed price (P50,000-P6,600) P43,400

Divide by billing percentage on cost 140% P31,000

Local Purchases 6,600

Branch inventory at cost (September 30) P37,600

Branch net income (P5,000 - P2,500 expenses) P2,500

Add: Overvaluation of Branch Cost of Sales:

Shipment from Home Office:


Billed price P280,000

Cost (P280,000/140%) 200,000 P80,000

Less: Inventory end

Billed price(P50,000 P6,600) P43,400

Cost (P43,400/140%) 31,000 12,400 67,600

Branch net income realized by Home Office P70,100

30. Nariza Company opened its Tuguegarao branch on January 1. Merchandise shipments from home
office during the month, billed at 120% of cost, is P125,000. Branch returned damaged merchandise
worth P15,620. On January 31, the branch reported a net loss of (P2,270) and an Inventory of P84,000.
What is the net income (loss) of the branch to be taken up in the books of the Home Office?
a. (P1,690)
b. P6,500
c. (P2,270)
d. P1,960

ANSWER: D
Rationale
Net income (loss) per branch books P(2,270)
Add: Realized profit from sale made by branch/
Overvaluation of cost of goods sold:
Beginning Inventory P -
Add: Shipments 125,000
Less: Returns 15,620
Cost of goods available for sale at billed price P109,380
Less: Ending Inventory, at billed price 84,000
Cost of goods sold at billed price P25,380
Multiplied by: Mark-up 20/120
Adjusted Branch Net Income P1,960

31. The ZG Corp. established its Bulacan Branch in January 2016. During its first year of operations,
home office shipped to its Bulacan branch merchandise worth P130,000 which included a mark up of
15% on cost. Sales on account totalled P250,000 while cash sales amounted to P80,000. Bulacan
reported operating expenses of P38,000 and ending inventory of P15,000, at billed price. In so far as the
home office is concerned, the real net income of Bulacan is:
a. P82,000
b. P147,000
c. P177,000
d. P192,000

ANSWER: D
Rationale
Sales (P250,000 + P80,000) P330,000
Less: Cost of goods sold:
Shipments from office P130,000
Less: Ending Inventory 15,000
Cost of goods sold at billed price P115,000
Multiplied by: Cost ratio 100/115 100,000
Gross Profit P230,000
Less: Operating Expenses 38,000
Net Income of the branch in so far as the
Home office is concerned P192,000

32. Selected accounts from the December 31,2016 trial balance of Sarang Trading Co. And its Baguio Branch
follow:
Debits Manila Baguio Branch
Inventory, January 1,2016 P 25,000 P 11,550
Baguio branch 58,300 -
Purchases 200,000 105,000
Freight in from home office - 5,500
Expenses 40,000 27,500
Credits
Home office P - P 53,300
Sales 160,000 150,000
Sales to branch 110,000 -
Allowance for overvaluation of
branch inventory at Jan. 1,2016 1,000 -

Additional Information:
1. The Baguio City branch gets all of its merchandise from the home office. The home office bills the goods
at cost plus a 10% mark-up. At December 31,2016 a shipment with a billed amount of P5,000 was still in
transit. Freight on this shipment was P300 and is to be treated as part of the inventory.
2. Inventories on December 31,2016, excluding the shipment in transit, follow:
Home office, at cost P30,000
Branch. At billed price(excluding freight of P520) 10,000
Compute the (1)net income of the home office from own operations, and (2)the net income of the branch in so far
as home office is concerned.
a. (1)P25,000; (2) P 770
b. (1)P20,000; (2) P10,470
c. (1)P20,000; (2) P 770
d. (1)P25,000; (2) P20,970
SOLUTION:
ANS: D
(1) Net income of the home office from own operations:
Sales P160,000
Less: Cost of goods sold:
Inventory, January 1,2016 P 25,000
Add: Purchases 200,000
Cost of goods available for sale P225,000
Less: Shipments to branch at cost 100,000
Cost of goods available for home office
Sale P125,000
Less: inventory, December 31,2016 30,000 95,000
Gross Profit P 65,000
Less: Expenses 40,000
Net Income P 25,000
(2) True Branch Income:
Sales P150,000
Less: Cost of goods sold:
Inventory, Jan.1,2016, at cost
(P11,550-P1,000 mark-up) P 10,550
Add: Purchases from home office, at cost
(P105,000+ P5,000 in transit) x
100/110 100,000
Freight-In (P5,500+P300 freight-in
transit) 5,800
Cost of goods available for sale P116,350
Less: Inventory, Dec. 31,2016, at cost from
Home Office:(P10,400 + P5,000)
X 100/110 P14,000
Add: Freight-In (P520+P300) 820 14,820 101,530
Gross Profit P 48,470
Less: Expenses 27,500
Net Income of the branch in so far as the
Home Office is concerned P 20,970

33. On December 31,2016, the Investment in Branch account on the home offices books has a balance of
P175,000. In analyzing the activity in each of these accounts for December, you find the following differences:
1. A P14,000 branch remittance to the home office initiated on December 28,2016, was recorded on the
home office books on January 2, 2017.
2. A home office inventory shipment to the branch on December 27,2016 was recorded by the branch on
January 4,2017; the billing of P26,000 was at cost.
3. The home office incurred P13,500 of advertising expenses and allocated P5,500 of this amount to the
branch on December 15,2016. The branch has not recorded this transaction.
4. A branch customer erroneously remitted P3,600 to the home office. The home office recorded this cash
collection on December 23,2016. Meanwhile, back at the branch, no entry has been made yet.
5. Inventory costing P51,600 was sent to the branch by the home office on December 11,2016. The billing
was at cost, but the branch recorded the transaction at P40,800.
Compute the balance as of December 31,2016:
Unadjusted Balance Adjusted
Of The Home Office Account Balance Of The Reciprocal Account
a. P84,300 P143,000
b. P122,300 P 96,000
c. P151,200 P139,200
d. P122,300 P161,000
Solution:
ANS: D
Investment in Home Office
Branch Account Current
Unadjusted balance(s), December 31,2016 P175,000 P122,300*
Add (deduct); adjustments:
1. Branch remittance not yet recorded by
the home office in 2016 ( 14,000)
2. Shipments not yet recorded by the
Branch in 2016 26,000
3. Unrecorded branch expenses 5,500
4. Branch customers remittance recorded
by the home office nut not yet recorded
by the branch ( 3,600)
5. Erroneous recording of branch shipments
(P51,600 P40,800) 10,800
Adjusted Balance(s) December 31,2016 P161,000 P161,000
*The P52,800 is computed by simply working back with P90,000 adjusted balance as the starting point

34. Nueva Ecija Branch of Malabon Company, at the end of its first quarter of operations has the following income
statement:

Sales...................................................................................................... P650,000
Cost of Sales:
Shipments from Home Office ......................................................P285,000
Local Purchases.......................................................................... 50,000

Total......................................................................................... P335,000
Inventory end.......................................................................... 80,000 255,000

Gross profit on sales............................................................. P395,000


Expenses.............................................................................. 150,000

Net Income.............................................................................. P245,000

Shipments to the branch were billed at 140% of cost. The branch inventory at September 30 amounted to
P50,000 of which P6,600 was locally purchased . Mark up on local purchases, 20% over cost. Branch expenses
incurred by head office amounted to P2,500 not yet recorded by the branch. In the combined income statement,
true branch net income:

A.P245,000
B.P314,029
C.P311,529
D.P314,029

Answer: C

Branch net income as reported................................................................................. P245,000


Add(Deduct):
Overvaluation of cost of goods sold/realized profit
From sales made by branch:
Shipments from home office...........................P285,000
Less:Ending inventory,at billed price
(50,000-6600)........................................... 43,400

Cost of goods sold from home office at


Billed price..............................................P 241,600
Multiplied by: Mark-up.................................... 40/140 69,029
Unrecorded branch expenses............................................. (2,500)

True Branch Net Income......................................................... P 311,529(C)

i.35.Which represents the proper journal entry for a periodic inventory system that should be made on the books
of the home office when goods that cost the home office $100,000 to manufacture are shipped to a branch at a
transfer price of $125,000 and the billed price is not recorded in the shipments to branch account?
A. Branch office $100,000

Shipments to branch $100,000

B. Branch office $125,000

Shipments to branch $125,000

C. Branch office $125,000

Shipments to branch $100,000

Unrealized profit 25,000

D. Shipment to branch $100,000

Unrealized profit 25,000

Shipments from home office $125,000

ii36.Whichrepresents the proper journal entry for a periodic inventory system that should be made on the books of
the branch when goods that cost the home office $100,000 to manufacture are shipped to the branch at a price of
$125,000?

A. Shipments from home office $100,000

Home office $100,000

B. Shipments from home office $125,000

Home office $125,000

C. Shipments from home office $125,000

Unrealized profit $ 25,000

Home office 100,000

D. Shipments to branch $100,000

Unrealized profit 25,000

Shipments from home office $125,000


i. 35.REQUIRED: The home office journal entry to reflect merchandise shipments at cost plus a markup.
DISCUSSION: (C) When goods are shipped from a home office to a branch at a transfer price that reflects
original cost plus a markup, the branch must record the shipment at the transfer price. The home office most
often reflects the shipments to branch at original cost. To maintain a reciprocal relationship between the
home office and the branch office accounts, an unrealized profit in branch inventories account reflects the
markup.
Answers (A) and (B) are incorrect because neither reflects the unrealized profit. Answer (A) is incorrect
because the branch office should be recorded at $125,000. Answer (B) is incorrect because the shipments to
branch should be recorded at original cost of $100,000. Answer (D) is incorrect because it is the work sheet
entry necessary to eliminate this intercompany transaction in the preparation of the financial statements.
Answer () is incorrect because

ii. 36.REQUIRED: The journal entry on the branch book s to reflect the receipt of merchandise shipments at a
transfer price that reflects cost plus a markup.
DISCUSSION: (B) In a periodic system, when merchandise is received by a branch from the home office, the
merchandise should be reflected as a shipment from the home office in the amount of the transfer price, with
a corresponding entry to the home office account to indicate the equity of the home office in the net assets of
the branch.
Answer (A) is incorrect because the shipments should be reflected at the transfer price. Answer (C) is
incorrect because the home office equity should be reflected at the transfer price. Answer (D) is incorrect
because it is the worksheet entry used to eliminate this intercompany transaction in the preparation of the
enterprises financial statements.

37. Error! Main Document Only.A gain should be reported on an acquisition if:

a. The fair value of the consideration paid is less than the book value of the net assets acquired.
b. The fair value of the consideration paid plus the present value of any earnings contingency is less than the
book value of the net assets acquired.
c. The fair value of the consideration paid is less than the fair value of net assets acquired plus the fair value
of identifiable intangibles acquired.
d.The fair value of the consideration paid plus the present value of any earnings contingency is less than the
fair value of identifiable net assets acquired.

ANS: D

38. In accounting for branch transactions, it is improper for home office to,
A. Credit cash received from a branch to the investment in Branch ledger account.
B. Maintain common stock and retained earnings ledger accounts for only the home office
C. Debit shipments of merchandise to the branch from the home office to the Investment in Branch Ledger
account.
D. Credit shipments of merchandise to the branch to the sales ledger account.
Answer: D

39.Aca, Inc. has several branches. Goods costing P10,000 were transferred by the head office to Cebu
Branch with the latter paying P600 for freight cost. Subsequently, the head office authorized Cebu
Branch to transfer the goods to Davao branch for which the was billed fot the P10,000 cost of the goods
and freight charge of P200 for the transfer. If the head office had shipped the goods directly to Davao
branch, the freight charge would have been P700. The P100 difference in freight cost would be
disposed of as follows:

a. Considered as savings
b. Charged to Cebu Branch
c. Charged to Davao Branch
d. Charged to the Head Office

ANS: D

In arriving at the cost of the merchandise inventory at the end of the period, freight charges are
properly recognized as part a part of the cost. But a branch should not be charged with
excessive freight charges when, because of indirect routing, excessive cost are incurred. Under
such circumstances, the branch acquiring the goods should be charged for no more than the
normal freight from the usual shipping point. The office directing the interbranch transfers are
responsible for the excessive cost should absorb the excess as an expense because it
represents management mistakes or inefficiencies.

40.Jayhawk Company has numerous branches in the state of Kansas. The home office purchases merchandise
and makes shipments to branches from a central warehouse at the request of branch managers. Which of the
following would be an improper accounting practice?

a. The Investment in Branch ledger account is debited in the accounting records of the home office
when merchandise is shipped to a branch, and the Shipments to Branch account is credited (assume
use of the periodic inventory system).
b. The home office debits Trade Accounts Receivable and credits Sales when merchandise is shipped
to a branch.
c. Cash received from a branch is credited to the Investment in Branch ledger account by the home
office.
d. Only the home office maintains a Common Stock ledger account and a Retained Earnings account.
Answer: B

Anda mungkin juga menyukai